*NURSING > EXAM > Pediatric Nurse Practitioner Exam 1|699 Questions with Verified Answers,100% CORRECT (All)

Pediatric Nurse Practitioner Exam 1|699 Questions with Verified Answers,100% CORRECT

Document Content and Description Below

Pediatric Nurse Practitioner Exam1|699 Questions with Verified Answers The concept of health promotion consists of efforts to prevent rather than to cure disease or disability. This description bes... t describes: a. tertiary prevention b. secondary prevention c. primary prevention d. morbidity prevention - CORRECT ANSWER c Standards for well child care and health promotion are set forth by the: a. Task Force on Preventative Services, US Department of Health and Human services b. American Academy of Family Physicians c. American Academy of pediatrics d. Healthy People 2010 and 2020 - CORRECT ANSWER c A 15 yo female comes to the clinic for a health maintenance visit. The patient reports no complaints. She does well in school, has many friends, and gets along well with her parents and two siblings. She has recently become sexually active with her boyfriend of 6mo. The physical exam is normal for her age. What screening, lab tests, and/or immunizations should the PNP order? a. Vision, hearing, urinalysis, STD screen, pelvic exam, and immunizations as needed b. STD screen, pneumococcal vaccine, - CORRECT ANSWER a Which of the following is the best written chief complaint? a. 5yo asian female with itchy red rash b. 1 mo with difficulty feeding c. 15yo african american female complaining of lower abdominal pain for 3 days d. 12yo caucasian with itchy eye for 1 week - CORRECT ANSWER c The past history should include a. family history b. social hx c. symptom analysis d. hospitalization - CORRECT ANSWER d The family history should include: a. the health status of the paternal grandparents b. the living conditions of the family c. the work status of the parents d. school information - CORRECT ANSWER a Which of the following statements would be considered subjective data obtained from the client during the review of systems for the skin? a. the skin appears dry & excoriated b. a lesion is noted on the lateral aspect of the right leg c. there is moderate acne on the forehead and back d. the patient denies changes in skin texture - CORRECT ANSWER d The PNP is completing the ROS on a 4mo. What response from the mother might indicate a cardiac problem in the infant & require a more thorough history? a. the baby gets the hiccups often. b. it takes the baby over 30min to complete a bottle c. the baby's heart seems to beat rapidly sometimes d. sometimes the baby sounds congested - CORRECT ANSWER b The PNP is obtaining a review of systems on a healthy 8yo girl. It would be important to include: a. the date of last breast self-exam. b. any limitations related to sports participation. c. the last hematocrit and hemoglobin d. the last lead screen - CORRECT ANSWER b The mother of a 3yo states that her son was completely toilet trained at 18mo. The PNP explores further, asking: a. how the mother did the toilet training b. if the child is dry at night and fully trained for urine and stool day and night c. at what age did the mother begin toilet training the other child. d. if the child gets up at night to use the bathroom - CORRECT ANSWER b Jasmine, a 5-month-old infant is in the clinic for a well child visit and is sleeping in her mother's arms. What should the PNP do first? a. Examine the ears, since this is uncomfortable for the infant. b. Quickly give any needed immunizations. c. Begin with assessment of the head and proceed from head to toe. d. auscultate the heart & lungs while the infant is sleeping. - CORRECT ANSWER d The mother of a newborn brings the baby to the clinic because she is afraid something is wrong with her baby ("his head is so big"). The PNP responds that: a. "this is normal. The head of a newborn is proportionately large to the rest of the body" b. "Yes, this does appear abnormal. I will make an appointment for the baby to be seen by a neurologist." d. "your head appears large so it is probably genetic." - CORRECT ANSWER a A healthy 12yo female is at the clinic for a well checkup. On physical examination, a marked elevation of the right scapula and right thoracic hump and spinal curve abnormally are noted. Spinal films indicate 20 degree curve. The PNP should: a. refer the patient to an orthopedist b. monitor the patient every 3 mo until menarche c. refer the patient for physical therapy d. recommend bedrest and back brace - CORRECT ANSWER a What is the % of risk an autosomal dominant gene is being passed on to each offspring? a. 25 b. 50 c. 100 d. none - CORRECT ANSWER b A 13yo female is at the clinic for a routine check up. What presenting symptoms might alert the PNP to the presence of a possible genetic disorder? a. <10% on growth chart, Tanner I, and learning difficulties b. Precocious puberty, developmental delays, gifted piano player c. Lactose intolerance, frequent ear infections, speech delays d. Peanut allergy, right-sided weakness, and exhibits bullying behavior - CORRECT ANSWER a The parents of a child with cystic fibrosis are considering another pregnancy. They want to know what their chances are of having another child with cystic fibrosis. The PNP explains that cystic fibrosis is an autosomal recessive disorder and that each conception of carrier parents has a: a. 25% chance of being affected b. 50% chance of being affected c. 75% chance of being affected d. 100% chance of being affected - CORRECT ANSWER a The parents of a son with hemophilia are considering another pregnancy. The mother has been identified as a carrier of the hemophilia gene. What chance does each female offspring have of having hemophilia. a. 100% b. 50% c. 25% d. none - CORRECT ANSWER d A male with Vitamin D-resistant rickets asks what his chances are of passing his disease to his parents. The PNP answers: a. all sons will be affected by only 25% daughters b. all daughters are affected by only 25% sons c. 50% of all offspring will have the disease d. all daughters will be affected but no sons. - CORRECT ANSWER d The PNP is following a 15-year-old male adolescent with consistent blood pressure readings of 132 to 138/84 to 86 mm Hg, which is classified as significant hypertension. After performing a workup, the PNP determines that the adolescent has primary hypertension. The most judicious recommendation for therapy is: A) Diet and exercise counseling and referral to a specialist B) A diuretic, low-salt diet, exercise, and counseling by a dietician C) Perform an extensive family history to determine other - CORRECT ANSWER a A 2-week-old neonate is brought to the clinic with excessive lacrimation of both eyes. The eyes are clear, with no areas of redness, but the corneas appear hazy. The PNP explains to the parents the possible diagnosis and treatment as follows: A) Congenital glaucoma requiring an ophthalmology referral B) Corneal foreign body requiring fluorescein staining and removal C) Chalazion requiring treatment with antibiotic eyedrops D) Allergic conjunctivitis requiring treatment with Benadryl eyedrops - CORRECT ANSWER a On examination of a 3-month-old infant the PNP is unable to elicit a red reflex in the right eye. Previous examinations failed to note the presence or absence of a red reflex. The PNP's response is to: A) Reassure the parents that this is not a problem because the infant has dark eyes B) Note this finding in the infant's chart and check again in a few months C) Refer the infant to an ophthalmologist to rule out retinal trauma D) Immediately refer the infant to a pediatric ophthalmologist to rule - CORRECT ANSWER d A 4-year-old child with chickenpox has vesicles on the skin of the right eyelid. The child complains of eye pain and blurred vision. The PNP should treat the condition by: A) Applying cool compresses to the eye and lesions B) Prescribing eyedrops containing steroids to decrease inflammation and pain C) Prescribing polyspoium ophthalmic ointment for the secondary bacterial infection D) Immediately referring the child to an ophthalmologist - CORRECT ANSWER d A 9-month-old infant was diagnosed with sickle cell disease shortly after birth. The mother telephones the PNP to report that the infant has a fever of 103.2°F. The best response to the mother is: A) "Take the infant to the emergency room immediately." B) "Administer a dose of ibuprofen, and call back in 6 hours if the fever continues." C) "Give extra fluids and acetaminophen, and call back tomorrow if the fever continues." D) "Give extra fluids and acetaminophen, and bring the infant to the cl - CORRECT ANSWER a A 5-year-old child has sudden onset of nonblanching purpuric lesions scattered over the body and petechiae scattered over the neck and shoulders. The mother reports that the child has been healthy, except for a cold a few weeks ago. The child is not taking any medications. Physical examination reveals a healthy, afebrile child with no other significant findings. The laboratory data show a hemoglobin level of 12.5 g/dL, white blood cell count of 6500/mm3, and platelet count of 20,000/mm3. Based o - CORRECT ANSWER a A 9-year-old child is brought to the clinic for the evaluation and treatment of a rash. While examining the child, the PNP detects a speech dysfluency. The most appropriate plan of treatment should include: A) Treating the rash and scheduling a well-child visit to address the speech problem B) Treating the rash and making a referral for speech, language, and hearing evaluation with a speech pathologist C) Treating the rash and making no referral because it is too late for speech therapy to be be - CORRECT ANSWER b A neonate is diagnosed with trisomy 21 based on karyotyping. What type of follow-up will the infant need? A) Echocardiography; thyroid function tests at birth, 3 months, and yearly thereafter; a CBC; and an audiology consult B) An ophthalmologic evaluation, neck x-ray films by age 3 years, and referral to early intervention and parent education C) Karyotyping, echocardiography, audiologic and ophthalmologic evaluation, and referral to early intervention and parent education D) Karyotyping; echoc - CORRECT ANSWER a The parents of a 9-year-old with primary enuresis request information regarding treatment options. When discussing alternative treatments for enuresis, the PNP offers pertinent information to help the child and parents make an appropriate decision about which treatment would be best. The family should be told: A) The alarm is the safest therapy, but the relapse rate is about 10% B) Pharmacologic therapy has the lowest relapse rate C) Motivational therapy should be used after pharmacologic treatm - CORRECT ANSWER d A parent asks for suggestions in helping a 6-year-old child who wets the bed. What intervention would the PNP recommend for this child diagnosed with primary nocturnal enuresis? A) Use a "wet night" calendar to mark the dates of wetting accidents B) Allow the child to take care of changing wet clothes and linens C) Criticize the child when a wetting accident has occurred D) Praise any progress made by the child - CORRECT ANSWER d A 15-year-old adolescent has pallor and fatigue. The CBC results are consistent with iron-deficiency anemia. What would be appropriate information to give this adolescent? A) Antacids increase the absorption of iron B) Dairy foods are good sources of dietary iron C) Juices fortified with vitamin C inhibit the absorption of nonheme iron D) Tannin-containing products, such as tea, inhibit the absorption of nonheme iron - CORRECT ANSWER d The PNP examines a 4-year-old child who is home-schooled and immunization-delayed. The child has pain when chewing; a fever; and enlarged, tender salivary nodes. The PNP diagnoses mumps and informs the mother that mumps is contagious for: A) 7 days before the onset of symptoms B) 10 days before the onset of symptoms C) 1 to 2 days before and 5 days after D) As long as 3 days after the onset of symptoms - CORRECT ANSWER a A 10-year-old child is brought to the clinic after jumping over a rusty fence and receiving a puncture wound to the buttocks. Immunizations are up to date with last tetanus given when child was 4 years old. The PNP cleanses the wound and: A) Administers a tetanus vaccine and prescribes penicillin for 14 days B) Starts prophylactic antibiotics C) Applies a sterile dressing D) Administers tetanus vaccine and educates family regarding signs of wound infection - CORRECT ANSWER d The mother of a 10-week-old calls the office because the infant is passing hard stools. What suggestions would the PNP offer a mother? A) "Do nothing. Time will take care of the problem." B) "Hold the infant upright so gravity can help the stool pass." C) "Give a little extra water with added pasteurized apple or prune juice." D) "Add honey to a 4-oz bottle of water." - CORRECT ANSWER c While examining a 3-year-old child for a well-child visit, labial adhesions are noted. The PNP discusses with the mother: A) Concern of sexual abuse B) Proper hygiene C) Causes of UTI D) Surgical treatment of the adhesions - CORRECT ANSWER b In providing anticipatory guidance to the parents of a 5-year-old child with diabetes, the PNP should teach them to recognize the symptoms of sweating, hunger, drowsiness, and confusion. These are all symptoms of: A) Diabetic ketoacidosis B) The dawn phenomenon C) Hypoglycemia D) The Somogyi phenomenon - CORRECT ANSWER c An 18-month-old child is brought to the office for irritability, nightmares, and generally being tired for the past few nights. The child has had an upper respiratory tract infection for which the grandmother has administered an over-the-counter medication for rhinorrhea. The child is afebrile and alert, with obvious clear rhinorrhea. The PNP: A) Orders sinus radiologic studies B) Prescribes amoxicillin C) Discusses side effects of decongestants and antihistamines D) Suggests that the grandmothe - CORRECT ANSWER c A 16-year-old adolescent has evidence of weight loss. The PNP notes on the chart documentation of frequent loose stools for 2 weeks. The differential diagnosis includes hyperthyroidism. What additional clinical finding would suggest this diagnosis? A) Dry mouth B) Hypotension C) Coarse hair D) Heat intolerance - CORRECT ANSWER d A child is brought to the office with irritability, agitation, muscle pain, and cramping in the large leg muscles. On examination a target lesion consisting of an erythematous ring around a pale center is noted. The child has been playing in the garage and does not recall seeing or being bitten by an insect. The child's face is flushed, and there is some diaphoresis. The PNP suspects: A) An allergic reaction to flea bites B) A black widow spider bite C) A brown recluse spider bite D) Ingestion o - CORRECT ANSWER b A 3-year-old child is brought to the clinic with noisy respirations. Which of the following signs or symptoms would be an indication for immediate referral and hospitalization? A) Mild restlessness B) Stridor at rest C) Fever of 102°F D) Slightly elevated white blood cell count - CORRECT ANSWER b A 10-year-old child is brought to the office with coughing, abdominal pain, splinting on the right side, fever, headache, and chills. The child appears to be in severe distress. There are decreased breath sounds, crackles, and dullness to percussion in the area of the right middle and right lower lobe. The diagnosis of pneumonia is made. The PNP orders an anteroposterior and lateral view of the chest and: A) Sputum stain B) Blood culture C) Complete blood cell count and blood culture D) Blood ga - CORRECT ANSWER c A 12-year-old child comes to the school-based clinic complaining of unilateral chest pain that radiates to the back and abdomen. On physical examination, tenderness over the costochondral junction is noted. The chest pain is most likely caused by: A) A fractured rib B) Gastric reflux C) Costochondritis D) Pneumonia - CORRECT ANSWER c A 10-month-old infant is noted to have cranial and facial asymmetry during a well-child visit. The PNP should: A) Observe the infant, and reevaluate cranial shape at the 1-year well visit B) Refer immediately to a neurologist C) Obtain a cranial radiologic film, CT scan of the skull, or both D) Perform a Denver Developmental Screening Test - CORRECT ANSWER c In evaluating a 7-year-old child for ADHD, the PNP questions the parents about reported behavior at school. Of the following symptoms, which would be most descriptive of ADHD? A) The child will not listen to direction or when spoken to, submits messy papers with careless mistakes, runs about or climbs excessively, has difficulty sustaining attention, and exhibits intrusive behavior B) The child has a short attention span, has difficulty listening, has difficulty organizing tasks, and interrupts - CORRECT ANSWER a A 7-year-old child with cerebral palsy is brought to the clinic. The child is unable to walk but is able to use a wheelchair. The child uses the arms and hands to eat and perform self-care tasks. The PNP diagnoses this child with: A) Spastic diplegia B) Spastic quadriplegia C) Athetoid cerebral palsy D) Spastic hemiplegia - CORRECT ANSWER a A mother brings a 1-year-old child with Down's syndrome to the clinic. She is concerned that the child has been congested for 10 days. The child was born at home and has never been seen in this practice. The mother reports that the child is doing well, has no problems, and is still being breastfed. The child has never been examined by a doctor. The PNP's initial plan of care for the child is: A) Treat the congestion, and start immunizations B) Start immunizations, and refer the parents for genet - CORRECT ANSWER a The PNP has completed vision screens on the children who attend a day care and preschool center. Which of the following children should be referred for further testing? A) A 3-year-old child (tested with a Sjögren hand chart) with the following results: right eye, 20/40; left eye, 20/20 B) A 4-year-old child (tested with an HOTV chart) with the following results: right eye, 20/40; left eye, 20/40 C) A 4-year-old child (tested with a Snellen E chart) with the following results: right eye, 20/20; - CORRECT ANSWER a An infant brought to the clinic is diagnosed with failure to thrive (FTT), persistent growth deficiency of the head, and delayed development. Some unusual facial abnormalities are noted. The mother has a history of consuming large amounts of alcohol. The PNP suspects fetal alcohol syndrome (FAS) and refers the infant: A) To a First Steps Program or Birth to Three Program B) For consultation with a neurologist C) For a genetic workup D) For consultation with an endocrinologist - CORRECT ANSWER a The PNP is assessing a neonate in the nursery. The cremasteric reflex is absent, and the right testicle cannot be palpated in the scrotum. The PNP should: A) Order a sonogram B) Consult with the physician C) Refer the neonate to the urology clinic D) Not allow the neonate to be discharged from the hospital - CORRECT ANSWER b The PNP is in the nursery to examine a neonate. The results of the neonatal screen are noted in the chart. The T4 level is low, and the TSH level is elevated. The PNP should: A) Consult with the physician, and repeat the tests B) Discharge the neonate, and schedule a follow-up visit in the office in 1 week C) Begin the neonate on thyroxine based on the test results D) Examine the neonate for signs of hypothyroidism - CORRECT ANSWER a A 7.5-year-old Hispanic child has prepubertal pigmented pubic hair. No other secondary sex characteristics are noted. The mother is concerned that the child will soon begin menstruating. What advice would the PNP give the mother? A) "Premature sexual development is common in Hispanic girls, and no treatment is required." B) "Premature adrenarche can be a normal variant in Hispanic girls, and no treatment is required, but we will monitor the changes at each annual visit." C) "Central precocious p - CORRECT ANSWER b An 11-year-old girl is brought to the office for an annual well-child visit. When discussing the onset of puberty with the preadolescent and the mother, which information would the PNP provide? A) "Pubic hair develops before breast buds." B) "Girls have their greatest linear growth after menses." C) "The average time from breast buds to menarche is 1.5 to 2.5 years." D) "The average age of onset for menarche is 10 to 12 years." - CORRECT ANSWER c The mother of a 2-year-old child is concerned about the child's fear of the dark. The PNP responds that: A) Fears of this type are a normal developmental process B) Young children with fears tend to have phobias as adults C) Fears that interfere with social development resolve as the child develops D) Parents should not allow the child to withdraw from a fearful situation - CORRECT ANSWER a The mother of a 2-year-old child is concerned about the child's thumb sucking during the day when not playing and at night while sleeping. The PNP suggests that: A) This is not a problem unless the thumb sucking continues past age 3 years B) The mother should obtain an orthodontic device for the child's mouth to discourage the sucking behavior C) The parents should send the child to time-out when caught thumb sucking during the day D) The parents should ignore the behavior because it is not harm - CORRECT ANSWER d A 3-year-old child is brought to the clinic for a well-child visit. The mother requests help with toilet training. The child refuses to use the toilet to have a bowel movement and squats, wearing a diaper, in a secluded corner to pass a stool. The parents offer encouragement through rewards and the "big kid" approach to using the toilet, but nothing seems to help. The PNP should respond that: A) Refusal to use the toilet after age 2.5 years is the result of caregivers (parents, day care workers, - CORRECT ANSWER d The mother of a 2-year-old child requests information regarding toilet training readiness. The PNP tells the mother that the most important factor in determining readiness to toilet train is that the child: A) Is not bothered by wet or soiled pants B) Is strong willed and shows an interest in the potty C) Is able to communicate needs and follow directions D) Wakes up from naps with soaked diapers - CORRECT ANSWER c The PNP counsels the mother of an infant about possible sibling rivalry a 2-year-old sibling may exhibit. To decrease sibling rivalry the PNP suggests that the parents should: A) Tell the older child that he or she is no longer a baby and can wait until his or her needs can be met B) Not allow the older child to touch the infant under any circumstances C) Not tolerate any regression on the part of the older child D) Set aside extra time to spend with the older child - CORRECT ANSWER d A mother brings her 2-year-old child to the clinic for a well-child visit. The mother's chief complaint is the child's rebellious behavior. The mother says that she has tried time-out, yelling, and spanking but nothing has worked. The child has never slept through the night and still gets a bottle in the middle of the night. In the examination room the PNP observes as the child repeatedly gets into the mother's purse when she is not paying attention, despite the mother telling the child "no." Ba - CORRECT ANSWER a A new parent asks why a light is shined in the infant's eyes. The PNP tells the parent that the infant must be checked for a red reflex in each eye to rule out: A) Glaucoma B) Strabismus C) Cataracts D) Amblyopia - CORRECT ANSWER c The mother of a 1-year-old child requests information about the child's speech and language development. The PNP explains that vocabulary acquisition may be slow during certain periods and that a vocabulary spurt is most common between ages: A) 10 and 12 months B) 12 and 14 months C) 16 and 24 months D) 2 and 3 years - CORRECT ANSWER c The mother is concerned that her 3-year-old child has begun stuttering over the past 4 to 6 weeks. The PNP should: A) Refer the child to a speech therapist and audiologist B) Instruct the mother to remind the child to speak more slowly and clearly when beginning to stutter C) Explain that dysfluency is normal at this age but should be reevaluated if it is persistent or worsens D) Instruct the mother not to make eye contact with the child while the child is speaking to decrease anxiety - CORRECT ANSWER c At a physical examination for Head Start, a mother reports being concerned about her 4-year-old child's speech. She worries that the child's speech is unclear and that the child is not saying enough words. The PNP explains to the mother that the child should: A) Have a vocabulary of approximately 1500 words B) Have speech that is 100% intelligible to strangers C) Speak in sentences of three to four words D) Not have problems with dysfluency (stuttering) - CORRECT ANSWER a A 2-year-old child is screened for lead poisoning. The blood level is 16 mg/dL. The PNP discusses nutritional needs, including a diet high in: A) Protein and calcium B) Vitamin C and iron C) Fat and Zinc D) Calcium and iron - CORRECT ANSWER d In counseling a parent regarding possible lead poisoning in the child, the PNP states that the most common source of lead is: A) Drinking water B) Soil C) Lead-based paint D) Batteries - CORRECT ANSWER c The 19-year-old mother of an infant in the PNP's care has just found out that she is expecting her second child. The mother's first prenatal visit is scheduled soon. The mother smokes two packs of cigarettes a day. What action should the PNP take? A) Inform the mother about the risks to herself and to her children from secondhand smoke B) Sign the mother up for smoking cessation classes C) Let the obstetrician/gynecologist assess and manage this issue D) Write the mother a prescription for nicot - CORRECT ANSWER a A 17-year-old adolescent is anticipating marriage after completing high school. The adolescent does not have sickle cell disease and is not a carrier of the trait, but the intended spouse has sickle cell trait. The adolescent asks if it is possible that any of their future children will have sickle cell disease. The adolescent is told that there is a 50% chance that: A) Their children will have sickle cell disease B) Their children will have sickle cell trait C) Only their sons will have sickle - CORRECT ANSWER b A 12-year-old child is brought to the office for a yearly physical. The child states that he has recently noticed an enlargement of his testes and scrotum. When counseling the boy as to what to expect next in pubertal development the PNP tells him: A) The penis will grow in length B) The penis will grow in width C) Facial hair will appear D) Changes in voice will occur - CORRECT ANSWER a An 11-year-old child is concerned about breast size. About 6 months ago when her breast development began, she noticed that her breasts were different sizes. The PNP responds: A) "This is abnormal and you need to see a specialist." B) "Menarche will appear soon." C) "A mammogram is needed." D) "The breasts will become closer to the same size within a few years." - CORRECT ANSWER d The PNP has prescribed salmeterol (Serevent) by inhalation for an asthmatic. The family should be warned about which of the following side effects that occur with this medication? A) Tachycardia and headache B) Weight gain and acne C) Nausea and vomiting D) Dysrhythmia and dry mouth - CORRECT ANSWER a The most appropriate advice the PNP can give the mother of a 10-month-old who weighs 21 lb concerning the use of car seats is: A) "A child weighing more than 20 pounds may face forward in the car seat." B) "A booster seat should be used when your child has outgrown a convertible car seat but is still too small to fit properly in a vehicle safety belt." C) "Keep using a rear-facing car seat until age 1 year." D) "It is safe to keep your child in a car seat in the front seat of the car as long as - CORRECT ANSWER c The mother of a 2-year-old child calls the office with concerns about temper tantrums. She asks if screaming, falling on the floor, and kicking are normal behaviors. The PNP explains A) "The child is expressing a need for independence." B) "The child is trying to get attention." C) "The child is probably spoiled and needs a spanking." D) "The child cannot yet express frustrations verbally." - CORRECT ANSWER d A 3-year-old is seen in the clinic for a well-child checkup. The mother states she is concerned because the child is a thumb sucker and worries this will cause dental problems. The PNP explains: A) There is no need for concern until the child reaches age 4 to 5 years B) An immediate dental referral is needed to prevent further damage C) Thumb sucking is not a problem until permanent dentition is completed D) A dental referral is needed within the next 6 months - CORRECT ANSWER a A 3-week-old infant is being breastfed by a new mother. The mother is unsure if the infant is getting enough breast milk. Besides documenting weight gain, what other signs can the mother observe as an indication of adequate breast milk intake? The infant: A) Sleeps 4 to 5 hours between feedings B) Cries vigorously before feedings C) Has 4 stools and 6 wet diapers per day D) Sucks eagerly on a pacifier and chews on hands after feeding - CORRECT ANSWER c The PNP is returning a phone call to a parent who has questions regarding the use of over-the-counter medications for a 2-year-old child. The mother reports that the child has been running a temperature of 101.2°F with no other symptoms of illness. The mother has been treating the fever by alternating doses of acetaminophen and ibuprofen every 3 to 4 hours. The child is playful and has a normal appetite. Appropriate counseling for this mother should include: A) Continue giving the medicines eve - CORRECT ANSWER c The PNP is performing a well-child examination on an 8-year-old child with a previous diagnosis of precocious puberty. The patient is Tanner stage III for breast development and pubic hair growth, with height and weight in the 95th percentiles. The patient has missed 6 months of her hormone therapy. The mother states that the child often complains of abdominal cramping, and the mother has noticed an increased vaginal discharge. The remainder of the examination is normal. Addressing the abdominal - CORRECT ANSWER c A 7-year-old child is brought to the clinic for a rash that has been present for 24 hours. The child was "a little droopy about 3 days ago" but had no fever or other symptoms. The immunizations are up-to-date. The child now has a "slapped cheek" appearance, with a lacy, erythematous rash on the torso and upper arms. The rash blanches. The PNP discusses with the mother the diagnosis of: A) A coxsackievirus and explains that the rash will disappear within a week and no treatment is necessary B) Me - CORRECT ANSWER c A 12-year-old girl has questions about the potential for her to get much taller. She aspires to be a professional basketball player. On examination she is in Tanner stage IV. Menarche was 1.5 years ago. In considering answers to her questions, it is important to realize which of the following? A) She will not reach her peak height velocity until she is in Tanner stage V B) Menarche occurs an average of 2 years after thelarche C) Girls reach peak height velocity immediately before menarche D) Boy - CORRECT ANSWER c A mother asks about the term Moro reflex. The PNP explains that the reflex is present in a neonate and disappears or phases out at about age: A) 3 to 5 months B) 6 to 8 months C) 7 to 9 months D) 10 to 12 months - CORRECT ANSWER a A 10-year-old child was seen in the clinic 5 days before for a routine sports physical examination. The child's blood cholesterol level was 186 mg/dL. The most appropriate intervention by the PNP would be to: A) Screen other members of the family for hyperlipidemia B) Implement a diet plan of less than 20% of fat in total daily calories C) Counsel the child and family regarding a healthy diet and exercise and perform another blood cholesterol test in 3 to 5 years D) Repeat the total cholesterol - CORRECT ANSWER d Which of the following do not meet the criteria for a selective screening for hyperlipidemia? A) A 16-year-old adolescent who is adopted, who smokes, and who has no known family history B) An 8-year-old child with a small restrictive ventricular septal defect C) A 10-year-old child whose father had a balloon angioplasty at age 49 years D) A 12-year-old child with BMI of 28 - CORRECT ANSWER b When assessing the heart of a healthy 4-year-old child, the PNP would expect to find: A) A visible apical pulse or point of maximal impulse B) Sinus tachycardia C) S2 heart sounds louder than S1 heart sounds at the apex D) Long, low-pitched heart sounds - CORRECT ANSWER a A 15-year-old girl is being evaluated for possible delayed puberty. There is a documented history of Tanner stage 2 breast development and Tanner stage 1 pubic development at age 9.5 years. What current findings would support the diagnosis of delayed puberty? A) Family history of abnormal puberty B) Palpable breast buds with areolar enlargement C) Dark, coarse, curly pubic hair spreading over the mons D) Weight at 5th percentile - CORRECT ANSWER b An 8.5-year-old child is brought to the clinic for the first time. On physical examination a penile length of 6.2 cm is noted. The differential diagnosis includes precocious puberty. What additional clinical finding is associated with precocious puberty? A) A testicular length of 2.1 cm B) Multiple small café au lait spots (neurofibromatosis) C) A growth spurt of 5 cm/y D) A blood glucose level of 97 mg/dL - CORRECT ANSWER b A 5-day-old neonate, diagnosed with trisomy 21, was noted to have an elevated thyroid-stimulating hormone (TSH) level. The neonatal screening was performed at age 3 days. What should the PNP do next? A) Reassure the mother that an elevated TSH at age 3 days is common and that no further treatment is necessary B) Repeat the test because it was not performed at the appropriate time C) Repeat the test when the child is age 3 weeks because this is the appropriate time D) Repeat the test and inform t - CORRECT ANSWER d The PNP is meeting with the parents of a 7-year-old child with type 1 diabetes to discuss blood glucose levels. A suggested blood glucose level for a 7-year-old child with diabetes is between: A) 80 and 120 mg/dL B) 100 and 200 mg/dL C) 80 and 180 mg/dL D) 70 and 150 mg/dL - CORRECT ANSWER c A 14-year-old Asian-American male is brought to the office for a well-child visit. The adolescent measures 145 cm (57 inches) in height and weighs 46 kg (102 lb). Physical examination reveals underdeveloped genitalia (i.e., Tanner stage 1, testes down, no hernia). Which diagnostic tests would the PNP order initially? A) Thyroid function tests, a bone age determination, and a complete blood cell count B) Tests to determine plasma luteinizing hormone (LH) and follicle-stimulating hormone (FSH) lev - CORRECT ANSWER b The PNP evaluates an infant born to a mother known to have autoimmune thyroid disease. Congenital hypothyroidism in infants is associated with which of the following? A) Excessive sleepiness B) Failure to thrive C) Tachycardia D) Diarrhea - CORRECT ANSWER a In caring for a noncompliant 16-year-old adolescent with diabetes, the PNP can best assess the diabetic control by: A) Measuring the insulin level B) Obtaining a glucose tolerance test C) Monitoring serum glucose levels for 3 consecutive days D) Measuring the hemoglobin A1c level - CORRECT ANSWER d The PNP is caring for a 10-day-old neonate who was born by vaginal delivery at a birthing center. The neonate was sent home at age 24 hours and received the initial thyroid function tests. The tests revealed a mildly low T4 and an elevated TSH level. The infant has been feeding without difficulty, has regained birth weight plus 3 oz, and appears alert during the examination. The PNP's management of the child should include: A) Beginning levothyroxine therapy and referring the family to an endocr - CORRECT ANSWER b A 13-year-old Tanner stage III/IV female adolescent who competitively runs cross-country track is scheduled for an annual sports physical examination. The adolescent complains of mild abdominal cramps and backache. When asked about voiding difficulties, the adolescent reports frequency symptoms. The dipstick screening reveals yellow urine, +1 blood, and no protein, leukocytes, and nitrites on the reagent strip. This morning the adolescent started having some dark vaginal discharge. The physical - CORRECT ANSWER d In the management of a child with a neurogenic bladder resulting from myelomeningocele, the most critical long-term goal is: A) Controlling incontinence B) Preventing kidney damage C) Preparing the child for bladder augmentation D) Preventing bladder spasms - CORRECT ANSWER b Separation anxiety is a common developmental stage in which the child exhibits fears and unusual behavior when a parent is absent. This behavior is most often expected at: A) Age 6 to 9 months B) Age 9 to 12 months C) Age 12 to 15 months D) Age 15 to 18 months - CORRECT ANSWER a A 5-year-old child is at the clinic for a routine well-child visit before beginning kindergarten. The mother reports the child's list of chores as follows: clean own room, make own bed, wash and dry own clothing, take out the trash, and help wash the dishes. If the chores are not completed, the child is grounded from other activities, such as playing with friends or watching television. The mother states that the child frequently must be grounded. How should the PNP respond? A) Tell the mother t - CORRECT ANSWER c One of a young child's greatest fears is: A) Separation from parents or caregivers B) Not getting enough food C) Not being accepted by peers D) Having limits set by caregivers - CORRECT ANSWER a A 4-year-old child is examined in the clinic. The parents report that the child has limited language skills and does not speak in three-word sentences. The PNP is interested in further assessing the child and chooses the following: A) Denver Articulating Screening Examination (DASE) B) Denver II C) Clinical Adaptive Test/Clinical Linguistics Auditory Milestone Scale (CAT/CLAMS) D) Vineland Social Maturity - CORRECT ANSWER a A 4-year-old child is brought to the office for a preschool physical examination. The PNP asks the child to sit on the floor in a cross-legged position and then get up without using the hands. The child is unable to do so and rolls onto all four extremities before standing. The PNP is concerned that the child may have a: A) Neuromuscular disorder B) Developmental problem C) Cognitive deficit D) Neurologic deficit - CORRECT ANSWER a A 6-month-old infant is brought to the clinic for a well-child visit. On physical examination, increased tone and lower leg scissoring are noted. The PNP recognizes this as a probable sign of: A) Muscular dystrophy B) Down's syndrome C) Cerebral palsy D) Fragile X syndrome - CORRECT ANSWER c A 16-year-old female adolescent with Down's syndrome, who is healthy and has had no major medical problems, comes to the clinic for a well-child visit. The adolescent has been "mainstreamed" into a regular classroom and is doing well. The physical examination reveals Tanner stage III breast development and pubic hair. Today the PNP should: A) Provide nutritional counseling, order cervical spine x-ray films, and refer the adolescent to a family support group B) Perform hearing and vision screens, - CORRECT ANSWER d The first step in the process of developing an IEP for a child with disabilities is to: A) Provide government funds to the child and family to supplement income B) Perform a comprehensive assessment of the child by a multidisciplinary team C) Perform a complete history and physical examination to identify organic causes D) Perform a battery of psychologic tests that are valid and reliable - CORRECT ANSWER b A father accompanies his 4-year-old child to the clinic. Significant past history reveals that the child had lipomyelomeningocele. The father states that there are no specific complaints, but during the history he reports that the child has complained of back pain over the last 3 months. Further evaluation reveals that the frequency of bowel movements has decreased from once daily to three times a week and that the child is "wetter" between catheterizations. The most likely cause is: A) Acute mu - CORRECT ANSWER b A 2-month-old infant with spina bifida and shunted hydrocephalus has a history of increased gagging with feedings, spitting up, intermittent stridor, and failure to thrive. The most likely explanation is: A) Tethered cord B) Shunt malfunction C) Arnold-Chiari deformity type II D) Esophageal structure - CORRECT ANSWER c A 2-week-old neonate is brought to the office for the first time. The mother states that the doctor in the hospital suspected Down's syndrome, and she begins to cry. On physical examination the infant has characteristics that may indicate Down's syndrome. The phenotypic features the PNP recognizes as commonly associated with Down's syndrome include: A) Large ears and small mouth, short fingers, and jaundice B) Small ears, short neck, and hypotonia C) Flat nasal bridge and normocephaly D) Protube - CORRECT ANSWER b The mother brings a 4-month-old infant to the office for a well-child visit. The infant received a hepatitis B (Hep B) immunization in the hospital at birth and another Hep B in the office at age 1 month but has received no other immunizations. The infant lives with the mother and grandfather. The grandfather is asthmatic and is taking steroids. What immunizations should the infant be given today? A) Hep B, DTaP, and Hib B) DTaP, Hib, and OPV C) DTaP, Hib, Prevnar, and IPV D) Hep B, DTaP, Hib, a - CORRECT ANSWER c The school counselor refers a 7-year-old child, who is not known to have received any immunizations, to the clinic. The child has no history of varicella. What immunizations are required at this visit? A) Hep B, MMR, Var, Td, and IPV B) Hep B, Var, Td, and IPV C) Hep B, Td, and IPV D) Hep B, MMR, Var, DTaP, and IPV - CORRECT ANSWER a The PNP is the primary care provider for a preterm infant (born at 31 weeks' gestation). The infant had no complications other than a minor ventricular hemorrhage at birth. The infant is 4 months' chronologic age and progressing well. What type of respiratory syncytial virus (RSV) prophylaxis should the PNP consider at today's visit? A) RSV prophylaxis in six monthly doses, if the local RSV season is approaching B) Nothing, because the infant did not have bronchopulmonary dysplasia at birth C) N - CORRECT ANSWER a A 4-year-old is brought to the office for a preschool physical examination. The child was hospitalized for Kawasaki's syndrome and treated with intravenous immune globulin (IVIG) 9 months ago. According to the immunization records, the child requires DTaP, polio, and MMR immunizations at this time. What immunizations should the PNP administer today? A) DTaP, IPV, and MMR B) Only MMR C) None (immunizations should be given at the school physical examination next year) D) DTaP and IPV (the parent s - CORRECT ANSWER a A 12-month-old child is brought to the clinic. On physical examination the PNP notes that the child's right eye drifts to the nasal side of the eye. The PNP explains to the mother that a complication of this condition may be: A) Myopia B) Astigmatism C) Amblyopia D) Strabismus - CORRECT ANSWER c What is the appropriate INITIAL intervention if the parent refuses immunizations? a. notification of child protective services of vaccine refusal b. discussion of the community benefit immunization c. validation of the parental concerns regarding individual immunizations d. provision of the "refusal to immunize" form for parental signature - CORRECT ANSWER c Which adolescent is considered the MOST likely to abuse illicit drugs? a. a 12yo caucasian male who admits to drinking beer once a month b. a 15yo african am male who admits to trying a variety of alcoholic beverages c. a 16yo caucasian male, who dropped out of school, was suspended twice for smoking and has smoked marijuana twice d. a 16 yo african am, male who is an honor student but admits to smoking marijuana occasionally - CORRECT ANSWER c The most appropriate analgesia for a 4yo with post operative tonsillectomy pain would include a. ibuprofen b. acetaminophen and oxycodone (Roxicet) c. Ketorolac d. aspirin and oxycodone (Percodan). - CORRECT ANSWER b A previously healthy 10mo has vomiting, intermittent periods of intense crying, and the passage or red stool. Which would be considered FIRST in child? a. refer to emergency dept b. obtain an abdominal x-ray c. obtain a complete blood count d. schedule surgical appointment for tomorrow. - CORRECT ANSWER a. Herpes Zoster is characterized by clustered vesicles and a. is transmitted via respiratory secretions b. is reactivated on exposure to a siblings chicken pox c. results frequently in post herpatic neuralgia d. often develops along the trigeminal nerver - CORRECT ANSWER d Which of the following should be included when educating a family regarding the care of children with atopic dermatitis? a. apply a barrier ointment to the child's face before feeding b. bathe children daily and encourage low humidity in the house c. expect symptom improvement within 1 week after starting probiotics d. investigate and eliminate possible food triggers - CORRECT ANSWER a A parent reports that her 18mo cries during the entire aerosol asthma treatment with albuterol and budesonide. What is an alternative managment? a. prescribing oral albuterol b. using MDI and Spacer c. using nebulizer when child is sleeping. d. adding montelukast - CORRECT ANSWER a In addition to hypoglycemia, the MOST common complications found in infants born to mothers with diabetes or gestational diabetes include a. LGA, neural tube defects, anemia, and renal vein thrombosis b. SGA, cardiomegaly, polycythemia and hyperthermia c. LGA, cardiomegaly, renal vein thrombosis and polycythemia d. SGA, hepatosplenomegaly, hypotonia, and anemia - CORRECT ANSWER c The parents of an adolescent with pectus excavatum and asthma are interested in surgery for their child, in the hopes of improving his asthma. Which would be MOST accurate response? a. the only purpose of surgery is cosmetic b. studies of post-operative cardiac and pulmonary functioning following pectus repair show inconsistent results. c. reduction of lung volume and degree of symptoms correlates well with extent of the deformity d. improvement of asthma after surgery is dependent on the family - CORRECT ANSWER b Compared to single component vaccines, combination vaccines a. are associated with significant incidence of adverse effects b. expose children to many more antigens than previous vaccines c. are more likely to overload an infant's immune system d. can be used if the child previously received component of the vaccine. - CORRECT ANSWER d One of the major psychosocial tasks of infancy is: a. development of secure attachment b. separation-individuation c. symbiosis d. regulation - CORRECT ANSWER a When examining 10 yo Bob's teeth, you note that the upper incisors slightly overlap the lower incisors. The second and lower first molars are absent. Your assessment is: a. malocclusion b. delayed mandibular dentition c. normal dentition d. hyperdontia - CORRECT ANSWER b Which one of the following best describes behavior associated with Piaget's concrete operations? a. Learning primarily by trial and error b. Interpreting events in relationship to self c. categorizing information d. drawing logical conclusions - CORRECT ANSWER c A 5.5yo asian child has fallen off of her growth curve. The best intervention would be to: a. Suggest infant breakfast drinks b. Incorporate traditional foods in diet c. educate family on need to increase calories & nutrients d. Refer to endocrine clinic for evaluation - CORRECT ANSWER b What is considered minimal weight gain in a normal newborn after discharge from the hospital? a. 10gms/day b. 20gms/day c. 30gms/day d. 40 gms.day - CORRECT ANSWER c The most likely weight of a 1yo child whose weight at birth was 6.5lbs would be what? a. 19-20lbs b. 13-14lbs c. 25-26lbs d. impossible to estimate - CORRECT ANSWER a You would expect a school age child to: a. grow 1.5in per year b. grow 0.5in per year c. Gain about 6lb per year d. Gain about 3lb per year - CORRECT ANSWER c Johnny, 5 yo, is in your clinic for his initial well check. You note he has not received any immunizations. Which of the following is not necessary at his age? a. MMR b. DTaP c. Hib d. Varicella - CORRECT ANSWER c As the NP, you are assessing an 8mo infant in an immunization clinic. You know that by 8mo the child should have had which of the following immunizations? a. Hep B (1st & 2nd), all primary DTaP series, two doses of polio b. All DTaP, polio series, MMR c. DTaP (1st & 2nd), MMR (1st), all Hep B series d. Varicella, DTaP (1st), Hep B (1st) - CORRECT ANSWER a Mom informs you that she and her 6mo infant are traveling to a place where measles is endemic. Your best response to her is: a. She should not take her baby with her b. Reassure her that her baby has passive immunity c. Give baby a MMR or monovalent measles injection d. Give baby gamma globulin prior to the trip - CORRECT ANSWER c Tommy is in for his 4mo well check. He was born preterm with a birth weight of 2.0 kg. The appropriate immunizations to give him today would be: a. DTaP, Hib, IPV, HepB-3 b. DTaP, Hib, IPV, PCV 7 c. DTaP, Hib, Hep B-3, PCV 7 d. Hold off on immunizations until he reaches his term due date - CORRECT ANSWER b In considering client situations, which one requires the use of an inactivated (not live) vaccine? a. history of nonspecific allergies b. immunocompromised client c. concurrent antimicrobial therapy d. mild acute illness - CORRECT ANSWER b Which of the following ongoing assessments have no specific indication for individuals which Turner's syndrome? a. Cardiac monitoring b. vision screening c. tanner staging d. thyroid screening - CORRECT ANSWER b While examining a 2.5 week old infant, you notice irritability when you lift the infant and an asymmetrical Moro reflex. A spasm of the sternocleidomastoid on the left side is also present. These findings suggest: a. Torticollis b. Genu varum c. Fracture clavicle d. Pierre-Robin syndrome - CORRECT ANSWER c Newborn screening for hypothyroidsm is done by measuring: a. thyroid stimulating hormone (TSH) b. Thyroxine (T4) and TSH c. Triiodothyronine (T3) d. T4 binding globulin (TBG) - CORRECT ANSWER b Infants who have been identified as IUGR are prone to developing hypoglycemia due to: a. decreased metabolic rate b. low levels of glycogen stored c. become acidotic d. a prone to develop sepsis - CORRECT ANSWER b You are performing a newborn physical exam and you notice white specks around the circumference of the iris. You will also have to assess for: a. hypertonicity b. cherry red spot on the macula c. heart murmur d. cleft palate - CORRECT ANSWER c What is the correct sequence of motor development in average infants? a. Reach & miss, sit alone, creep, climb stairs, stand alone b. creep, sit alone, climb stairs, stand alone, reach & miss c. creep, sit alone, climb stairs, reach and miss, stand alone d. sit alone, creep, climb stairs, reach and miss, stand alone - CORRECT ANSWER a 3 mo rachel cannot remain seated upright without support. At what age will she acquire the seated position with support? a. 7mo b. 9mo c. 12mo d. 14mo - CORRECT ANSWER a During a well-child visit you teach parents that an infant may first transfer an object from hand to hand at: a. 2mo b. 4mo c. 7mo d. 9mo - CORRECT ANSWER c A child just starting to reach for objects placed in front of him is probably around: a. 1mo b. 4mo c. 7mo d. 9mo - CORRECT ANSWER b Children do not generally exhibit a fear of strangers until after about? a. 2mo b. 4mo c. 6mo d. 2yo - CORRECT ANSWER c According to Erik Erickson, the "cornerstone of vital personality" is the development of: a. Symbiosis b. Autonomy c. Independence d. basic trust - CORRECT ANSWER d A mother of 10mo old infant asks about the progression of solid foods into her baby's diet. From the choices below, you advise her to introduce which food last to her baby's diet? a. egg yolk b. egg white c. fruits d. vegetables - CORRECT ANSWER b A 9mo old infant boy is seen with a 2 day hx of diarrhea. He had 3-4 wet diapers in the past 24hrs and his anterior fontanel is slightly depressed. Capillary refill is WNL. What % of dehydration does this infant have? a. 3% b. 5% c. 8% d. 10% - CORRECT ANSWER c Molly is a 12yo girl who has diffuse stomach pain and acute onset of diarrhea described as frequent urge to defecate. She is passing large amounts of flatus, small amounts of stool, and has rectal tenderness with digital exam. Which lab test will confirm your diagnosis? a. serum albumin & ESR b. abdominal ultrasound c. stool for ova & parasites d. Upper GI - CORRECT ANSWER b Vomitus that is bilious suggest: a. obstruction proximal to the pylorus b. obstruction below the ampulla of Vater c. Pyloric stenosis d. peptic ulcer disease - CORRECT ANSWER b An 18yo female college freshman, Bailey, presents to college health with an 8hr hx of abdominal pain that began in the periumbilical area and then localized to the RLQ. She has been on OCPs for teh past 6mo. Bailey denies a medical hx of UTIs, pregnancy, STDs, or chronic illness. Upon her physical exam, she is guarding on the RLQ with generalized tenderness and no masses. Her temp is 99.6 and her vitals are normal. Based on this H&P, which of the following is the most likely diagnosis for Baile - CORRECT ANSWER c An 18yo female college freshman, Bailey, presents to college health with an 8hr hx of abdominal pain that began in the periumbilical area and then localized to the RLQ. She has been on OCPs for teh past 6mo. Bailey denies a medical hx of UTIs, pregnancy, STDs, or chronic illness. Upon her physical exam, she is guarding on the RLQ with generalized tenderness and no masses. Her temp is 99.6 and her vitals are normal. Which of the following is the least helpful in the eval of abdominal pain in thi - CORRECT ANSWER c An 18yo female college freshman, Bailey, presents to college health with an 8hr hx of abdominal pain that began in the periumbilical area and then localized to the RLQ. She has been on OCPs for teh past 6mo. Bailey denies a medical hx of UTIs, pregnancy, STDs, or chronic illness. Upon her physical exam, she is guarding on the RLQ with generalized tenderness and no masses. Her temp is 99.6 and her vitals are normal. Which of the following is the most suggestive of Bailey's diagnosis? a. Leukocyt - CORRECT ANSWER c You receive a call from a mother of a 2yo girl with a temp of 101F and diarrhea for 5 days with 6-8 liquid stools/day. How would you advise this mother? a. Go to ER now b. Come to the office tomorrow morning c. NPO d. Loperamide 10ml after each loose stool - CORRECT ANSWER b An 18yo male comes to see you with a painless mass in the right side of his scrotum. The most likely diagnosis is: a. epididymitis b. testicular torsion c. incarcerated hernia d. hydrocele - CORRECT ANSWER d Kali is HIV + and has just delivered a 3.8kg male infant. Which of the following should be included in her postpartum counseling? a. suggest use of commercially prepared formula b. encourage HIV testing of the infant at birth & at 2mo c. avoid day care until infant has had 2 negative HIV tests d. instruct that oral polio vaccine is recommened - CORRECT ANSWER a Higher-functioning adolescents with autism are at increased risk for: a. significant phobias b. panic attacks c. substance abuse d. obsessive-compulsive disorder - CORRECT ANSWER d Which of the following is an example of a fine motor skill achieved by preschool aged children? a. being able to alternate feet b. walking on a balance beam c. holding a glass with one hand d. balancing on one foot - CORRECT ANSWER c according to Erikson, the major psychological task between 1-2yrs of age is the development of: a. trust b. identity c. industry d. autonomy - CORRECT ANSWER d A mother of a 16mo old toddler announces that she is giving her child skim milk. You tell her she should switch to whole milk because: a. skim milk is not easily digested b. Skim milk does not have enough calcium c. Skim milk has too little protein d. Skim milk does not contain enough essential fatty acids - CORRECT ANSWER d What is the most sophisticated and advanced type of play? a. associate b. parallel c. cooperative d. onlooker - CORRECT ANSWER c In boys, Tanner III is distinguished from Tanner stage II by: a. Penile growth in width b. Penile growth in length c. Fine, downy hair growth at the base of penis d. Adult-like pubic hair that does not extend to the thigh - CORRECT ANSWER b During an annual physical exam, you notice that Lisa has developed breast buds. You tell her she can expect which of the following in 2 years: a. Pubic hair growth b. Peak height velocity c. Onset of menses d. axillary hair - CORRECT ANSWER c The time of ovulation is usually: a. about 14 days before onset of the next menstrual period b. about 14 days after the onset of the previous menstrual period c. during the menstrual period d. about 7 days after the onset of the menstrual period - CORRECT ANSWER a Jess has recently noticed that a change in his voice, the shape of his body, and pubic and facial hair growth. What is he noticing is the development of: a. primary sexual characteristics b. secondary sexual characteristics c. deciduous sexual characteristics d. laten sexual characteristics - CORRECT ANSWER b According to Erikson, the major psychosocial task of adolescence is the achievement of: a. autonomy b. industry c. identity d. generativity - CORRECT ANSWER c Which component of identity is likely to develop first? a. physical b. vocational c. moral d. ideological - CORRECT ANSWER a Which of the following is responsible for sexual maturation of males? a. estrogen b. testosterone c. progesterone d. adrogen - CORRECT ANSWER b Which of the following is the correct sequence for adolescent female development? a. growth acceleration, breast development, pubic hair, menarche, axillary hair b. growth acceleration, pubic hair, breast development, axillary hair, menarche c. breast development, growth acceleration, axillary hair, pubic hair, menarche d. axillary hair, breast development, pubic hair development, menarche, growth acceleration - CORRECT ANSWER a Adolescent girls are especially prone to developing which of these disorders: a. hirsutism b. hypothyroidism c. folate deficiency anemia d. iron deficiency anema - CORRECT ANSWER d When discussing issues concerning confidentiality, the NP should tell the adolescent (>18yo) or when under the state age of majority that: a. the parents must have access to all information in the health hx b. parents cannot access information without adolescent's permission c. parents must be present during all encounters d. it is at the discretion of the provider to decide whether to release information - CORRECT ANSWER b During auscultation of a 3yo's chest, you note an irregular heartbeat of 90bpm that slows when resp rate slows & accelerates when the child breathes faster. The rest of the exam is normal. What is appropriate response to this finding from the choices below? a. record the finding in the child's chart b. order a chest x-ray c. order an echocardiogram d. refer the child to a cardiologist - CORRECT ANSWER a Which of the following drugs is important in symptom management and the prevention of complications in Kawasaki's disease? a. aspirin b. corticosteroids c. acetaminophen d. penicillin - CORRECT ANSWER a While performing a well child check exam on a 3yo you discover a heart murmur. In your eval, you remember the following about innocent murmurs: a. they are best heard in diastole b. you must radiate to the axilla c. the intensity is no greater than I or II/VI d. There is no variation with changes in position - CORRECT ANSWER c Following tympanostomy tube insertion, it is important that the tubes remain patent. Which of the following methods may be used to determine patency? a. visual inspection b. impedance tympanometry c. valsalva maneuver d. instillation of an ototopical suspension - CORRECT ANSWER a 9yo Tom is in the clinic for a "cold" that lasted for 10 days. He has no complaints of breathing problems. A physical exam reveals temp of 100F, edematous cervical lymph nodes, mucopurulent drainage from the middle meatus, erythematous pharynx & malodorous breath. Tom's management should include: a. culture of nasal drainage b. radiograph of the sinuses c. measurement of erythrocyte sedimentation rate d. use of an antibiotic - CORRECT ANSWER d After sinusitis dx, two days later, Tom is back in the clinic because now he has a "swollen eye". Redness, inflammation, and impaired extraocular movement of his right eye are observed. Which action is most appropriate form the choices below: a. reassure mom that this is common & benign b. treat for bacterial conjunctivitis c. order anti-inflammatory eye drops d. refer tom immediately - CORRECT ANSWER d 7.5yo Jimmy presents to your clinic with inspiratory stridor, drooling, and a temp of 105F. He insists on sitting up during the exam. What is your diagnosis? a. foreign body aspiration b. reactive airway disease c. croup d. epiglottitis - CORRECT ANSWER d In a child with chronic sinusitis, the most accurate method of identifying abnormalities is: a. dark room transillumination of the sinuses b. percussion of the paranasal sinuses c. AP, lateral, and occipitomental sinus radiographs d. CT scan of the sinuses - CORRECT ANSWER d An 8yo afebrile child presents with a ST. Upon examination, you note that his tonsils are 4+ without exudate. Differential diagnosis includes: a. normal b. strep throat c. hodgkin disease d. phayngitis - CORRECT ANSWER c A 2.5yo child presents with a dried pea stuck in his nose passed the first turbinate. An appropriate intervention would be: a. Refer to ENT b. NaCl flush c. Attempt removal with forceps d. Close ipsilateral nares and encourage sneezing - CORRECT ANSWER a Which of the following is NOT a goal for appropriate asthma management? a. limited activity & exercise b. prevent recurrent exacerbations c. prevent chronic troublesome symptoms d. Maintain near normal pulmonary functions - CORRECT ANSWER a 9yo Jon has recently been diagnosed with mild intermittent asthma. Which of the following is not a routine part of his management? a. spirometry eval b. metered dose inhaler technique demonstration c. environmental triggers & control methods review d. allergy skin testing - CORRECT ANSWER d Tammy has mild persistent asthma. Appropriate daily medications for this 4yo preschooler should include: a. an inhaled low dose corticosteroid b. short acting beta2 agonists c. an oral systemic corticosteroid d. a cough suppressant - CORRECT ANSWER a Sally, a 9yo, with moderate persistent asthma, is still having mild symptoms. She uses a short acting inhaled beta 2 agonist 3x/day and cromolyn sodium. You alter this treatment to include which of the following? a. broad spectrum antibiotic b. addition of systemic corticosteroid x5 days c. replace cromolyn with inhaled corticosteroid, and decrease the beta agonist to prn when symptoms subside d. addition of an inhaled anticholinergic - CORRECT ANSWER c During a visit to your clinic, Mitch, 2yo, presents large muscular calves & demonstrates difficulty rising from sitting position. Which lab test should you order below? a. serum calcium b. serum magnesium c. serum phosphorus d. serum creatine kinase - CORRECT ANSWER d Which of the following tests is not an appropriate test for development dysplasia of the hip in an 18mo infant? a. allis sign b. skinfold symmetry c. galeazzi sign d. ortolani maneuver - CORRECT ANSWER d Tina, 12yo, presents with a lateral curvature of her spine. Which of the following tests would diagnose scoliosis? a. positive gower sign b. negative gower sign c. positive adam's sign d. negative adam's sign - CORRECT ANSWER c Johnny, a 16yo high school student is in for a sports physical. He is in good health, but is concerned about a "bad" headache he had several weeks ago. He is afraid he might have a brain tumor. You tell him the most common type of headache with onset in adolescents is: a. sinus headache b. vascular headache c. tension headache d. migraine headache - CORRECT ANSWER c A CBC is ordered for a 10yo female. Results indicate decr MCHC & decr MCV. Differential diagnosis should include: a. sickle cell anemia b. vitamin B12 deficiency anemia c. Pernicious anemia d. iron deficiency anemia - CORRECT ANSWER d lab results reveal a hypochromic, microcytic anemia in a 3yo child. Your differential diagnosis must include: a. lead poisoning b. pernicious anemia c. hemophilia d. folic acid deficiency - CORRECT ANSWER a 11yo Mark is diagnosed with constitutional growth delay. Appropriate management includes: a. starting him on low dose testosterone therapy now b. counseling regarding delayed onset of puberty c. thyroxine replacement d. nutritional counseling - CORRECT ANSWER b Which of the following observations demonstrates the function of the oculomotor nerve? a. the child frowns during the examination b. the child offers appropriate verbal responses to the nurse practitioner during the examination c. the child visually tracks the parent's movements around the room during the examination d. the child sticks out the tongue during the examination - CORRECT ANSWER c A 6yo child is being evaluated after sustaining seizures. Which of these tests would be most relevant in this screening? a. determination of the levels of anticonvulsant drugs b. electroencephalography c. Denver II screening test d. Electromyography - CORRECT ANSWER b A 13yo adolescent comes to the school based clinic with a headache that began this morning before school. The headache is described as unilateral and increasing in intensity. What type of HA is it? a. cluster b. tension c. migraine d. sinus - CORRECT ANSWER c Which of the following types of headaches requires immediate referral to a physician? a. acute recurrent b. chronic nonprogressive c. acute d. chronic progressive - CORRECT ANSWER d A newborn is brought to the office for a 2-week well-baby exam. The PNP notes that the infant's head circumference is 2 SDs below the norm. A neurologist evaluated the child in the newborn nursery but detected the abnormal circumference without significant findings. The baby is otherwise growing well. What is the most appropriate recommendation? a. F/U in 2 weeks, including measurement of the head circumference b. refer the patient to a neurologist. c. order skull x-ray films & MRI d. do nothing - CORRECT ANSWER a The PNP is examining a 6mo old infant with what is believed to be craniosynostosis. The suture lines reveal a prominent bony ridge over the occipital and coronal sutures. The PNP should: a. refer the infant to a neurologist b. remeasure the head circumference in 2mo c. discuss with the parents the need for PT d. order chromosomal studies - CORRECT ANSWER a Which of the following is the most appropriate first choice management strategy for a 30mo old child with learning difficulties and ADD/ADHD? a. referral for appropriate education b. referral to an ophthalmologist c. behavior modification d. prescription for Ritalin - CORRECT ANSWER c A 20mo old child is reported to have been "shaking" for a short period. The parent reports that the child had a fever of 102F before the episode but seems to have returned to normal. Which intervention should be undertaken first? a. refer child to the ED b. refer child to neuro c. conduct an exam of child d. have a parent bring the child back for F/U in 1 mo - CORRECT ANSWER c A 6-year-old boy is brought to your office for a routine well-child examination. Significant findings in the child's history, reported by his mother, include unspecified "birth trauma" and a 9-year-old brother who has "difficulty in school." The mother also reports that her son cannot "sit still" or play with toys for long periods and notes that he recently had to change schools when the family moved. Physical examination reveals a well-nourished, appropriately dressed 6-year-old boy and no r - CORRECT ANSWER a A 6-year-old boy is brought to your office for a routine well-child examination. Significant findings in the child's history, reported by his mother, include unspecified "birth trauma" and a 9-year-old brother who has "difficulty in school." The mother also reports that her son cannot "sit still" or play with toys for long periods and notes that he recently had to change schools when the family moved. Physical examination reveals a well-nourished, appropriately dressed 6-year-old boy and no r - CORRECT ANSWER c On the basis of your exam you determine a child has ADHD: distractibility, impulsivity, lack of attention, accompanied by hyperactivity. Which of the following conditions is a differential diagnosis? a. seizures b. social disturbance c. dyslexia d. cerebral palsy - CORRECT ANSWER b Education for parents whose child has sickle cell disease should include that the majority of pain crises are triggered by which of the following? a. no identifying cause b. temperature changes c. cigarette smoke d. stressful situations - CORRECT ANSWER a A 3 year old, who attends daycare, has diarrhea that began with three days of low grade fever and foul smelling watery stools. One month later, he continues to have loose stools and now has abdominal distention. The most probable causative organism is a. rotavirus b. Shigella toxicum c. Giardia lamblia d. Staphylococcal enterococcus - CORRECT ANSWER c A 5 year old complains of a painful left eye after being accidentally scratched by a sibling two hours ago. Fluorescein exam shows a small central corneal abrasion. The most appropriate management during the first 24 hours is a. frequent application of topical antibiotic b. observation of the injured eye c. frequent application of topical non-steroidal anti-inflammatory drops d.occlusive patching of the injured eye - CORRECT ANSWER a Which POST competition guidance is most helpful for best nutrition practices for the athlete? a. additional protein consumption is needed for tissue repair b. protein supplements will increase muscle size if taken after competition c. fat loading is most important within the first 1-2 hours d. replacement of carbohydrates should optimally occur within 2 hours - CORRECT ANSWER d Which of the following best demonstrates the oral behavior of a 12 month old in relation to readiness for feeding? a. begins chewing meat with rotary mouth movements b. controls bite of soft cookie c. controls liquids taken from a cup d. sucks in anticipation of the spoon - CORRECT ANSWER b Which of the following is associated with alcohol use in adolescence? a. family history of alcoholism b. authoritarian parenting style c. passive temperament d. high socioeconomic status - CORRECT ANSWER a The intervention currently advised for a child with 4 episodes of acute otitis media in the past 5 months is a. prophylactic antibiotics b. myringotomy and tympanostomy c. adenoidectomy and tonsillectomy d. observation - CORRECT ANSWER b Significant progression of a 20 degree scoliotic curve is most likely to occur in a female who is a. 10 years, Tanner 3, Risser sign of 3 b. 11 years, Tanner 2, Risser sign of 2 c. 13 years, Tanner 4, Risser sign of 4 d. 15 years, Tanner 4, Risser sign of 5 - CORRECT ANSWER b Nutrition assessment of the child on a strict vegan diet should include regular growth monitoring, diet analysis, and laboratory assessment of a. vitamin b12, zinc, and iron status b. vitamin b12, calcium, and electrolytes c. electrolytes, iron, and vitamin b6 d. vitamin b6, zinc, and calcium - CORRECT ANSWER a A 13 year old has repeatedly exhibited behaviors of purposeful cruelty to younger children, property destruction, and school truancy. These behaviors fit BEST the diagnostic critera for a. major depressive disorder b. oppositional defiant disorder c. conduct disorder d. dysthymic disorder - CORRECT ANSWER c Credentialing of the pediatric nurse practitioner requires which of the following? a. recognition by third party payers b. verification of license c. authorization to perform specific services d. certification of specialty - CORRECT ANSWER b A 4 year old with close contact 10 days ago to a school classmate, now hospitalized with meningococcemia, currently has no symptoms and a normal physical exam. No other cases have been reported in the community. Appropriate management would include a. oral rifampin prophylaxis b. nasopharyngeal culture c. meningococcal vaccine d. reassurance - CORRECT ANSWER d Which immunization statement applies to a 3 year old who completed chemotherapy 2 months ago for acute lymphocytic leukemia (ALL)? a. the child should not receive immunizations containing live viruses b. serum immunity should be evaluated prior to administration of vaccine c. a repeat of all recommended immunizations is necessary d. a repeat of live virus vaccines is warranted - CORRECT ANSWER a Which is the MOST accurate to convey when counseling a family about the effects of congenital cytomegalovirus (CMV) infection on hearing? a. half of symptomatic infants will have conductive hearing loss b. hearing loss may be progressive after the newborn period c. asymptomatic infants are free of hearing loss d. infection acquired from breast milk is associated with hearing loss - CORRECT ANSWER b Which of the following is the best INITIAL counseling for a family of a 5 year old with nocturnal enuresis? a. wake the child frequently during the night so urination is effective b. symptoms are often a developmental lag and will be outgrown c. medication therapy has the best long term response rate d. an alarm system is the best choice as it is usually covered by health insurance - CORRECT ANSWER b In addition to a developmental evaluation, which baseline studies should be included for an infant diagnosed with 22q11 deletion syndrome? a. echocardiography, thyroid function and renal ultrasound b. calcium levels, abdominal ultrasound and immunologic screening c. echocardiography, renal ultrasound, and immunologic screening d. thyroid function, calcium levels, and abdominal ultrasound - CORRECT ANSWER c The parent of a 7 year old is concerned about the child's dislike of school. It is late fall, and the child has now been absent for 20 days. The FIRST challenge for the parent is to a. obtain psychological counseling b. send the child to school everyday c. determine what the physical illness may be d. have the child catch up on missed school work - CORRECT ANSWER b A teen whose diet consists of frequently eating at fast food restaurants is most likely to have deficits of which nutrients? a. iron and calcium b. phosphorous and iron c. calcium and niacin d. phosphorous and niacin - CORRECT ANSWER a A previously healthy 4 year old has had the following immunizations: 6 MONTHS: DTaP/HIB, IPV, Hep B 15 MONTHS: DTap/HIB, IPV, Hep B, MMR ,Varicella At this visit the most appropriate recommendation is a. DTAP/HIB, IPV, Hep B, PCV 7 b. DTaP, HIB, Hep B, MMR c. DTaP, IPV, Hep B, MMR d. DTaP, IPV, PCV 7 - CORRECT ANSWER c Which diagnostic study warrants the MOST attention when evaluating a child with autism? a. audiologic evaluation b. EEG to rule out seizure disorder c. state-mandated metabolic screening d. brain imaging - CORRECT ANSWER a Which of the follwoing statements regarding alternating doses of ibuprofen and acetaminophen for fever is most accurate? a. alternating drugs increases risk of dosage errors and toxicity b. alternating allows for lower total dosage of both drugs c. alternating drugs results in more rapid resolution of the fever than using either drug alone d. is appropriate only for children over the age of six months - CORRECT ANSWER a The parents of a 6 year old reports that the child has been allergic to most fruits since early life. Which history question should be asked FIRST? a. what were the child's symptoms? b. how many different fruits is the child allergic to? c. has the child had any previous testing? d. how old was the child with the first reaction? - CORRECT ANSWER a The parent a of 2 week old states that even though the infant was fed Enfamil LIPIL with iron in the hospital, he is now receiving ProSobee LIPIL formula due to concern about allergies in an older child. What is the BEST response? a. the infant should be switched back to the Enfamil because the ProSobee does not contain an adequate amount of iron b. the infant should be maintained on Enfamil until there are obvious signs of allergy such as vomiting and bloody diarrhea c. both formulas contain eq - CORRECT ANSWER c Which alternative therapy is most likely to be explored by parents of a child with attention deficit hyperactivity disorder (ADHD)? a. eliminations of food additives b. use of multi-vitamin therapy c. elimination of dietary sugar d. visual training - CORRECT ANSWER a Which criteria is essential for the diagnosis of obsessive-compulsive disorder (OCD) for children? a. behaviors occur up to an hour a day b. behaviors are purposeless c. presence of repetitive thoughts, images, or processes d. child recognizes the actions are excessive - CORRECT ANSWER c Of the following, the MOST effective method to prevent childhood gun injuries and death is a. parents preventing children's access to guns b. educating children with the Eddie the Eagle program c. educating children with the STAR Program d. children's awareness of gun presence in the home. - CORRECT ANSWER a An example of a pleiotropic genetic disorder is: A) Marfan's syndrome B) Cri du chat syndrome C) Vitamin D-resistant rickets D) Tay-Sachs disease - CORRECT ANSWER a A deficiency of which of the following vitamins may cause night blindness, xerophthalmia, or keratomalacia? A) Vitamin A B) Vitamin K C) Vitamin D D) Vitamin E - CORRECT ANSWER a A 6-month-old infant is brought to the office for a well-child visit. The history obtained from the mother reveals that the infant has had "cold symptoms" for 1 week. The mother reports that the infant's eating and sleeping habits have not changed and that the infant has not experienced any other symptoms. Physical examination reveals a temperature of 100.9°F (38.3°C) and right otitis media. The immunization record reads as follows: Birth-Hep B Age 1 month-Hep B Age 2 months-DTaP, Hib, Prevnar - CORRECT ANSWER c A 6-year-old child in foster care is brought to the clinic by the foster parents. They are concerned that the child's immunizations may not be current. The child's history indicates no contraindications to any immunizations. The family has an official immunization record for the child that lists the following: Age 2 months DTP, OPV, Hib, and Hep B Age 4 months DTP, OPV, Hib, and Hep B Age 6 months DTP, OPV, and Hib Age 4 years DTP, OPV, Hib, and Hep B The immunizations recommended for today are - CORRECT ANSWER a The PNP sees a 6-week-old, HIV-exposed neonate in the clinic. The mother received zidovudine (AZT) starting in the second trimester of pregnancy and intravenous AZT during labor. The neonate was started on oral AZT after birth. As a prophylaxis for Pneumocystis carinii pneumonia (PCP) at this 6-week visit, it would be appropriate in conjunction with the HIV specialist managing the infant to: A) Continue ZDV, and start PCP prophylaxis if the neonate is HIV positive B) Continue ZDV, and start TM - CORRECT ANSWER d Individuals living with an infant who has been diagnosed with pertussis should receive: A) A cephalosporin B) Pertussis immune globulin C) Erythromycin prophylactic D) The pertussis vaccine - CORRECT ANSWER c The PNP understands that many factors complicate the inheritance patterns known as the Mendelian inheritance patterns. Neurofibromatosis can be used as an example of all of the following influencing factors except: A) Reduced penetrance B) Genomic imprinting C) New mutation D) Variable expression - CORRECT ANSWER b A 3-year-old child is brought to the office for the first time. The child is adopted, and little is known about the prenatal and birth history. A diagnosis of fetal alcohol effects (FAE) is suspected. The physical findings that lead the PNP to suspect this diagnosis are: A) Carious teeth, hyperactivity, toe walking, and macrocephaly B) Balance problems, irritability, multiple scars on the arms and legs, and microcephaly C) Growth retardation, thin or wide lips, flat midface, and finger anomal - CORRECT ANSWER c Which of the following children does not need PCP prophylaxis? A) An HIV-exposed newborn B) A 9-month-old, HIV-positive infant with a CD4 lymphocyte count of 1600 (no immune suppression) C) A 3-year-old, HIV-positive child with a CD4 lymphocyte count of 1000 (no immune suppression) and a history of PCP D) A 3-year-old, HIV-positive child with a CD4 lymphocyte count of 600 (moderate immune suppression) - CORRECT ANSWER d A 12-year-old child reveals to the PNP during a visit for acne that she may be pregnant. In this community the pregnancy rate among females aged 15 to 19 years is 3 in 100. What is the most important area to explore? A) The possibility of sexual abuse B) The expected date of confinement C) Plans for telling her family D) The source of prenatal care - CORRECT ANSWER a Most states require that a battery of screening tests be performed on the blood of neonates, usually after the neonate has taken formula or breast milk. Many of the diseases being screened for are rare. The most common disease screened for is: A) Phenylketonuria B) Galactosemia C) Maple syrup urine disease D) Hypothyroidism - CORRECT ANSWER d A 5-year-old child is brought to a community health clinic for a physical examination and immunizations before starting kindergarten. The father says he is unsure whether the child had chickenpox. The immunization record reads as follows: Age 1 month Hep B Age 2 months DTaP, Hib, Prevnar, and IPV Age 4 months DTaP, Hep B, Hib, Prevnar, and IPV Age 6 months DTaP, Hep B, Prevnar, and Hib Age 15 months Hib, Prevnar, and MMR Age 18 months DTaP and IPV What would be the most appropriate action? A) O - CORRECT ANSWER b The PNP examines a 9-year-old child with a history of reactive airway disease since birth for the first time. In the past 2 weeks the child has had three exacerbations. After evaluating the child, the PNP determines that the mostappropriate medication is: A) Oral steroids and albuterol B) Pirbuterol acetate (Maxair) or Salmeterol C) Albuterol (Ventolin, Proventil) D) Beclomethasone (Beclovent) or Fluticason (Flovent) - CORRECT ANSWER a The foster mother of a 16-year-old adolescent calls the office to make an appointment. She suspects that the adolescent is "doing drugs" and wants to secretly test for drugs. Which one of the following responses is most appropriate? A) Explain that such drug testing is unethical B) Tell the foster mother that drug use is rare in 16-year-old adolescents C) Schedule an appointment for the adolescent to be tested D) Schedule separate appointments for the foster mother and the adolescent - CORRECT ANSWER d A 2-year-old child is currently receiving chemotherapy for neuroblastoma. An older sibling is due to receive a poliovirus vaccine before starting kindergarten. The appropriate action at this time is to: A) Give IPV B) Withhold IPV, and give it 1 year after the sibling completes chemotherapy C) Withhold IPV, and give it as soon as the sibling completes chemotherapy D) Give oral poliovirus (OPV) - CORRECT ANSWER a A 7-year-old child has been seen in the clinic and emergency room several times in the past month for epistaxis. The nosebleeds last approximately 20 minutes and eventually stop with the application of pressure, but the child tends to begin bleeding again within a couple of days. While the PNP is obtaining a family history, the mother states that she experienced frequent nosebleeds as a child. When questioned, the mother states that she now has heavy menses. What is the mostlikely diagnosis for - CORRECT ANSWER d A 13-month-old child is brought to the office for a well-child visit. The child is healthy today and has no history of allergies or reactions to previous immunizations. The PNP reviews the immunization record and finds the following: Age 5 months DTaP, IPV, Hib, Prevnar, and Hep B The child should receive the following immunizations today: A) DTaP, OPV, Hib, Hep B, MMR, and Var (the child should return in 2 months for DTaP, IPV, and Hib) B) DTaP, IPV, Hib, and Hep B (the child should return in - CORRECT ANSWER c When performing a preemployment physical on a 16-year-old adolescent in an urban health care center the PNP learns that the patient's girlfriend was recently treated for a vaginal infection. Based on this information, the PNP should: A) Contact the primary care provider of the adolescent's sexual partner for further information B) Obtain a urine specimen to rule out chlamydial infection C) Treat immediately with doxycycline D) Do no further workup because the patient is asymptomatic - CORRECT ANSWER b The test most commonly used to elicit strabismus in infants and children older than 6 months is the: A) Snellen Eye Chart B) Tumbling E Game C) Cover-uncover test D) Allen Picture Cards - CORRECT ANSWER c The PNP is managing care for a 5-year-old child with growth failure. After a complete history is taken and a physical examination is performed, the mostrelevant and cost-effective screening tests to obtain are: A) Tests to determine total protein level, bone x-ray films to determine bone age, urinalysis, and CBC B) Bone x-ray films to determine bone age, CBC, urinalysis, and tests to determine potassium and chloride levels C) Bone x-ray films to determine bone age; CBC; T4, TSH, and liver pro - CORRECT ANSWER c The PNP is examining an infant recently adopted from Western Europe. When should a foreign-born child undergo PPD testing? A) At the first health examination after entering the United States B) After the child develops signs and symptoms suggesting TB exposure C) Never (if the child has had BCG, a chest x-ray examination should be ordered) D) At the same time as infants born in the United States - CORRECT ANSWER a The PNP is concerned that a child may have A. lumbricoides (roundworm). Which of the following symptoms would lead the PNP to this diagnosis? A) Fever and runny nose B) Nausea and vomiting C) No symptoms D) Vomiting and rash - CORRECT ANSWER c A 17-year-old adolescent is seen in the "fast track" of the emergency room with symptoms of respiratory distress, headache, and euphoria. The PNP suspects: A) Glue sniffing B) Overdose of antihistamines C) Alcohol ingestion D) Acetaminophen overdose - CORRECT ANSWER a A husband and wife, both aged 42 years, have adopted a 10-month-old infant from Eastern Europe. The infant has been in an orphanage since birth. The infant's family history is unavailable. A physical examination of the infant was required before the infant entered the United States, and the results were documented as "normal.'' The adoptive parents are bringing the child in for the first well-child visit with the PNP today. What initial screening tests should be performed? A) CBC, urinalysis, h - CORRECT ANSWER d A 4-month-old infant is brought to the office for a well-child visit. While giving the infant's history, the grandmother states that she dreaded coming to the office today because the infant "cried for hours" after the first set of immunizations. The immunization record reads as follows: Birth Hep B Age 2 months DTP/Hib, Hep B, Prevnar, and IPV The following immunizations should be given today: A) DTaP, Hib, and MMR B) DT, Hib, Hep B, and IPV C) DTaP, Hib, Prevnar, Hep B, and IPV D) DTP/Hib - CORRECT ANSWER b A 6-month-old infant is brought to the office for a well-child visit. The infant has had several upper respiratory tract infections but is healthy today. The mother states that the infant attends day care 3 days a week. The immunization record reads as follows: Birth Hep B 4 months DTaP, IPV, Hib, and Hep B What immunizations should the infant be given today? A) DTaP, IPV, Hib, Prevnar, and Hep B B) DTaP, IPV, Prevnar, and Hib C) DTaP, OPV, Hib, and Rv D) DTaP, IPV, Hib, and Rv - CORRECT ANSWER b The PNP is examining a 15-year-old Vietnamese adolescent during a health maintenance visit. The adolescent immigrated to the United States 3 weeks ago. Which of the following screening tests is most important at this visit? A) Mantoux skin test B) Malaria smear C) Microfilaria smear D) Papanicolaou test - CORRECT ANSWER a Once a child is started on oral iron therapy for the treatment of iron-deficiency anemia, what changes are evident in the laboratory studies? A) There is an immediate rise in the reticulocyte count, and the hemoglobin level returns to normal in 2 weeks B) There is a rise in the reticulocyte count beginning on the third day, and the hemoglobin level returns to normal in 6 to 10 weeks C) There is a concurrent rise in the hemoglobin level and the reticulocyte count, with both returning to normal - CORRECT ANSWER b What should the PNP consider when immunizing a child for measles, mumps, and rubella (MMR) in relationship to a TB skin test? A) Give MMR first, and give the PPD in 1 to 2 weeks B) Give PPD first, and 6 months later give MMR C) PPD and MMR should never be given at the same time D) Give both at the same time, or give MMR and wait 4 to 6 weeks to apply PPD - CORRECT ANSWER d A child with hemophilia falls off a bike, hurting the knee. The child is brought to the emergency room because the mother notices swelling of the affected joint and the child is complaining of pain. What is the highest priority in this child's care? A) Applying ice to the affected knee B) Obtaining a radiograph of the affected knee C) Administering factor VIII concentrate D) Splinting the affecting knee - CORRECT ANSWER c A pregnant woman who has had no prenatal care comes to the hospital in active labor. After delivery, the enzyme-linked immunosorbent assay (ELISA) and Western blot laboratory studies performed upon admission reveal that the mother is HIV positive. The priority for the PNP caring for the neonate in the nursery is to start: A) PCP prophylaxis with TMP/SMX B) AZT the day of discharge C) AZT within 24 hours of birth D) Dapsone orally within 48 hours of birth - CORRECT ANSWER c An 8-year-old child brought to the office by the parent has a cough, nasal congestion lasting 3 days, and an oral temperature of 99°F. After the PNP has reviewed the history and completed a physical assessment, a viral upper respiratory tract infection is diagnosed. The child has a history of varicella. A review of the immunization record reveals the following: Age 2 months DTP, OPV, and Hib Age 5 months DTP, OPV, and Hib Age 7 months DTP, OPV, and Hib Age 18 months DTP, Hib, and MMR Age 5 year - CORRECT ANSWER b A 4-year-old child has had a productive cough with yellow-green sputum but no fever for 7 days. In developing a plan of treatment, the PNP understands the following: A) The production and color of the sputum may indicate a bacterial infection B) Prolonged cough illnesses are often allergic or viral in nature C) A cough that continues up to 14 days after onset is probably a bacterial infection D) Infection and reactive airway disease are common causes of productive cough in children - CORRECT ANSWER d For which of the following individuals is there no indication for obtaining a chromosomal analysis? A) A child with phenylketonuria (PKU) B) A child with multiple congenital anomalies and mental retardation C) A couple with a history of multiple miscarriages D) A man with large testes and mental retardation - CORRECT ANSWER a In what Tanner stage for girls would the PNP expect to see enlargement of the breasts and areolae, as well as curly, coarse pubic hair? A) Tanner 4 B) Tanner 2 C) Tanner 3 D) Tanner 5 - CORRECT ANSWER c It is early fall, and an 18-month-old child with a history of asthma is brought to the office because of an acute exacerbation. The PNP reviews the child's immunization record with the mother. The child's breathing improves after a nebulizer treatment. While giving the history, the mother states that the child gets a rash after eating eggs. The immunization record reads as follows: Birth Hep B Age 2 months DTaP, IPV, Hib, Prevnar, and Hep B Age 6 months DTaP, IPV, Hib, and Prevnar Age 10 months - CORRECT ANSWER b A 15-year-old adolescent is brought to the office for a routine examination. The adolescent is healthy and has no history of allergies or reaction to previous immunizations. There is no reliable history of varicella. The PNP reviews the adolescent's immunization record and finds the following: Age 3 months DTP and OPV Age 5 months DTP and OPV Age 9 months DTP and OPV Age 18 months DTP, Hib, and MMR Age 5 years DTP, OPV, and MMR The PNP should administer the following immunizations today: A) Td, - CORRECT ANSWER a An example of a genetic disorder occurring because of genomic imprinting is: A) Marfan's syndrome B) Prader-Willi syndrome C) Trisomy 18 D) Down's syndrome - CORRECT ANSWER b A 13-year-old adolescent is brought to the office because of a partial-thickness burn on the leg caused by a campfire. The immunization record reads as follows: Age 3 mo-DTP, OPV, & Hib Age 7 mo-DTP, OPV, & Hib Age 10 mo-DTP, OPV, & Hib Age 20 mo- DTP, Hib, & MMR Age 5 years-DTP and OPV The following immunizations should be administered today: A) Td, IPV, and Var (if there is no history of disease) B) None (because a tetanus-containing vaccine was administered within the last 10 years) C) Td, - CORRECT ANSWER d A single, 15-year-old mother who attends high school brings a 3-month-old infant to the clinic. The infant seems to be healthy and thriving, but the mother is vague when reporting the infant's feeding and sleeping habits. To get more accurate information about the specifics of the infant's routine and care, the PNP should: A) Make a referral to a home nursing agency B) Ask about the day care arrangements C) Ask the mother if she has a learning disability D) Make a referral to a social servic - CORRECT ANSWER b A 14-month-old child is brought to the office for a well-child visit. The child is healthy today and has no history of allergies or reactions to previous immunizations. The family lives with a grandmother who is currently undergoing chemotherapy for cancer. The PNP reviews the child's immunization record and finds the following: Birth-Hep B Age 2mo-DTaP, IPV, Hib, & Hep B Age 4mo-DTaP, IPV, and Hib Age 6mo-DTaP and Hib Age 9mo-Hep B The PNP should give the child the following immunizations today - CORRECT ANSWER b Diagnostic studies considered part of normal screening for the child with developmental delays of unknown etiology include chromosomal studies or karyotyping, deoxyribonucleic acid (DNA) studies to detect fragile X syndrome, and: A) Measurement of urine and plasma amino acid levels B) Microscopic urinalysis C) Cystometrography D) Measurement of serum antinuclear antibody levels - CORRECT ANSWER a A 2-year-old child is brought to the clinic for a well-child visit. The mother expresses concern about the child's language development. Which of the following limitations would be an indication that this child has a language delay? A) The child can name two pictures B) The child has a vocabulary of 20 words C) The child's speech is halfway understandable D) The child can put two words together - CORRECT ANSWER b What is the probability of a couple having a child with cystic fibrosis (CF) if the mother carries the △F508 gene and the father does not? A) 0% B) 100% C) 25% D) 50% - CORRECT ANSWER a A 15-month-old child who began treatment for leukemia 3 weeks ago is brought to the clinic by the parents for a previously scheduled well-child visit. The immunizations that are due today include an inactivated poliovirus (IPV) vaccine and the measles, mumps, rubella (MMR) vaccine. What immunizations should be administered? A) Administer IPV today and delay MMR until 1 year after treatment has been completed B) Delay both vaccines until treatment for leukemia is completed C) Administer vaccin - CORRECT ANSWER a Which hepatitis virus vaccines are available and approved for use in children? A) Hepatitis A and C B) Hepatitis B and C C) Hepatitis A, B, and C D) Hepatitis A and B - CORRECT ANSWER d A 10-month-old infant is assessed for ocular alignment with the corneal light reflex test. The PNP is testing for: A) Anisometropia B) Strabismus C) Functional loss of vision D) Hyperopia - CORRECT ANSWER b The PNP is seeing a 1-month-old infant for routine follow-up. The infant received both the Hep B and the hepatitis B immune globulin (HBIG) vaccines because the mother is hepatitis B surface antigen (HBsAg) positive. At this visit the PNP should: A) Obtain a Hep B core antigen B) Obtain an anti-HBe C) Readminister only the HBIG D) Give the second dose of Hep B - CORRECT ANSWER d A low-birth-weight neonate is examined in the clinic. The mother is Native American and smoked heavily during the pregnancy. The PNP recognizes that the Native American population has an increased incidence of: A) Sudden infant death syndrome (SIDS) B) Reactive airway disease C) Developmental disability D) Neonatal sepsis - CORRECT ANSWER a A 5-year-old child was born in Mexico and received bacille Calmette-Guérin (BCG) vaccine as an infant. A PPD is administered as part of a routine office visit. After 48 hours, there is a 10-mm reaction. How should the PPD be interpreted? A) Interpret the PPD as if the child had not received BCG B) This child should not have undergone PPD testing because the result will always be positive C) Interpret the PPD as a normal reaction D) This child should not have undergone PPD testing because th - CORRECT ANSWER a What is the American Academy of Pediatrics' recommendation for routine skin testing for TB in a child with no risk factors who lives in a low prevalence area of the country? A) Test at age 3 months, 1 year, and yearly thereafter B) PPD should be performed yearly C) Routine skin testing is not indicated in this group D) Test at age 1 year, entrance to kindergarten, and high school - CORRECT ANSWER c A 12-month-old infant is brought to the office for a well-child visit. The infant is healthy today, has tested positive for HIV, and has no history of allergies or reactions to previous immunizations. The mother is HIV positive. The child's immunization history is as follows: Birth Hep B Age 3 months DTaP, Hib, Prevnar, Hep B, and IPV Age 6 months DTaP, Hib, Prevnar, and IPV Age 9 months DTaP, Hib, Prevnar, and Hep B The PNP should administer the following immunizations today: A) IPV and Var B - CORRECT ANSWER c A 3-year-old child with sickle cell disease is brought to the clinic for a well-child visit. What is an appropriate intervention to decrease the child's risk of infection? A) Obtain a CBC with differential and platelet count every 6 months B) Obtain a yearly Hgb electrophoresis and administer the pneumococcal vaccine annually C) Prescribe penicillin 20 mg/kg and administer Hib at this visit D) Continue penicillin prophylaxis 20 mg/kg and administer the pneumococcal vaccine at this visit - CORRECT ANSWER d A 14-year-old adolescent comes to the school-based clinic for a physical examination. During the interview the adolescent claims to be a member of a gang. What response would be appropriate in the initial discussion with this adolescent? A) "Can you tell me if you are currently using drugs?" B) "Can you tell me who else belongs to your gang?" C) "Tell me more about your relationship with your family and close friends." D) "Have you ever been arrested?" - CORRECT ANSWER c A 7-year-old child with no record of prior immunizations received initial immunizations including Td, IPV, MMR, Var, and Hep B today. When should the child return for the second set of immunizations? A) In 6 months B) In 2 weeks C) In 1 year D) In 2 months - CORRECT ANSWER d An 8-year-old child who was born and raised in Vermont is healthy and has no known risk factors for tuberculosis (TB). A PPD skin test results in 7-mm induration and redness. How should this be interpreted? A) As a negative skin test result B) As a positive skin test result C) As a negative skin test result (but testing should be repeated in 3 months) D) As an unequivocal skin test result (and testing should be repeated now) - CORRECT ANSWER a Diagnostic studies included in the routine screening of the child with developmental delays of unknown etiology include chromosomal karyotyping, deoxyribonucleic studies (for fragile X syndrome), and: A) Cystometrography B) Tests to detect urine and plasma amino acids C) Microscopic urinalysis D) Tests to detect serum antinuclear antibodies (ANAs) - CORRECT ANSWER b A 7-year-old child with factor IX-deficient hemophilia falls off the slide at recess and hits his head on the ground. Since returning home from school, the child has complained of headache and dizziness. What actions should the PNP take first? A) Order magnetic resonance imaging, and then perform a complete neurologic examination B) Administer factor IX concentrate C) Perform a complete neurological evaluation to rule out CNS injury D) Order a computed tomography (CT) scan of the head - CORRECT ANSWER b A 13-month-old child is brought to the clinic for a routine visit. The child is healthy today and has no history of allergies or previous reactions to immunizations. The PNP checks the immunization record and finds the following: Age 2 months DTaP, IPV, Hib, Prevnar, and Hep B Age 5 months DTaP, IPV, Hib, Prevnar, and Hep B Age 8 months DTaP and Hib The PNP should give the child the following immunizations today: A) DTaP and IPV B) MMR and Var (a return visit should be scheduled for 1 month la - CORRECT ANSWER c A 15-year-old adolescent comes to the school-based clinic with the complaint of nausea in the morning and breast tenderness. Menses are 5 days late. It is not abnormal for her to be late. A pregnancy test is performed and is positive. The PNP advises the adolescent to first: A) Discuss the situation with her parents B) Discuss the results with the adolescent to determine what this means to her C) Consult with an obstetric physician or adolescent clinic D) Repeat the pregnancy test in a week - CORRECT ANSWER b A 6-month-old infant who is new to the practice is brought to the office for a well-child visit. The infant is healthy today and has no history of allergies or reactions to previous immunizations. The PNP reviews the immunization record and finds the following: Birth Hep B Age 2 months DTaP, IPV, Hib, Prevnar, and Hep B Age 4 months DTaP, IPV, Prevnar, and Hib Today the infant should receive: A) DTaP, Hib, Prevnar, and Rv B) Hib, Hep B, and Rv C) DTaP, Hib, Prevnar, and Hep B D) DTaP, Hib, a - CORRECT ANSWER c A 5-month-old infant is brought to the office for a routine well-child visit. The infant is healthy today and has no history of allergies or reactions to previous immunizations. The PNP checks the immunization record and finds the following: Birth Hep B Age 1 month Hep B Age 2 months DTaP, IPV, and Hib Today the infant should receive the following immunizations: A) DTaP, IPV, Hib, and Var B) DTaP, IPV, Hib, Prevnar, and Hep B C) DTaP, IPV, Hib, Prevnar D) DTaP, OPV, and Hib - CORRECT ANSWER c The PNP sees a 6-week-old, HIV-exposed neonate in the clinic. The mother received ZDV starting in the second trimester of pregnancy and intravenous ZDV during labor. The infant was started on oral ZDV after birth. The most accurate test to determine whether the infant is HIV positive is: A) Ribonucleic acid (RNA) PCR B) ELISA C) Western blot D) Deoxyribonucleic acid (DNA) polymerase chain reaction (PCR) - CORRECT ANSWER d Which of the following genetic disorders can be detected by cytogenic testing? A) Huntington's chorea B) Sickle cell anemia C) Tay-Sachs disease D) Down's syndrome - CORRECT ANSWER d The mother of a 10-year-old girl is concerned that the child has been sexually molested. Upon genital examination of a prepubescent girl after penile penetration, the PNP would expect to find: A) Permanent tears of the vaginal wall B) An intact hymen C) Normal-appearing genitalia D) A wide-open vaginal orifice - CORRECT ANSWER c An 18-month-old is seen in the emergency room with a second-degree "glove distribution" burn. A more detailed history is needed because of the concern for: A) Staphylococcal scalded skin syndrome B) Immersion injury caused by child abuse C) Possible contact with a caustic material D) Accidental burn from an iron - CORRECT ANSWER b A 2-week-old infant comes to clinic accompanied by his 15-year-old mother. The primary goal for this visit is to discuss: A) Available birth control methods B) Establishing fatherhood through the legal system C) Returning to school immediately D) Specific parenting skills by using concrete examples - CORRECT ANSWER d A 4-year-old is seen in the office for a preschool physical. The past history is significant for hospitalization for Kawasaki's disease and treatment with intravenous immune globulin (IG) 2 months ago. The immunization record indicates that the child needs a DTaP, polio, and MMR at this time. What immunizations should the PNP administer today? A) DTaP, OPV, and MMR at this visit, and instruct the parent to schedule a return visit in 8 weeks B) MMR only at this visit C) None, give immunization - CORRECT ANSWER d The PNP is evaluating a postterm (42 weeks' gestation) neonate in the nursery. On the physical examination the PNP would expect to find: A) Excessive vernix caseosa B) Smooth soles of the feet C) Cracked, peeling skin D) An elevated weight/length ratio - CORRECT ANSWER c At an 18-month-old child's well-child visit, the father states that the child is a picky eater and has 4 oz of milk by bottle at nap time and 8 oz of milk by bottle at bedtime. The rest of the child's fluids are taken by cup. Of the following, what is the most pertinent information for the PNP to obtain? A) The total intake of milk for 24 hours B) A 24-hour recall diet C) A list of foods the child refuses D) The family's financial status - CORRECT ANSWER b Routine pediatric office developmental testing is conducted to: A) Diagnose children with mental retardation B) Diagnose children with learning disabilities C) Determine the child's school readiness D) Identify children who may require formal evaluation - CORRECT ANSWER d A macrobiotic diet can best be described as a diet containing: A) No animal foods, including milk and eggs B) Whole grains, vegetables, fruits, and white meat of fish C) Grains, fruits, vegetables, and milk but no eggs, meat, poultry, or seafood D) Grains, fruits, vegetables, milk, eggs, fish, and fowl but no red meat - CORRECT ANSWER b Precocious puberty is defined as sexual development beginning before the youngest acceptable age. Which of the following statements is true regarding precocious puberty? A) The intellectual, emotional, psychosocial, and psychosexual development of these children far exceeds their sexual maturity rating B) Short stature in adulthood may result from the rapid skeletal maturation attributable to the early secretion of sex hormones C) The sexually precocious child rarely encounters psychosocial d - CORRECT ANSWER b Which of the following methods is best for diagnosing obesity? A) Skinfold measurements B) BMI C) Weight for height measurements D) Mother is obese - CORRECT ANSWER b Which of the following statements is true regarding childhood obesity? A) The most common problem associated with childhood obesity is diabetes B) If one parent is obese and the other is of normal size, 9% of their children will become obese C) The older the obese child is, the more likely it is that he or she will become an obese adult D) Endocrine causes of obesity are common - CORRECT ANSWER c The PNP sees a 4-week-old neonate for a well-child visit. The parents report that the infant seems to eat well but does not gain weight and has frequent, greasy-looking stools. The PNP suspects that the neonate might have: A) Cystic fibrosis B) Failure to thrive C) Gastroenteritis D) Intussusception - CORRECT ANSWER a The first sign of puberty in girls is usually: A) Pubic hair growth B) Thelarche C) Menarche D) Acne - CORRECT ANSWER b The PNP is examining a 12-year-old female child with enlarged breasts and areolas and curly, dark, coarse pubic hair. The PNP uses Tanner staging and classifies this child as: A) Tanner stage II B) Tanner stage III C) Tanner stage IV D) Tanner stage V - CORRECT ANSWER b A 5-year-old child is brought to the clinic for immunizations. The child has never had immunizations. What immunizations should the child receive today? A) Tetramune, IPV, MMR, and Varivax (and should return in 6 weeks) B) DPT, IPV, and Varivax (and should return in 8 weeks) C) DTaP, IPV, MMR, Varivax, and Hep B (and should return in 12 weeks) D) DTaP, IPV, MMR, Varivax, and Hep B (and should return in 6 weeks) - CORRECT ANSWER d A 2-week-old neonate is brought to the clinic for a well-child visit. The infant weighed 3.18 kg at birth; today the weight is 2.95 kg. The mother is breastfeeding about every 2 hours; the infant is latching on well and nursing 15 minutes per side. How many calories a day does this infant need for growth? A) 500 Kcal B) 375 Kcal C) 345 Kcal D) 300 Kcal - CORRECT ANSWER c The first nurse practitioner program was designed to: A) Provide emergency care to children in Colorado B) Provide limited access to health care for low-income children in Colorado C) Provide increased access to health care for children in Colorado D) Manage severely ill children at Colorado General Hospital - CORRECT ANSWER c National certification of PNPs has been offered since the 1970s. The following credentials may identify nurses who have national specialty certification: A) Master of science in nursing (MSN) B) Advance practice nurse (APN) C) Certified (C) D) Nurse practitioner (NP) - CORRECT ANSWER c The most common form of reimbursement for services provided by PNPs is: A) Fee charged at 85% of the physician's payment B) Fee charged at 90% of the cost under the physician's billing C) Fee for service D) Same fee as physician, "equal pay for equal service" - CORRECT ANSWER c In most cases the PNP is covered by employee professional liability for: A) Off-duty situations, such as athletic physicals not sponsored by the agency B) Off-duty personal liability C) Off-duty coverage for personal injury, such as discussing a patient in public D) Those acts covered as normal work-related services and duties - CORRECT ANSWER d The PNP is employed by a physician and performs services under direct supervision of the physician; these services provided by the PNP are billed under the physician's name and are called: A) Incidence to service B) Direct billing for service C) Capitation reimbursement D) Salary - CORRECT ANSWER a In the health care system the major perceived barrier to use of an PNP's services in a primary health care setting is: A) Lack of consumer confidence B) Lack of quality of service provided C) Organized medicine D) Organized nursing - CORRECT ANSWER c Title V of the Social Security Act of 1935 was amended in 1981 and 1986 to expand coverage for children with special health care needs under: A) Title XIX of the Social Security Act B) The Supplemental Security Income (SSI) Program for the Aged, Blind, and Disabled C) The U.S. Public Health Service D) The Maternal and Child Health Services Block Grant - CORRECT ANSWER d Legal authority for prescriptive privileges for PNPs is granted by the: A) Food and Drug Administration B) State Nurse Practice Act C) Nurse practitioner certification boards D) State pharmacy licensing boards - CORRECT ANSWER b The School Lunch Program; Women, Infants, and Children (WIC) Program; and Food Stamp Program are administered by: A) The Food and Drug Administration B) The U.S. Department of Agriculture C) Social Security Block Grants D) The U.S. Department of Public Health - CORRECT ANSWER b Standards for well child care/health promotion are set forth by the: A) Task Force, Preventive Services, Department of Health and Human Services B) American Academy of Family Physicians C) American Academy of Pediatrics (AAP) D) Healthy People 2000/2010 documents - CORRECT ANSWER c Medical professionals are legally required to report abuse when: A) Appropriate resources are available to investigate B) Abuse is suspected C) Forensic evidence is collected D) Protocols are used to evaluate the report - CORRECT ANSWER b When approaching adolescents about sensitive issues, such as drug use and sex, an effective interviewing approach is to: A) Introduce the topics early in the history B) Let the adolescent know the parents are concerned C) Begin by asking about their friends' activities D) Avoid promising confidentiality - CORRECT ANSWER c A well-child examination has been completed on a 1-year-old child who appeared slightly pale. The results of the routine CBC are as follows: hemoglobin, 7.8 mcg/dL; mean corpuscular volume, decreased; and red cell distribution width, elevated. The remainder of the CBC is normal. What would be the most likely cause of this child's anemia? A) Thalassemia B) Megaloblastic anemia C) Iron deficiency D) Lead poisoning - CORRECT ANSWER c An 8-year-old is brought to the clinic with "hard bumps" in the neck area and axillae for the past week. The child has been living with grandparents for the past 6 months and visits a nursing home frequently. The history is negative for other symptoms such as weight loss. Upon exam the nodes are enlarged but painless. The PNP is suspicious of: A) Epstein-Barr virus (EBV) B) Cytomegaly virus (CMV) infection C) Histoplasmosis D) Tuberculosis (TB) - CORRECT ANSWER d While examining a 2-month-old, the PNP notices leukocoria. This is the most common physical finding associated with: A) Neuroblastoma B) Retinoblastoma C) Retrolental fibroplasia D) Congenital cataracts - CORRECT ANSWER b Which of the following patients should be referred for further developmental evaluation? a. Term 12-month-old child not walking independently whose older sibling walked at 10 months b. Premature 7-month-old child born at 28 weeks gestation not sitting independently c. Term 12-month-old infant not using single words d. Term 6-month-old infant with poor head control - CORRECT ANSWER d Which of the following is accurate about the emancipated minor? a. The adolescent patient who disagrees with a parent/guardian about medical treatment can be treated as an emancipated minor b. All 50 states have identical legal statutes with regard to the emancipated minor. c. The emancipated minor may not legally consent to medical care involving reproductive health issues d. The emancipated minor may legally consent to all types of medical care - CORRECT ANSWER d An important assumption underlying the "family systems theory" is: a. Problems in the family can be traced back to individual family members b. Family dysfunction is best addressed by emphasizing past (rather than current) family dynamics. c. The family is a closed system which does not interact with its environment d. Changes in one part of the family affect all other parts of the family system - CORRECT ANSWER d The leading cause of death for children 1-18 years of age in the United States is: a. Accidents b Homicide c. Cancer d. Congenital anomalies - CORRECT ANSWER a Which of the following statements is most accurate concerning the pediatric nurse practitioner's scope of practice? a. Prescribing medications is not within the pediatric nurse practitioner's scope of practice b. Pediatric nurse practitioners diagnose and treat most common childhood illnesses c. Pediatric nurse practitioners may practice independently without physician oversight in all 50 states d. Pediatric nurse practitioners may only provide routine health maintenance and immunizations - CORRECT ANSWER b Which of the follow parental characteristics is associated with an increased risk of child maltreatment (abuse)? a. Older parents b. Immigrant parents c. History of childhood abuse/neglect for the parent d. Wealthy parents - CORRECT ANSWER c The pediatric nurse practitioner performs the "cover test" on a 3-year-old patient to screen for strabismus. When the left eye is covered, the right eye moves outward to fix on a near object. The cover test has revealed which of the following conditions? a. Pseudostrabismus b. Left eye strabismus c. Right eye strabismus d. Normal ocular alignment - CORRECT ANSWER c Which of the following is accurate about transmission of an autosomal recessive trait, such as the defective gene of cystic fibrosis? a. 25% of children born to two carriers of the gene mutation will be affected by the disease b. All male offspring born to two carriers of the gene mutation will be affected by the disease, while none of the female offspring will be affected c. Half of the female offspring born to two carriers of the gene mutation will be carriers of the gene mutation and half of - CORRECT ANSWER a Family-centered care is characterized by which of the following? a. Care team acts as the experts and instructs the patient and family in how to properly carry out the care plan b. Collaboration between the care team and patient/family regarding decision-making and clinical care c. Patient/family direct the clinical care team's actions d. All members of the patient's family must agree on a specific care plan before it is implemented - CORRECT ANSWER b The pediatric nurse practitioner is evaluating a well-appearing 18-month-old patient with diarrhea for 6 weeks. The family reports that the patient develops progressively looser stools throughout the day. Dietary history reveals that the patient drinks 12 ounces cow's milk and 24 ounces apple juice daily. Growth and development are within normal limits for age. Stool studies are negative for enteric pathogens and blood. The most likely diagnosis is: a. Malabsorption secondary to cystic fibrosis - CORRECT ANSWER c A 13-year-old female patient is being evaluated by the pediatric nurse practitioner for allergic rhinitis. During the visit, the patient discloses that she is considering have sexual intercourse with her boyfriend. Later in the visit, she mentions that she has been feeling "depressed" recently. Which of the following should be the pediatric nurse practitioner's first priority in caring for this patient? a. Evaluating the patient further to determine the best contraceptive method for this patient - CORRECT ANSWER b Informed consent is characterized by: a. The duty to provide all relevant information to a patient about their condition and alternative treatments b. The duty to respect a patient's individual values and choices c. The duty to benefit patients and protect their interests d. The duty not to inflict harm on patients - CORRECT ANSWER a Which of the following is the best approach for conducting a physical examination of the 2-3 year-old child? a. Proceed systematically from head-to-toe b. Perform the most uncomfortable or traumatic parts of the examination first c. Have the parent leave the room before conducting the examination d. Allow the patient to remain with the parent and perform the uncomfortable/traumatic parts of the examination last - CORRECT ANSWER d The pediatric nurse practitioner reads a healthy 10-year-old patient's screening PPD (tuberculin skin test) as positive (12 mm in duration) 48 hours after the PPD was placed. The pediatric nurse practitioner's next action should be to: a. Begin multi-drug therapy for tuberculosis b. Repeat PPD testing in 6 months c. Begin monotherapy with isoniazid (INH) d. Obtain a chest x-ray - CORRECT ANSWER d Which of the following approaches to cultural differences between healthcare providers and patients/families is most effective? a. Encourage patients and families to conform to the "majority culture" when interacting with the health care system b. Urge parents to follow the advice of healthcare providers over that of extended family members c. Contact the appropriate child protective agency if a family is non-compliant with medical care due to cultural differences d. Utilize knowledge of the fam - CORRECT ANSWER d A pediatric nurse practitioner evaluates an established patient for fever. The nurse practitioner documents the chief complaint (fever), history of present illness (duration, height of fever, associated symptoms, and timing), and a review of systems involving 3 organ systems. Physical examination is documented and includes full vital signs, general appearance, and head/neck/lung/cardiac/abdominal/skin examinations. Final diagnosis is fever with lower respiratory tract infection. The pediatric nu - CORRECT ANSWER b Which of the following vaccines is contraindicated in the immunocompromised pediatric patient? a. Diphtheria, Tetanus, and acellular Pertussis (DTaP) b. Inactivated Influenza c. Measles, Mumps, Rubella (MMR) d. Hepatitis B - CORRECT ANSWER c The pediatric nurse practitioner notes two irregular areas of blue-gray discoloration over the sacral area of a 4-week-old Hispanic infant. The most likely etiology of this finding is: a. Inflicted injury (abuse) b. Acrocyanosis c. Mongolian spots (congenital dermal melanocytosis) d. Cutis marmorata - CORRECT ANSWER c The pediatric nurse practitioner is seeing a 9-year-old patient for routine health maintenance whose 40-year-old father recently suffered an acute myocardial infarction. Which of the following screening tests would be most appropriate for the pediatric nurse practitioner to order next? a. Hypercoagulability screening panel b. Fasting lipoprotein levels c. Serum troponin levels d. Brain natriuretic peptide (BNP) levels - CORRECT ANSWER b Which of the following best defines the term health literacy? a. An individual's formal health education level b. An individual's ability to read written information about health education c. An individual's capacity to obtain and understand basic health information needed to make appropriate health decisions d. An individual's motivation to make good personal health decisions - CORRECT ANSWER c The pediatric nurse practitioner is performing a well-child examination on a developmentally normal infant who recognizes his parents, can sit independently for 1-2 minutes, rolls over from supine or prone starting position, drops one object when another is offered, and babbles one-syllable sounds. This patient is approximately: a. 3 months old b. 9 months old c. 6 months old d. 12 months old - CORRECT ANSWER c The pediatric nurse practitioner suspects autism spectrum disorder in a 2-year-old patient. The nurse practitioner should: a. Refer the patient for further neurodevelopmental evaluation if symptoms persist at the 3-year-old health maintenance visit b. Promptly refer the patient for further detailed neurodevelopmental evaluation c. Recommend that the patient receive no further immunizations d. Assist the family in locating a long-term placement facility - CORRECT ANSWER b Which of the following statements is most accurate regarding nutrition in the school-age child? a. Most school-age children eat more than the recommended amount of whole grains b. Most school-age children need to increase their average sodium intake for improved nutritional status c. Meals consumed at fast food restaurants are usually more healthful than meals prepared at home d. Consumption of excess sugars, starches and fats in the average school-age child contributes to childhood obesity - CORRECT ANSWER d The siblings of a child who develops a serious illness, injury, or disability: a. Should be disciplined for verbalizing feelings of resentment and anger toward the affected sibling or parents b. Often develop behavior and adjustment problems due to disruptions in the family routine when a sibling is seriously ill or injured c. Should not be involved in the care and treatment of the affected sibling d. Should be removed from their usual activities in order to spend more time with the rest of the - CORRECT ANSWER b Which of the following represents the best organizational approach when changing one's practice based on clinical evidence? a. Establish clinical question, develop and implement care plan, evaluate effectiveness of care plan, collect and evaluate relevant research b. Collect and evaluate relevant research, establish clinical question, develop and implement care plan, evaluate effectiveness of care plan c. Establish clinical question, collect and evaluate relevant research, develop and implement - CORRECT ANSWER c The pediatric nurse practitioner is seeing a 13-year-old female patient with Tanner stage 4 breast and pubic hair development who has not had menarche. The pediatric nurse practitioner suspects: a. Precocious puberty b. Normal puberty c. Androgen insensitivity d. Ovarian tumor - CORRECT ANSWER b Which of the following is true about the children of parents who smoke cigarettes? a. Elevated risk for Sudden Infant Death Syndrome (SIDS) in infants of mothers who smoke b. Decreased likelihood that the child will smoke cigarettes as an adult c. Smoking cigarettes outside eliminates the risk that second-hand smoke poses to children d. No significant difference in the risk of asthma and lower respiratory tract infections when compared to children of non-smoking parents - CORRECT ANSWER a At a routine health maintenance visit, an 11-year-old patient with Tourette Syndrome informs the nurse practitioner that he is struggling both socially and academically at school. His parents have spoken with his teacher at school, but the problems have continued. He confides that he often goes to the school nurse's office with vague complaints to avoid some of his regular school day. The pediatric nurse practitioner's next action should be to: a. Refer the patient to a neurologist so that medic - CORRECT ANSWER c The mother of a 4-year-old female patient reports that the child frequently awakens screaming and agitated about 1 hour after falling asleep and does not appear to recognize her mother. The child falls asleep again within 10 minutes and does not recall these events in the morning. The pediatric nurse practitioner should: a. Tell the mother to speak loudly to the child during these episodes to fully awaken her b. Refer the family for counseling c. Refer the patient to a pediatric neurologist d. R - CORRECT ANSWER d The pediatric nurse practitioner is evaluating a one-month-old infant male with a unilateral undescended testicle (cryptorchidism). The patient was born at term after an uncomplicated pregnancy and physical examination reveals no other abnormalities. The pediatric nurse practitioner should: a. Refer the patient to pediatric surgery urgently for orchiopexy b. Refer the patient to pediatric surgery if testis remains undescended at 6 months of age c. Inform the parents that orchiopexy is indicated - CORRECT ANSWER b Which of the following is not a typical clinical feature of Kawasaki disease? a. Hand and foot edema b. Bilateral non-exudative conjunctivitis c. Risk of coronary artery aneurysms d. Bacterial growth in peripheral blood cultures - CORRECT ANSWER d The pediatric nurse practitioner notices that the mother of her 6-year-old patient looks down at the floor when denying concern for domestic violence. During the rest of the conversation, the mother maintains good eye contact with the nurse practitioner. The pediatric nurse practitioner should: a. Ask an open-ended question about domestic violence with the mother in private b. Ask the 6 year-old patient about violence at home c. Tell the mother she appears to be lying about domestic violence d. - CORRECT ANSWER a Which of the following techniques is most effective in communicating with the shy, 4-5-year-old patient? a. Stand up when speaking with them so that they are looking up at you b. Communicate through the child's transition object (e.g., doll) before addressing them directly c. Establish physical contact early d. Speak only with the patient's adult family members, rather than addressing the child directly - CORRECT ANSWER b In conducting educational health programs for patients with chronic illness, the most effective model involves: a. Individualized education aimed at encouraging patients to take charge of their own health b. Disseminating accurate information about self-care and treatment options without considering each patient's individual strengths or challenges c. Eliminating patients from the program who don't change their behavior based on the health information provided d. Single-session programs to provi - CORRECT ANSWER a The pediatric nurse practitioner evaluates an injured pediatric patient and suspects that the patient's injuries were caused by abuse. The pediatric nurse practitioner should: a. Report the suspected abuse only if the patient's family is acting suspiciously b. Report the suspected abuse to the appropriate state agency c. Report the suspected abuse to the appropriate state agency only if the physician agrees d. Document his/her concerns in the patient chart without reporting the suspected abuse - CORRECT ANSWER d Which of the following is most accurate about childhood lead exposure? a. Houses built after 1978 are most likely to have lead-based paint b. Children are less likely than adults to be affected by lead poisoning because their bodies absorb a lower percentage of lead when exposed c. Most pediatric lead poisoning occurs via normal hand-to-mouth behavior in the presence of environmental lead dust d. Children in minority ethnic groups have a lower incidence of lead poisoning than Caucasian children - CORRECT ANSWER c Which of the following factors associated with poor health outcomes would be least attributable to lack of health insurance? a. Difficulty accessing health care services due to geographic distance b. Non-compliance with prescribed medications due to cost c. Delay in seeking care for an acute condition due to financial hardship concerns d. Less utilization of preventive health care services - CORRECT ANSWER a Most stage-based theories of development focus primarily: a. the continuity of development b. the discontinuity of development c. persistance of inherent personality characteristics d. the influence of context on development - CORRECT ANSWER b The common practice using "time outs" with young children is direct application of: a. operant conditioning b. classical conditioning c. separation-individuation d. maturational reinforcement - CORRECT ANSWER a good communication between families, schools, and primary care providers is an example of which ecological concept? a. microsystem b. mesosystem c. exosystem d. macrosystem - CORRECT ANSWER b Which of the following findings would be most likely associated with asymmetric intrauterine growth retardation? a. weight, length, & head circumference ranging from 3rd to 5th percentile b. heavy maternal smoking throughout pregnancy c. Weight at 3rd percentile and length at 25th d. Gestational diabetes - CORRECT ANSWER c Early reflexive responses that are not r/t survival include all but: a. babinski b. moro c. swimming d. rooting - CORRECT ANSWER d The most likely weight of 1yo child whose weight at birth was 6.5lb would be: a. 19-20lbs b. 13-14lbs c. 25-26lbs d. Impossible to estimate - CORRECT ANSWER a One of the major psychosocial tasks of infancy is: a. development of secure attachment b. separation-individuation c. symbiosis d. regulation - CORRECT ANSWER a Which developmental theory best explains the multifactorial etiology of failure to thrive? a. organismic-maturational theory b. social learning theory c. transactional theory d. psychoanalytic theory - CORRECT ANSWER c Most healthy infants are able to reach, grasp, & hold onto a rattle or other small toy by about: a. 2mo b. 6mo c. 8mo d. 10mo - CORRECT ANSWER b The pincer grasp is a fine motor skill that involves the ability to pick up a small object such as a raisin or piece of cereal with the thumb and forefinger that usually is mastered around: a. 4mo b. 6mo c. 9mo d. 16mo - CORRECT ANSWER c You would be concerned about the language development of a child who: a. repeats simple phrases at 32mo b. stutters when excited or tired at 42mo c. has a vocabulary of 10 words at 12mo d. pronounces words that are understandable at 36mo - CORRECT ANSWER b The most common temperamental profile is: a. easy b. difficult c. slow-to-warm-up d. intermediate - CORRECT ANSWER a The underlying emotion of an insecurely attached (avoidant) relationship is: a. ambivalence b. deprivation c. anger d. conditional love - CORRECT ANSWER c The stage of cognitive development that Piaget described as characteristic of the way preschoolers think is the: a. preoperational stage b. mental combinations stage c. tertiary circular function stage d. sensorimotor stage - CORRECT ANSWER a A preschool boy whose parents have separated and are beginning divorce procedures: a. may think that he caused the divorce by misbehaving b. should not be told of the impending divorce until the parents are sure of their decision c. is likely to experience gender identity confusion d. should be able to make a decision about which parent he prefers living with - CORRECT ANSWER a Which behavior would you expect to decrease during the preschool years? a. rough-and-tumble play b. instrumental aggression c. hostile aggression d. cooperative play - CORRECT ANSWER b A preschool child who says that the sky is blue because it is his favorite color is illustrating the concept of: a. symbolic thinking b. egocentrism c. centration d. imaginary audience - CORRECT ANSWER b Which of the following strategies would not be appropriate to include as part of your management of a 9yo boy who is obese? a. referral to nutritionist for weight reduction plan b. increase physical exercise c. behavior modification strategies to deal with stress and/or reinforce treatment plan d. involve family in management program - CORRECT ANSWER a Which of the following issues or concepts is relevant to the school-aged child? a. operational thinking b. initiative c. concrete operations d. separation-individuation - CORRECT ANSWER c The 1st physical sign indicating the onset of female puberty is: a. sparsely distributed fine, pale pubic hairs b. breast buds c. menarche d. peak height velocity - CORRECT ANSWER b Which of the following findings would be helpful in distinguishing obesity vs. large body frame in an adolescent who is concerned with her weight? a. tricep skinfold measurement b. weight-for-height ratio c. body mass index d. percent of ideal body weight - CORRECT ANSWER c the most common form of child abuse seen in pediatric primary care is: a. burns b. fractures c. soft tissue injury d. shaken baby syndrome - CORRECT ANSWER c a differential diagnosis for child abuse would include all of the following except: a. birth marks b. unintentional injury c. inadequate parenting d. Prader-Willi syndrome - CORRECT ANSWER d Which of the following symptoms are not typical of a child with ADHD? a. easily distracted b. difficulty playing quietly c. doesn't follow directions d. frequently angry & resentful - CORRECT ANSWER d Which of the following situations does not necessarily warrant immediate mental health assessment and/or referral? a. a 13yo girl who has been "down" for the last mo with varied somatic complaints b. a 9yo boy whose parents recently separated and filed for a divorce and seems to be doing well c. a 16yo girl who has a hx of long standing depression but seems to be doing well in school d. a 15yo boy who expresses suicidal thoughts - CORRECT ANSWER b The diagnostic criteria for autistic disorders includes which of the following? a. speech delay, ataxia, mental retardation b. impairments in social interactions, interpersonal communication, and staring spells c. mental retardation, impairments in social interactions, & stereotypical restricted pattern of interests & activities c. impairments in social interactions, in interpersonal communication, and stereotypical restricted pattern of interest and activities - CORRECT ANSWER d In addition to specific academic skill deficits, learning disabilities are commonly associated with which of the following characteristics? a. perceptual-motor impairments, normal motor function b. perceptual-motor impairments, impulsiveness c. perceptual-motor impairments, down syndrome d. perceptual-motor impairments, lack of impulsiveness - CORRECT ANSWER b Preventative health guidelines include references to: a. immunizations, health screening, disease prophylaxis, education, and infection control b. immunizations, counseling, health screening, disease prophylaxis, and education c. health screening, disease prophylaxis, counseling, and CPR d. health screening, disease prophylaxis, education, immunizations, and CPR - CORRECT ANSWER b The nurse practitioner role was initially established to: a. improve access to care and partially solve physician shortage b. reduce the nursing shortage and improve access to care c. improve working conditions of nurse while improving access to care d. improving nursing's image through expansion of the role - CORRECT ANSWER a early nursing research focused on: a. the response of policy makers to the nursing shortage b. the effectiveness of the NP as a primary care giver c. an effort to demonstrate quality and cost effectiveness of NP d. the role of the NP as physician extender - CORRECT ANSWER c Which of the following is not a major factor influencing healthcare delivery services? a. provider b. payers c. insurers d. agencies - CORRECT ANSWER d All definitions of primary health care include: a. the concept of universal access and accountability b. the concept of universal access and AIDS prevention c. the concept of universal access and focus on self-responsibility for health d. the concept of universal access and a focus on reimbursement for services rendered - CORRECT ANSWER a standards of practice are: a. authoritative statements used to measure quality b. used to measure outcome but are not authoritative c. designed for legal purposes d. not designed for legal purposes and cannot be used to measure quality - CORRECT ANSWER a Quality improvement activities include: a. patient satisfaction surveys only b. peer review, patient satisfaction surveys, chart audits c. defining four practice domains d. systems to decrease risk of injury to patients - CORRECT ANSWER b Most risk management programs are based on the assumption that: a. many injuries to patients are preventable b. most legal liability is a result of poor documentation c. most injuries to patients are not preventable d. malpractice insurance is generally unnecessary - CORRECT ANSWER a If an APRN practices beyond his/her scope: a. malpractice insurance will protect him/her from a charge of practicing medicine without a license b. malpractice insurance will not protect him/her from a charge of practicing without a license c. he or she is legally accountable to the certifying body d. The collaborating physician is legally accountable to the certifying body - CORRECT ANSWER b Standards of practice may be used to: a. establish minimal levels of performance b. establish reimbursement schemes for APRN c. Mandate nursing practice across the nation d. Mandate nursing practice in certain states - CORRECT ANSWER a Scope of practice: a. is identical across the states b. is determined by the federal government c. is mandated by the federal government d. varies from state to state - CORRECT ANSWER d The knowledge base of the APRN is based on: a. medical content b. theoretical content only c. scientific content and theory d. theory and research - CORRECT ANSWER c The role of the APRN has traditionally focused on: a. the delivery of primary health care to all people b. the delivery of acute health care to all people c. chronic care d. the medical model - CORRECT ANSWER a Legal authority for APRN practice is granted by: a. federal law b. regulations from the department of health and human services c. state law and regulations d. the board of medicine in most states - CORRECT ANSWER c malpractice insurance: a. protects an APRN from charges of practicing medicine without a license when they are practicing outside the legal scope of practice b. does not protect an APRN from charges of practicing medicine without a license when they are practicing outside the legal scope of practice c. does not pay for legal defense if the APRN is practicing beyond the legal scope of practice d. is important, but should not be purchase if the facility in which the APRN is employed carries good c - CORRECT ANSWER b Current prescriptive authority for APRN: a. varies among states b. is fairly consistent among the states c. provides DEA numbers for APRN d. allows APRN to move freely from state to state - CORRECT ANSWER a Managed care is a term that describes: a. an established system of healthcare delivery that is mandated by the federal government b. a network of providers who contract to provide services for a specific group of enrollees c. a system that does not recognize APRN as a primary provider d. a network of hospitals and nursing homes that provide care to chronically ill people - CORRECT ANSWER b Certification is: a. procedure through which the government appraises and grants certification to the APRN b. Granted by the individual states c. Governed by each state's Board of Nursing d. A process in which a nongovernmental agency or group verifies that an APRN has met certain predetermined standards for specialty practice. - CORRECT ANSWER d Licensure is: a. federal process that is used to standardize healthcare facilities b. is granted by a state government agency and grants permission to engage in the practice of a given profession c. Cannot be used to prohibit anyone from practicing a given profession d. is a federal process that is used to standardize educational programs - CORRECT ANSWER b Reimbursement under managed care: a. requires that the provider accept the financial risk for the care provided to a specific population of enrolled patients b. requires that the managed care organization accept the financial risk for the care provided to a specific population of enrolled patients. c. does not reward efficient care delivery d. is not available to APRN - CORRECT ANSWER a An integrated delivery system: a. one that delivers high quality care but is often not cost effective b. delivers a vertical integration of services with capitated payment c. does not include rationing of resources d. does not include a capitated payment scheme - CORRECT ANSWER b The APRN Consensus Model is: a. mandate from the NCSBN that defines advanced practice nursing b. a proposed regulatory model for advanced practice nursing c. a proposal for federal legislation for advanced practice nursing d. approved only by the american nurses association - CORRECT ANSWER b Which of the following is the most accurate description of the Dubowitz tool? a. it evaluates the physical condition of the newborn at birth. b. the criteria evaluated are influenced by labor & birth; therefore a second exam may be indicated to pick up any changes in the newborn c. It is a gestational assessment tool that evaluates the infant's newborn reflexes including moro reflex, tonic neck, grasping, rooting, and sucking d. Estimations of gestational age are performed by examining physical - CORRECT ANSWER d A parent complains that their child awakens crying and agitated during the last half of night sleep. The child is awake and can be comforted. The parent denies this child is experiencing a. sleep terrors b. nightmares c. somniloquy d. sleep starts - CORRECT ANSWER b The parents of a 2 year old are concerned that their child is having temper tantrums in public settings. What is the BEST option? a. The temper tantrum is an indication that the child is tired and needs to go home. b. Make sure to ensure safety while ignoring the child's display of behavior. c. You can pick up the child and take him to a quiet place for a time out. d. In a public place, it is okay to give into the child's desires to maintain peace. - CORRECT ANSWER b The most common hematological presentation of leukemia in children is anemia along with which of the following? a. neutropenia and thrombocytopenia b. leukocytosis and monocytosis c. neutropenia and blast cells on peripheral smear d. monocytosis and blast cells on peripheral smear - CORRECT ANSWER a Recommendations regarding the current management of croup with increased work of breathing include the use of a. nebulized bronchodilators b. oral corticosteroids c. oral antibiotics d. cool mist therapy - CORRECT ANSWER b Which counseling for parents of a 2 year old with an elevated lead level, would be effective in decreasing lead exposure? a. avoid kerosene heaters b. dry dusting c. sweeping d. high efficiency particulate air (HEPA) filters for vacuuming - CORRECT ANSWER d Which of the following should be included in the plan of care for preterm infants? a. use of a car seat without a shield harness b. supplemental vitamin D at 4 months for breast fed infants c. immunizations beginning at 2 months corrected age d. evaluation of vision at 3 months - CORRECT ANSWER a Which represents information obtained during a review of systems (ROS)? a. mother of the child has asthma b. height and weight c. history of birthmarks or skin lesions d. number of previous hospitalizations - CORRECT ANSWER c A teenager sustains a puncture wound in the sole of a foot after stepping on a protruding nail. The most appropriate INITIAL management of this injury would be a. debridement of epidermal edges b. wound probing for retained material c. initiation of prophylactic antibiotics d. superficial irrigation of the wound - CORRECT ANSWER d A 15 year old previously healthy teen is having school problems, feelings of sadness, insomnia, and disinterest in activities previously enjoyed. The parents reported the teen spends hours alone, avoiding social situations. When questioned, the teen denies any desire or plan to hurt himself or others. Which would be the MOST appropriate INITIAL management of this patient? a. prescribe a tricyclic antidepressant b. arrange for home schooling c. refer for immediate inpatient evaluation d. refer fo - CORRECT ANSWER d Nurse practitioners can best assist teens in reducing behaviors that present health risks while socializing with peers by a. maintaining awareness of local social events for teens b. incorporating conversations with teens regarding safe sex, substance use, and body art with regular health visits c. discussing safe sex, substance use, and opportunities for body piercing and tattoos with teens in the presence of their parents d. discouraging teens to participate in group activities without adult s - CORRECT ANSWER b For the parents of an 8 month old, the MOST appropriate age-related anticipatory guidance related to safety should include which of the following? a. cover electrical outlets, keep small objects out of reach, turn car seat to forward-facing position b. keep medications out of reach, continue to use car seat in rear-facing position, cover electrical outlets c. avoid finger foods that can be easily aspirated, teach stranger safety, place Mr. Yuk stickers on toxic materials d. keep matches out of r - CORRECT ANSWER b The most important PRIMARY care goal for a school aged child with a prosthetic heart valve is to a. prohibit exercise and sports participation b. maintain endocarditis prophylaxis c. provide home schooling d. maximize carbohydrate calories for growth - CORRECT ANSWER b A term infant born by C-section, with birthweight of 5 lbs 5 oz (2.4 kg), develops tachypnea with a respiratory rate of 68 at 2 hours of life. The most likely diagnosis is a. transient tachypnea of the newborn b. pneumothorax incurred during delivery c. respiratory distress syndrome d. congenitally acquired pneumonia - CORRECT ANSWER a 16yo Sarah makes the following statements to you during a health visit. Which of the following pieces of information should not be kept confidential? a. "I have been sexually active with 3 of my boyfriends" b. I sometimes smoke marijuana c. I want to get pregnant d. Sometimes I feel like ending my life - CORRECT ANSWER d In performing a physical examination on a 9mo old infant, which of the following developmental fears woud not be appropriate for you to consider? a. Stranger anxiety b. Pain c. Separation from parents d. Bodily harm - CORRECT ANSWER d When performing a physical examination on a toddler, which of the following body parts would you examine last? a. heart & lungs b. abdomen & genitals c. ears & throat d. hips & extremities - CORRECT ANSWER c Role play with equipment during the course of a physical exam would be most beneficial with with of the following age groups? a. toddlers b. preschoolers c. young school aged children d. older school aged children - CORRECT ANSWER b Providing reassurance of "normalcy" during the course of an exam would be most important for: a. preschool children b. young school aged children c. older school aged children d. adolescents - CORRECT ANSWER d Blood pressure should be measured at well child visits, beginning at age: a. 2yo b. 3yo c. 4yo d. 5yo - CORRECT ANSWER b Head & chest circumferences should be equal at: a. 6mo b. 1yo c. 2yo d. 3yo - CORRECT ANSWER b The anterior fontanel usually closes by: a. 2mo b. 6mo c. 18mo d. 24mo - CORRECT ANSWER c An infant should no longer have a head lag when pulled from the supine to sitting position at what age? a. 2mo b. 4mo c. 6mo d. 9mo - CORRECT ANSWER c A white instead of red reflex upon eye exam of a 1yo child would suggest: a. an accommodation error b. retinoblastoma c. papilledema d. retinal detachment - CORRECT ANSWER b Physiological splitting of the second heart sound during inspiration in a child: a. is normal b. should be evaluated with an EKG c. suggests an ASD d. should be referred to a cardiologist - CORRECT ANSWER a Gynecomastia in a male may not be a finding in which of the following? a. normal pubertal development b. steroid usage c. hyperthyroidism d. testicular tumor - CORRECT ANSWER c Which of the following would usually not be considered a sign of a pituitary tumor in a an adolescent female? a. dysfunctional uterine bleeding b. galactorrhea c. loss of peripheral vision d. increase in headaches - CORRECT ANSWER a Which of the following infant reflexes should not disappear by 6mo of age? a. moro b. rooting c. tonic neck d. plantar grasp - CORRECT ANSWER d Spasticity in an infant may be an early sign of: a. neurofibromatotis b. hydrocephalus c. cerebral palsy d. muscular dystrophy - CORRECT ANSWER c Which of the following does not suggest a UTI? a. increased protein b. increased WBCs c. increased RBCs d. increased nitrites - CORRECT ANSWER a A mantoux test in a child with no risk factors is considered positive with a reaction of: a. at least 5mm induration b. at least 8mm induration c. at least 10mm induration d. at least 15mm induration - CORRECT ANSWER d The PNP teaches new parents that when breastfeeding is well established they can expect baby to have: a. as many as 4 wet diapers each day b. a stool every 3-4hrs c. 1 wet diaper an hour d. 5-6 wet diapers and 2-3 stools each day - CORRECT ANSWER d Cholesterol screening should be done: a. children 2yrs+ who have a parent with a total cholesterol level of 240mg/dL or greater b. once all children at 6yrs of age c. overweight children with a family history of premature cardiovascular disease d. once for all children at 12yrs of age - CORRECT ANSWER a For which of the following screening tests should children fast for 12hrs before the test is done? a. total cholesterol b. serum chemistry profile c. serum lipid profile d. hematocrit - CORRECT ANSWER c Which of the following is the most important history taking question for sports evaluation? a. has the child ever had head injury? b. has the child ever fainted or lost consciousness during exercise? c. does the child ever get short of breast with exercise? d. has the child ever had prior surgeries? - CORRECT ANSWER b Which of the following topics would not be appropriate to include when providing anticipatory guidance to the parent of a 4mo old infant? a. introduction of solid foods b. teething c. negativism d. introduction of a cup - CORRECT ANSWER c Which is the correct order for introduction of solid foods to an infant? a. fruits, cereal, vegetables, & meats b. cereal, meats, vegetables, fruits c. fruits, cereal, meats, and vegetables d. cereal, vegetables, fruits, meats - CORRECT ANSWER d Which of the following topics is not appropriate to include when providing anticipatory guidance to the parent of an 18mo old? a. temper tantrums b. toilet training c. dental care d. stranger anxiety - CORRECT ANSWER d Appropriate anticipatory guidance for the parents of an 8yo girl should not include: a. preparation for an increase in nervous mannerisms & restless activity b. preparation for pubertal changes c. information that friends begin to serve as allies against adults d. information that their daughter will take idols & heros - CORRECT ANSWER a A new mom calls the PNP on post partum day 5. She reports her newborn wants to nurse for 30min every 2hrs. Which of the following is your best responses? a. This is a very healthy breastfeeding pattern. Be sure to rest when you can. You are doing a great job. b. Your baby is too demanding. If you can continue to feed that often you will spoil your child. c. You are not making enough milk and your baby will need to go to the ER to be evaluated. d. Your baby has an oral fixation and you should off - CORRECT ANSWER a A pelvic exam should not be performed on which of the following adolescents? a. a 14yo who is sexually active b. a 15yo who has just started menarche c. a 17yo who is having irregular menses d. an 18yo healthy female - CORRECT ANSWER b A 9yo boy presents with a fever of 102F and complaints of leg pains. His mother reports that he had upper respiratory infection with a sore throat approximately two weeks ago, which subsided without therapy. On physical exam, he has tender, swollen knees bilaterally. His heart rate is 120/min and blowing systolic murmur is heard at apex. No murmur was noted on previous well-child visit. The most likely diagnosis is: a. Kawasaki disease b. Rheumatic fever c. Sickle cell anemia d. Viral illness - CORRECT ANSWER b Which of the following primary interventions assists in the prevention of adolescent depression? a. recommending the allocation of social isolation time b. anticipatory guidance that mild depression is normal c. recommending help from professionals rather than friends d. discussing changes associated with adolescence - CORRECT ANSWER d Foods containing high levels of iron appropriate to suggest for a 2 yo with iron deficiency anemia include a. almonds, brown rice, peanut butter b. brown rice, Cheerios, kidney beans c. pancakes, peanut butter, Cheerios d. pancakes, broccoli, almonds - CORRECT ANSWER b Adolescents who receive teaching about condoms should also have information about a. the brands least likely to break during intercourse b. the application of petroleum jelly to prevent dry intercourse c. the us of nonoxynol 9 as a lubricant and spermicide d. the use of oil based lubricants as safe lubricants - CORRECT ANSWER c Which institution legally defines the scope of practice for pediatric nurse practitioners? a. graduate schools b. certification board c. state legislation d. NP membership organiation - CORRECT ANSWER c A healthy 24 month old can be expected to have achieved which of the following language milestones? a. counts three objects correctly b. has a 30-50 word vocabulary c. states full name d. tells a story - CORRECT ANSWER b Which symptoms would prompt the ordering of a genetic test for Prader- Willi syndrome? a. obesity, daytime hyperactivity, increased libido b. large hands/feet, thin saliva c. neonatal hypotonia, feeding difficulties d. failure to thrive, hyperphagia, clitoromegaly - CORRECT ANSWER c Emphasis on promoting healthy behavioral changes for the overweight child should include which of the following? a. establishing family goals b. maintaining family holiday meal rituals c. weighing child daily d. using weight loss as a sign of success - CORRECT ANSWER a Which should always be included in the evaluation of a child with a suspected learning disorder? a. full history and physical exam b. Denver II developmental assessment c. genetic and chromosomal testing d. intelligence quotient measurement - CORRECT ANSWER a What is the MOST important safety question to ask a 17 yo? a. Are you aware of ways to avoid accidents? b. Do you play violent video games more than 2 hours a day? c. Do you always wear a seat belt when driving or riding in a car? d. Is there a gun anywhere in your house? - CORRECT ANSWER c A MOST concerning cardiac finding in the initial newborn assessment is a. a soft, grade III/IV systolic murmur b. a cuff blood pressure of 65/35 c. decreased femoral pulses d. a regular heart rate of 95 while sleeping - CORRECT ANSWER c The American Academy of Pediatrics (AAP) recommends that child ASSENT to participate in clinical research requires evidence of which of the following? a. full understanding of the risk and potential benefits of the treatment b. rejection of non-research alternatives c. acceptance of unfamiliar procedures d. deveolpmentally appropriate understanding of the nature of his condition - CORRECT ANSWER d A 10 yo obese child whose family history is unknown has a total cholesterol (TC) of 205 when first screened for hyperlipidemia. The MOST appropriate course of action is to a. repeat TC level in 2-4 weeks b. repeat TC level after 3 months on a low-fat diet c. perform a lipoprotein analysis now d. perform a lipoprotein analysis after 3 months on a low-fat diet - CORRECT ANSWER c Internationally adopted children from ANY country should receive testing for a. tuberculosis, hep B and C, syphilis, HIV b. measles, hep A and C, tuberculosis, malaria c. syphilis, cytomegalovirus, hep A and C, HIV d. measles, hep A and C, cytomegalovirus, malaria - CORRECT ANSWER a In addition to screening by checklist of a 17 month old suspected of having autism, what other testing should be included? a. occupational therapy evaluation b. MRI of the brain c. hearing screening d. chromosomal analysis - CORRECT ANSWER c In supporting parents of a school aged child with a learning disability, the most important PNP role is to: a. understand the development of the IEP (Individualized Education Plan) and 504 Plan b. have a collaborative relationship with the local psychiatrist who works with learning disabled children c. identify the participants in the student study team or child guidance group d. prescribe the most common medications used for ADHD - CORRECT ANSWER a The parents of a 5 yo are frustrated by their child's need to select her own clothing, despite weather and safety. The BEST example of a parent statement to the child that wil promote self esteem a. "You must wear shoes and socks, but you can choose between the red or black ones." b. "Go ahead and wear the sandals, but you will be cold." c. "Wear the sandals, but take your shoes and socks in case you change your mind." d. "It is too cold to wear sandals, stay in your room until you decide to put - CORRECT ANSWER a Anticipatory guidance for a parent with a child whose venous lead level is 15 mcg/dL should include a. retesting in 2 months along with an evaluation of the living environment b. retesting required every month along with a neurodevelopmental evaluation c. emergent retesting required and evaluation for intravenous chelation d. an abdominal x-ray with bowel decontamination - CORRECT ANSWER a Diagnostic laboratory testing indicated for metabolic disorders in the newborn period always includes a. glycosylated hemoglobin b. urine electrolytes c. serum amino acids d. serum AST, GGT, ALT - CORRECT ANSWER c Developmental milestones for a 5 month old include a. rolling from back to front, reaching for an objects and smiling spontaneously at familiar faces b. sitting unsupported, transferring an object form hand to hand, smiling, and cooing c. making sounds that mimic one syllable words, rocking on hands and knees, fine pincer grasp d. crying when parent leaves the room, beginning to creep, holding onto a toy - CORRECT ANSWER a Which BEST represents the description of Tanner stage 3 in an adolescent female? a. areola and breast enlarges with no separation of contours, presence of pubic hair over the middle of the pubic bone c. areola and breast enlarges with no separation of contours, appearance of fine, downy pubic hair along the labia d. distinctive projection of the areola, appearance of fine, downy pubic hair along the labia - CORRECT ANSWER a Children with Down syndrome have 15 to 20 times incidence of developing which of the following as compared to other children? a. asthma b. neuroblastoma c. leukemia d. diabetes - CORRECT ANSWER c Which of the following is the most appropriate information to use in providing anticipatory guidance for parents with a colicky baby? a. colicky babies are hypersensitive to stimuli b. teething can stimulate a colicky response c. symptoms increase at 3-4 months of age d. fever is often an accompanying symptom of colic - CORRECT ANSWER a The parents of an alert, active 4 yo diagnosed with immune thrombocytopenia purpura (ITP) are frustrated because they feel "nothing was done" for their child. Physical exam reveals multiple purpura and petechiae, but no mucocutaneous bleeding. A repeat CBC shows platelets at 45,000/mm^3, mild anemia and no abnormalities of the red or white blood cells. The NEXT step should include a. discussion regarding safety concerns b. recommendation of hospitalization c. initiation of steroid therapy d. ref - CORRECT ANSWER a The risk factor that is MOST predictive of suicide in an adolescent patient is a. major mood disorder b. previous suicide attempt c. substance and alcohol abuse d. family history of suicide - CORRECT ANSWER b The immunization history of a 23 month old is as follows: Immunizations: 9 months: DTaP, HIB, Hep B, IPV, PCV 7 13 months: DTaP, HIB, Hep B, IPV, PCV 7, MMR, Var The most appropriate recommendation is to now administer a. Hep B, IPV, PCV 7 b. DTaP, HIB, Hep B c. DTaP, HIB, Hep B, IPV, PCV 13 d. DTaP, HIB, Hep B, IPV, PCV 7 - CORRECT ANSWER c An infant brought to the clinic is diagnosed with failure to thrive (FTT), persistent growth deficiency of the head, and delayed development. Some unusual facial abnormalities are noted. The mother has a history of consuming large amounts of alcohol. The PNP suspects fetal alcohol syndrome (FAS) and refers the infant: A) For consultation with an endocrinologist B) For consultation with a neurologist C) For a genetic workup D) To a First Steps or Birth to Three Program - CORRECT ANSWER d A parent brings a 1-week-old neonate into the clinic. The mother is concerned that the infant's testicles do not seem "even." The PNP notes that the cremasteric reflex is absent and the right testicle cannot be palpated in the scrotum. The PNP should: A) Not allow the neonate to be discharged from the hospital B) Order a sonogram C) Consult with the physician D) Refer the neonate to the urology clinic - CORRECT ANSWER c A 10-year-old child has been referred for evaluation of scoliosis, which was identified in a school screening program. On physical examination the PNP finds a curvature of the spine when it is anteriorly flexed from the trunk. There is slight shoulder asymmetry, with prominence of the scapula. The curvature is at 10 degrees. The PNP should: A) Refer the child to an orthopedist for further evaluation B) Inform the parents that the child is going through a growth spurt and schedule an appointment - CORRECT ANSWER c A father brings his 9-month-old because the infant has been tugging on the right ear. While examining the ear, the PNP notes that the infant has cranial and facial asymmetry. There is no evidence of otitis media. The PNP should then: A) Refer the child to a neurologist for consultation B) Perform an extensive developmental assessment C) Continue to observe the infant, and recheck cranial shape at the 1-year well child visit D) Consult with a physician and obtain a cranial radiologic film, CT sca - CORRECT ANSWER d A 9-year-old child is brought to the clinic for the evaluation and treatment of a rash. While examining the child, the PNP detects a speech dysfluency. The most appropriate plan of treatment should include: A) Treating the rash and scheduling a well child visit to address the speech problem B) Treating the rash and making no referral because it is too late for speech therapy to be beneficial C) Using a clinical screening tool to determine whether a language disorder exists D) Treating the rash a - CORRECT ANSWER d A 12-month-old child is brought to the clinic for a repeat urinalysis and culture. Results of the urine culture obtained at the 12-month well-child visit were abnormal. There is no previous history of UTI and no fever of unknown origin. The PNP finds that the child is not circumcised and the foreskin does not retract with moderate traction. The parents state that they have not routinely retracted and cleaned under the foreskin. The most appropriate treatment would be to: A) Refer to a urologist - CORRECT ANSWER a A 1-year-old child is brought to the clinic with a temperature of 102ºF and left flank pain. The urine dipstick test indicates nitrates and leukocyte esterase. The presenting signs and symptoms suggest left pyelonephritis. Based on the data, the most appropriate action for the PNP is to: A) Send urine for culture and sensitivity testing B) Provide symptomatic treatment for 24 hours, and repeat the urine dipstick test C) Refer the child to a urologist for diagnosis and treatment D) Consult with - CORRECT ANSWER d An 18-month-old child is brought to the clinic with a rash and temperature of 103ºF for the past 5 days. The child has bilateral conjunctivitis; dry, cracked lips; and strawberry-like tongue. Cervical lymph adenopathy and a heart murmur are noted on physical examination. The PNP should: A) Order a CBC B) Order an ECG C) Refer the child to the infectious disease clinic D) Consult with the physician - CORRECT ANSWER d An 18-month-old child is being evaluated in the clinic for falling with increasing frequency and loss of speaking ability. The PNP considers: A) Urine amino acid screening B) Referral to a physical therapist C) Referral to a pediatric neurologist and MRI of the brain D) Chromosomal studies for Tay-Sachs disease - CORRECT ANSWER c The PNP is in the nursery to examine a neonate. The results of the neonatal screen are noted in the chart. The T4 level is low, and the TSH level is elevated. The PNP should: A) Consult with the physician, and repeat the tests B) Discharge the neonate, and schedule a follow-up visit in the office in 1 week C) Examine the neonate for signs of hypothyroidism D) Begin the neonate on thyroxine based on the test results - CORRECT ANSWER a A newly delivered mother wants to breastfeed but admits to using marijuana occasionally. She reports last using marijuana over 6 weeks ago. Her drug toxicology screen is positive for marijuana. What would the PNP advise regarding breastfeeding? A) Discourage the mother from using marijuana and recommend not breastfeeding until she stops using marijuana and her toxicology screen is negative B) Recommend breastfeeding C) Tell the mother she is not a good candidate for breastfeeding because her tox - CORRECT ANSWER d The father of a 10-month-old is concerned because over the lack of eruption of primary teeth in the infant. The PNP suggests: A) This is within the range of normal dental development B) Consulting the pediatrician at the 15-month-old well-child visit C) The infant's teeth will erupt on a similar schedule as the parents D) An immediate referral to the dentist - CORRECT ANSWER a A mother brings a 4-year-old child to the clinic for a well-child visit. She is concerned that the child has autism. Atypical behavior traits that appear in autistic children include: A) An intense relationship with the mother during the first year of life B) Obsession with repetitive hand cleansing C) Onset of behaviors noted after the third year of life D) Impairment in reciprocal social interaction - CORRECT ANSWER d The PNP sees a 14-year-old adolescent in the clinic. The adolescent is sexually active and does not want to become pregnant. There is a history of high blood pressure and cardiac problems in the family. The patient states that she does use condoms to prevent STDs. The patient asks the PNP what the best form of birth control to use to prevent pregnancy? A) Diaphragm B) Depo-Provera C) Ovral D) Intrauterine device (IUD) - CORRECT ANSWER b The PNP is discussing laser therapy for a capillary vascular malformation with the parents of a child and advises the parents: A) "Multiple treatments over many months may be required." B) "Transient edema can be expected to last several months." C) "Gray-black discoloration will fade in 3 to 4 days." D) "The birthmark will gradually fade over 1 to 2 years." - CORRECT ANSWER a A 16-year-old adolescent comes to the clinic because of a latex allergy. The adolescent is concerned about sexually transmitted diseases (STDs) and using condoms. He states that he and a girlfriend have been "experimenting." The PNP suggests that: A) The girlfriend should use a cervical cap B) The girlfriend should use a diaphragm C) The adolescent boy should use a double condom (i.e., a non-latex condom covered by a latex condom) D) The girlfriend should take oral contraceptives - CORRECT ANSWER c A 2-month-old breastfed infant weighing 11 lb 8 oz (birth weight 8 lb) is seen in the clinic. The mother appears exhausted and reports that she is nursing on demand, every hour for 5 to 10 minutes. The PNP suggests that the mother: A) Supplement the infant with formula after breastfeeding B) Nurse longer than 10 minutes at each feeding C) Give 1 tablespoon of rice cereal once a day to the infant D) Continue nursing on demand - CORRECT ANSWER b A parent is concerned her young child may have food allergies. "They run in my husband's family." The parent would like to know the most common food allergies. The PNP responds: A) Dairy products, nuts, eggs B) Soy milk, bananas, nuts C) Fish, wheat cereal, eggs D) Beef, oat cereal, wheat cereal - CORRECT ANSWER a While obtaining a family history, a mother tells the PNP that her maternal grandfather, her mother, herself, her son, and her daughter have von Willebrand disease. The PNP describes this inheritance pattern as: A) Mosaic B) Autosomal recessive C) Autosomal dominant D) X-linked - CORRECT ANSWER c A grandfather brings his 8-month-old grandson to the clinic and states that the infant screams and cries whenever anyone besides the grandfather comes near. The PNP diagnoses stranger anxiety. The grandfather wants to know if he should be concerned. The PNP responds: A) Force the infant to be held by others B) Reassure the parents this is normal and will pass C) Ignore the screaming D) This is not normal, and a referral to a child psychologist may be helpful - CORRECT ANSWER b A 17-year-old adolescent who is at the clinic for a "football physical" relates to the PNP that he and a girlfriend have had sex two times this past month. The adolescent is concerned about preventing pregnancy and protection from STDs. This was his first sexual experience, and he has questions about condoms but does not want his parents to know that he is sexually active. The PNP offers the following information: A) "Use natural condoms because they are easier to apply." B) "Use a petroleum-bas - CORRECT ANSWER c A 12-month-old child is brought to the clinic. On physical examination the PNP notes that the child's right eye drifts to the nasal side of the eye. The PNP explains to the mother that a complication of this condition may be: A) Strabismus B) Astigmatism C) Amblyopia D) Myopia - CORRECT ANSWER c Which of the following explanations can help parents of preschool-aged children distinguish night terrors from nightmares? A) Nightmares are scary dreams followed by complete awakening B) The child does remember night terrors and talks about them in the morning C) Night terrors usually occur during the second half of the night D) The child does not remember nightmares in the morning - CORRECT ANSWER a The mother of a 2-year-old child is concerned about the child's fear of the dark. The PNP responds that: A) Parents should not allow the child to withdraw from a fearful situation B) Fears that interfere with social development resolve as the child develops C) Young children with fears tend to have phobias as adults D) Fears of this type are a normal developmental process - CORRECT ANSWER d A parent asks at what age separation anxiety is most often expected. You explain to the parent that separation anxiety usually occurs at: A) 12 to 18 months of age with peaking at 24 to 26 months B) 9 to 15 months of age with peaking at 20 to 22 months C) 3 to 9 months of age with peaking at 18 to 20 months D) 1 to 6 months of age with peaking at 9 to 12 months - CORRECT ANSWER c What suggestions would the PNP offer a father calling because his 3-month-old infant is passing hard stools? A) "Add honey to a 4-oz bottle of water." B) "Time will take care of the problem." C) "Offer the infant a little extra water with added pasteurized apple or prune juice." D) "Hold the infant upright so that gravity can help the stool pass." - CORRECT ANSWER c An 18-month-old child is brought to the office for irritability, nightmares, and generally being tired for the past few nights. The child has had an upper respiratory tract infection for which the grandmother has administered an over-the-counter medication for rhinorrhea. The child is afebrile and alert, with obvious clear rhinorrhea. The PNP: A) Orders sinus radiologic studies B) Prescribes amoxicillin C) Suggests that the mother administer acetaminophen D) Discusses side effects of decongestan - CORRECT ANSWER d The PNP preceptor is discussing with the PNP student the use of an aminoglycoside in the treatment of an infant in a special care nursery. The use of an aminoglycoside may result in the following complication: A) Permanent liver damage B) Ototoxicity and nephrotoxicity C) Bone marrow depression (if not properly monitored) D) Encephalopathy - CORRECT ANSWER b The mother of a 3.5-year-old child is counseled about the need to maintain a therapeutic level of an antibiotic prescribed to treat otitis media. The PNP informs the mother that the term therapeutic level refers to the: A) Concentration of a drug that is required to elicit the desired clinical response without causing toxic effects B) Lowest concentration of the drug reached between doses C) Amount of drug taken into the body, which is equal to the amount excreted D) Time required for 50% of a d - CORRECT ANSWER a A mother calls the office because her 2.5-year-old has a fever of 101°F (38.2°C). The child is playing and taking plenty of fluids. The PNP advises the mother: A) To dress the child warmly to prevent chilling B) An appointment is needed to determine the cause of the fever C) A fever can actually be a good thing as long as the child is comfortable D) To treat the fever with acetaminophen or ibuprofen if it is greater than 100.1°F - CORRECT ANSWER c An infant with congenital hypothyroidism is to start treatment with Levothyroxine. When describing the administration of the medication to the parents, it is important to reinforce: A) Evening dosing is preferable to morning dosing B) Cow's milk formula interferes with absorption C) Dosage must be monitored and increased significantly as the child grows D) A crushed tablet should be administered to the infant each morning - CORRECT ANSWER c An 8-year-old child is evaluated for "growing pains." The pains are intermittent and last up to 2 hours. The child awakens at night, and the pain is not specific to joints. The physical examination is normal. The PNP decides to order: A) A monospot test B) A test to determine ESR C) A throat culture D) An x-ray film of the left hand - CORRECT ANSWER b A 5-year-old child has a 3-day history of sore throat, fever (101ºF to 103ºF), headache, and one episode of vomiting. The child's symptoms started with a runny nose and slight cough yesterday. The physical examination is normal except for +2 to +3 tonsils, palatal petechiae, erythema, and a clear runny nose. The PNP's best course of action is to: A) Perform a throat culture before prescribing an antibiotic B) Discuss symptom control for seasonal allergies C) Prescribe antibiotics for strep thr - CORRECT ANSWER a A 2-year-old child is brought to the clinic with periumbilical pain and vomiting that began this morning. The examination is unremarkable with the exception of tenderness noted on abdominal palpation. The differential diagnosis includes appendicitis. What should the PNP do next? A) Order an abdominal x-ray examination, and instruct the parents not to feed the child B) Order ultrasonography, and instruct the parents to feed the child only clear liquids C) Advise "watchful waiting," and instruct t - CORRECT ANSWER c A 15-year-old adolescent is brought to the clinic complaining of chronic nasal congestion and fatigue. The father gave the adolescent an antihistamine that did not help with the congestion. Upon examination a small perforation is noted in the nasal septum. The PNP suspects: A) Foreign body perforation B) Allergic rhinitis C) Chronic use of cocaine D) Nasal septal defect - CORRECT ANSWER c A 2-month-old infant is brought to the clinic because of continued vomiting. Because the PNP suspects GER, time has been arranged for the visit to include a feeding. During the feeding the PNP observes the infant for: A) Calm, attentive feeding position B) Head moving back and forth and searching C) Disinterest in the bottle D) Back arching and crying - CORRECT ANSWER d The PNP examines a 12-year-old child with a dry, unproductive cough; a low-grade fever; and copious amounts of sputum. On physical examination, coarse breath sounds and moist rales are heard in the upper part of the lungs. The PNP makes the diagnosis of bronchitis and treats with: A) A decongestant and ibuprofen B) Amoxicillin and a cough suppressant C) An albuterol inhaler and acetaminophen D) An antihistamine; a cough suppressant, such as dextromethorphan hydrobromide; and use of a vaporizer - CORRECT ANSWER b A foster mother phones the office because her 10-year-old child has knocked out a permanent tooth. The PNP instructs the foster mother to: A) Rinse the tooth in warm water, and call the dentist B) Place the tooth back in the socket, and call the dentist immediately C) Take the child and the tooth to the dentist tomorrow D) Take the child to the hospital emergency room immediately - CORRECT ANSWER b Which of the following is a true statement regarding nocturnal enuresis? A) Psychologic stressors are a common cause of nocturnal enuresis B) There is a true organic cause in most cases of nocturnal enuresis C) Physiologic causes of enuresis include having a small bladder capacity D) Urinalysis is unnecessary in most cases of enuresis - CORRECT ANSWER c The most appropriate laboratory tests to obtain for evaluation of a neonate with jaundice presenting in the first 36 hours of life include: A) G6PD assay and an Rh type test B) Direct Coombs' test and G6PD assay C) Total and direct serum bilirubin levels and a reticulocyte count D) Maternal and infant blood group and Rh type tests and a direct Coombs' test - CORRECT ANSWER d A 17-year-old adolescent comes to the clinic because of weight loss, irritability, insatiable hunger, and heat intolerance. The PNP screens for hyperthyroidism. In the patient with hyperthyroidism: A) The TSH level is normal, and the T4 level is elevated. B) The TSH and the T4 levels are elevated. C) The TSH level is normal, and the T4 level is below normal. D) The TSH level is below normal, and the T4 level is elevated. - CORRECT ANSWER d The PNP suspects that a sexually active adolescent has pityriasis rosea. Based on the differential diagnosis, the PNP screens for: A) Chlamydia B) Secondary syphilis C) Erysipelas D) Molluscum contagiosum - CORRECT ANSWER b A 10-year-old child is brought to the clinic for what appear to be nongenital warts on the hand. The PNP treats the warts with: A) Referral to a dermatologist for laser treatment of the lesions B) Cryosurgery and referral to a dermatologist C) Topical salicylic acid applied twice a day for several weeks D) Podophyllum resin - CORRECT ANSWER c The most important aspect of bruising in relation to possible maltreatment or nonaccidental trauma is: A) Dating of the bruise B) The history of the cause of the bruise C) The location of the bruise D) The size of the bruise - CORRECT ANSWER b The PNP examines a 17-year-old adolescent with fatigue, joint stiffness in the morning, anorexia, weight loss, and a rash across the nose. The rash is bleeding into the cheeks. The PNP suspects systemic lupus erythematosus and orders: A) An ANA B) Electrophoresis C) An ESR D) A CBC and differential count - CORRECT ANSWER a The PNP sees a 7-year-old child for "an ear recheck." Clinical findings include tympanic membranes that are opaque bilaterally, with visible landmarks and light reflexes. A fluid line is visible on examination. The diagnosis of otitis media with effusion is made. Of the following treatment plans, which is the most appropriate? A) Amoxicillin 60 mg/kg three times a day for 10 days (erythromycin if the child is allergic to penicillin) B) Prednisone 10 mg/kg/day for 7 days C) ENT referral D) Observ - CORRECT ANSWER d RSV affects infants and young children and presents with: A) Sore throat, rash, and cough B) Apnea spells, poor feeding, and wheezing C) Sore throat, wheezing, and poor feeding D) Rash, lymphadenopathy, and cough - CORRECT ANSWER b A 17-year-old adolescent comes to the school-based clinic because of swelling, pain, and tenderness involving the Achilles tendon. The adolescent has been running long distances several times a week. The PNP diagnoses tendonitis and recommends rest, elevation, and: A) Heat applied to the area of the Achilles tendon B) Ice applied to the area of the Achilles tendon C) Steroids D) Ibuprofen - CORRECT ANSWER d A 7-year-old is brought to the clinic with "a cold that just won't go away" and green, thick rhinorrhea of 10 days' duration. The PNP diagnoses sinusitis and treats with: A) Amoxicillin and a decongestant B) Amoxicillin and an antihistamine C) Saline nasal drops and a decongestant D) A topical nasal steroid and cefixime (Suprax) - CORRECT ANSWER a The PNP is evaluating a 10-year-old child with a 12-day history of rash and intermittent fever. The child is alert and febrile (100.8°F) and has an annular, indurated lesion measuring approximately 16 to 20 cm on the posterior lateral thigh. There also are two small papules with 3 to 4 cm of surrounding erythema on the posterior thorax. The child reports feeling tired and "achy," primarily in the legs, but has been pursuing normal activities. The mother reports no known injuries or insect bites - CORRECT ANSWER b A 5-year-old child is brought to the clinic because of a "runny nose." The rhinorrhea is clear, and on physical examination the PNP notes that the nasal mucosa is pale, boggy, and edematous. The PNP prescribes: A) Amoxicillin B) Loratadine (Claritin) C) Erythromycin D) Fexofenadine (Allegra) - CORRECT ANSWER b At an office visit, the mother of a 15-year-old basketball player reports that the adolescent arrived home from a postgame party and was euphoric, hostile, and hyperactive. The PNP suspects use of: A) Glue B) Marijuana C) Alcohol D) Amphetamines - CORRECT ANSWER d The PNP is examining a full-term neonate with jaundice at age 15 hours. The bilirubin level is 13 mg/L. The mother tested positive for group B streptococcus, but antibiotics were not administered during labor because there was not enough time. There were no other complications during birth. The infant is breastfeeding well. Currently the only other significant finding on the infant's examination is a large cephalohematoma. The least likely cause of jaundice is: A) Sepsis B) Rh or ABO incompatibi - CORRECT ANSWER c A 5-year-old child is brought to the clinic for drop-foot gait. The child lifts the leg to avoid tripping over the drooping distal foot and toes. The PNP suspects that the child has: A) Duchenne muscular dystrophy B) Chronic sensorimotor neuropathy C) Myasthenia gravis D) Becker muscular dystrophy - CORRECT ANSWER b The PNP is examining a 16-year-old adolescent with painful vesicular skin lesions. The most likely diagnosis is: A) Molluscum contagiosum B) Herpes infection C) Coxsackievirus infection D) Impetigo - CORRECT ANSWER b During a routine sports physical examination the PNP asks a 13-year-old adolescent whose parents smoke about tobacco use. Which of the following is the best rationale for asking if the adolescent smokes? A) Smoking will inhibit athletic performance B) Children whose parents smoke are less likely to begin to use tobacco C) The average age at which tobacco use begins is 14 years D) The incidence of nicotine dependence is higher in boys - CORRECT ANSWER c A 14-year-old adolescent is brought to the clinic for primary dysmenorrhea. The PNP prescribes: A) Ibuprofen B) Acetaminophen and codeine C) Codeine D) Acetaminophen - CORRECT ANSWER a When considering child abuse or maltreatment, which of the following statements is correct? A) The most common reporters of abuse are primary care providers B) The most common perpetrator of physical abuse is the father C) Abusive deaths are most often caused by biological family members D) Disabled children are safe from abuse and neglect - CORRECT ANSWER c An 8-year-old child is in the clinic because of an inability to lift the right arm. After playing on the rock wall in the playground, the child complained to the mother of arm pain and inability to lift the arm. On examination the PNP notes decreased voluntary control of the right arm, specifically with an inability to lift the arm. There is no obvious deformity of either arm, and the child has full range of motion in the neck and other limbs. The only abnormal finding is slight tenderness in th - CORRECT ANSWER b Which of the following statements is true regarding childhood obesity? A) The most common problem associated with childhood obesity is diabetes B) If one parent is obese and the other is of normal size, 9% of their children will become obese C) Endocrine causes of obesity are common D) The older the obese child is, the more likely it is that he or she will become an obese adult - CORRECT ANSWER d A 9-year-old child is brought to the clinic for joint pain, a rash on the trunk, and a "racing heart." The child looks ill, and the skin is hot to the touch. The child had a sore throat about 3 weeks before this illness but recovered without medical treatment. The PNP orders a: A) Immunofluorescent stain to detect ANA antibodies. B) Antistreptolysin O (ASO) titer C) Monospot test D) CBC - CORRECT ANSWER b When approaching adolescents about sensitive issues, such as drug use and sex, an effective interviewing approach is to: A) Introduce the topics early in the history B) Begin by asking about their friends' activities C) Let the adolescent know that the parents are concerned D) Avoid promising confidentiality - CORRECT ANSWER b A 17-year-old adolescent attended a party and returned home. The grandmother calls the office and is concerned that the adolescent's eyes "look funny." They are puffy, the pupils are slow to react or nonreactive, and tearing is present. With these presenting signs and symptoms the PNP suspects use of: A) Cocaine B) Phencyclidine hydrochloride (PCP) C) Alcohol D) Marijuana - CORRECT ANSWER d A 3-year-old child is brought to the clinic with the complaint of vomiting; low-grade fever; and watery, frequent stools for the past 24 hours. The PNP recommends: A) Increased liquids and a modified diet of bananas, rice, applesauce, and toast B) A diet of bananas, rice, applesauce, toast, and milk C) Oral antidiarrheal agents D) Clear fluids and rest for 24 hours - CORRECT ANSWER a A 3-year-old child is brought to the clinic with painful sores in the mouth. On inspection the PNP notes two to five pin-sized vesicles covered with a yellow-gray membrane on the child's lips and in the mouth. The PNP diagnoses: A) Epstein-Barr virus B) GABHS infection C) Aphthous stomatitis D) Herpangina - CORRECT ANSWER c An 18-month-old child is examined because of a cough and runny nose lasting 7 days. The nasal discharge is yellow-green. The mother has been using over-the-counter cold medications every 6 hours. There is no fever, and the child's appetite, activity, and elimination are as usual. The mother had the same symptoms and was put on an antibiotic for a sinus infection. She is concerned that the child has the same infection. The most appropriate step for the PNP to take is to: A) Prescribe an appropria - CORRECT ANSWER c A 3-year-old child is brought to the office for the first time. The child is adopted, and little is known about the prenatal and birth history. A diagnosis of fetal alcohol effects (FAE) is suspected. The physical findings that lead the PNP to suspect this diagnosis are: A) Carious teeth, hyperactivity, toe walking, and macrocephaly B) Growth retardation, thin or wide lips, flat midface, and finger anomalies C) Abnormal hair pattern, cherubic lips, temper tantrums, and protuberant belly D) Balan - CORRECT ANSWER b Appropriate interventions for a 3-month-old infant diagnosed with sickle cell anemia include the following: A) Initiate penicillin prophylaxis, and begin folic acid supplementation B) Teach spleen palpation to parents, and administer Pneumovax vaccine C) Administer Pneumovax vaccine, and initiate penicillin prophylaxis D) Begin folic acid supplementation, and administer Pneumovax vaccine - CORRECT ANSWER a A 10-year-old child is brought to the office because of persistent fever (102ºF) lasting for 5 days. The child complains of a headache and sore throat and was seen twice by different providers in the past 5 days. An earlier rapid antigen-detection test for GABHS was negative, and a viral infection was diagnosed. Physical examination now reveals slightly enlarged anterior cervical lymph nodes and a red throat. A faint, pink, slightly raised rash is present on the abdomen and in the groin area; t - CORRECT ANSWER b The PNP examines a 4-month-old infant with hypotonia and motor delay since birth. The most significant finding indicating lower motor neuron involvement in this infant would be: A) Lack of deep tendon reflexes B) Hyperactive deep tendon reflexes C) Persistence of a Moro reflex D) Persistence of the tonic neck reflex - CORRECT ANSWER a The PNP is evaluating a 2-year-old child with a 48-hour history of a dry, coarse cough and fever. Upon examination the child appears ill but nontoxic. The child has a harsh, forceful cough with intermittent stridorous breaths after a coughing spasm but no stridor at rest. The parents have been giving the child an expectorant and cough suppressant that have provided little relief. The PNP makes the diagnosis of croup without respiratory distress. Treatment for this child should include: A) Nebuli - CORRECT ANSWER d Appropriate management of a full-term, 3-day-old, breastfed infant with a bilirubin level of 16 mg/L would consist of: A) Observation, close follow-up, parental instructions to breastfeed every 2 hours, and an office visit to determine serum bilirubin level in 10 days B) Exposure of the infant to indirect sunlight, supplementation with formula after breastfeeding if the mother's milk has not yet come in, and close clinical follow-up C) Phototherapy and follow-up in the office 10 days later to de - CORRECT ANSWER b A 4-year-old child is brought to the clinic with ulcers on the tongue and oral mucosa. There are vesicles on the palms, soles, and interdigital areas. The PNP diagnoses: A) Infectious mononucleosis B) Hand-foot-and-mouth disease C) Rubeola D) Herpangina - CORRECT ANSWER b The parents of a 6-month-old infant report that the infant is "different" from their previous child. Which of the following behaviors would alert the PNP to a diagnosis of autism? The infant: A) Is extremely passive, with little interaction with others or the environment B) Is cuddly only when held by the mother C) Initiates eye contact only with siblings D) Seems to have normal language skills - CORRECT ANSWER a The PNP is evaluating an 8-year-old child with an eye injury. About 1 hour ago the child was hit in the eye with a foreign body. When the eye is stained with fluorescein, a vertical linear abrasion is visible. The pupils are equal in size and reactive, and a positive red reflex is noted bilaterally. There are no abnormal extraocular movements. No foreign body is visible. An appropriate plan of care for this patient would include the following: A) Instill antibiotic drops, and apply patches over - CORRECT ANSWER c A dark-skinned 14-year-old adolescent comes to the school-based clinic with symptoms of a nonpruritic rash, with loss of pigment. The PNP diagnoses tinea versicolor and treats with: A) Steroid cream and erythromycin B) Selenium sulfide shampoo and systemic ketoconazole C) Systemic ketoconazole and steroid cream D) Silver sulfadiazine cream and an antihistamine - CORRECT ANSWER b A 10-year-old child is examined in the school-based clinic for unilateral chest pain that radiates to the back and abdomen. On physical examination, tenderness over the costochondral junction is noted. The PNP diagnoses that the chest pain is caused by: A) Costochondritis B) Gastric reflux C) Skeletal deformity of rickets D) A fractured rib - CORRECT ANSWER a The PNP is examining a 2-year-old child with a history of vomiting, diarrhea, and fever for the past 24 hours. The physical examination reveals a tender abdomen. The PNP should: A) Reassure the parents that the child has gastroenteritis B) Obtain a more definitive history and physical examination C) Send a stool specimen for culture of ova and parasites D) Treat with clear fluids, and call the parents in 24 hours - CORRECT ANSWER b A 9-year-old child has circumscribed, coalescent, generalized, erythematous raised lesions of various sizes. The PNP recognizes that the most common cause of hives is: A) Ingestion of foods and food additives B) Cold exposure C) Sun exposure D) Insect bites - CORRECT ANSWER a A 14-year-old adolescent has a second-degree burn along the hairline from using a curling iron. The lesions are erythematous and blistering. The adolescent complains of pain. The PNP advises: A) Icing the involved area to decrease pain and beginning a prophylactic antibiotic B) Applying silver sulfadiazine cream and icing the involved area to decrease pain C) Taking ibuprofen for pain and applying silver sulfadiazine cream D) Taking prophylactic antibiotics and ibuprofen for pain - CORRECT ANSWER c The PNP is evaluating a 15-year-old adolescent with acne. Upon examination the adolescent has papules and a few pustules on the face, chest, and back. An appropriate plan of care for this patient would include: A) A combination topical antibiotic and exfoliating agent B) Oral tetracycline and anticipatory guidance for skin care C) Oral isotretinoin and pregnancy prevention counseling D) A topical exfoliating agent - CORRECT ANSWER a The PNP examines a fair-skinned 17-year-old adolescent. The adolescent had a maculopapular, red, scaling rash on the trunk a few weeks ago and now has five scaling lesions that follow a line of cleavage confined to the trunk. The PNP diagnoses: A) Eczema B) Tinea versicolor C) Pityriasis rosea D) Contact dermatitis - CORRECT ANSWER c A 16-year-old adolescent comes to the school-based clinic with chest pain. The most common cause of cardiac pain in children and adolescents is: A) Tachycardia B) Mitral valve prolapse C) Myocarditis D) Preventricular contractions - CORRECT ANSWER b A single, 15-year-old mother, who attends high school, brings a 3-month-old infant to the clinic. The infant seems to be healthy and thriving, but the mother is vague when reporting the infant's feeding and sleeping habits. To get more accurate information about the specifics of the infant's routine and care, the PNP should: A) Ask about the day care arrangements B) Make a referral to a social service agency C) Ask the mother if she has a learning disability D) Make a referral to a home nursing - CORRECT ANSWER a A 12-year-old child reveals to the PNP during a visit for acne that she may be pregnant. In this community the pregnancy rate among females aged 15 to 19 years is 3 in 100. What is the most important area to explore? A) The source of prenatal care B) The expected date of confinement C) The possibility of sexual abuse D) Plans for telling her family - CORRECT ANSWER c The PNP is evaluating a 9-year-old child with a 2-week history of heel pain. The onset of pain coincides with the start of football practice. The child does not limp but experiences heel pain with weight bearing and plantar flexion. Laboratory findings rule out an infectious or inflammatory disorder. This presentation is consistent with a diagnosis of: A) Osteochondritis B) Osgood-Schlatter disease C) Legg-Calvé-Perthes disease D) Freiberg's disease - CORRECT ANSWER a A 2-year-old child is examined as part of a well-child visit. The mother's main concern is bowing of the child's legs, which is evident on examination. The PNP: A) Assures the mother that this is normal development B) Refers the child to an orthopedic clinic for evaluation C) Obtains a detailed family history for genu varum D) Suggests that the child may need bracing - CORRECT ANSWER b In counseling a parent regarding possible lead poisoning in the child, the PNP states that the most common source of lead is: A) Soil B) Batteries C) Lead-based paint D) Drinking water - CORRECT ANSWER c The PNP is examining a 2-month-old infant. At delivery, the infant's cord was wrapped around the neck three times. The parents are concerned that the infant is "floppy." On physical examination, the PNP notes that the deep tendon reflexes are increased, there is poor tone, and scissoring of the legs is marked. The PNP: A) Orders chromosomal studies B) Discusses the findings with the staff physician C) Orders a computed tomography (CT) scan D) Orders routine vaccinations and schedules a return vi - CORRECT ANSWER b The PNP is performing well-child examinations on 2-year-old twins. One twin is verbal and interactive, initiating play with the PNP. The other twin is nonverbal, acts fearful, and refuses to make eye contact with the PNP. The mother reports that the second twin rarely speaks intelligible words. Growth parameters are normal for both children, and the physical examinations are normal. Intervention for this family should include: A) Encouraging the mother to initiate and play verbal games with the - CORRECT ANSWER b A mother brings her 2-year-old child to the clinic for a well-child visit. The mother's chief complaint is the child's rebellious behavior. The mother says that she has tried time-out, yelling, and spanking but nothing has worked. The child has never slept through the night and still gets a bottle in the middle of the night. In the examination room the PNP observes as the child repeatedly gets into the mother's purse when she is not paying attention, despite the mother telling the child "no." Ba - CORRECT ANSWER d A neonate in the neonatal nursery has a spongy area over the parietal region of the skull. The PNP diagnoses craniotabes and: A) Assesses for hydrocephalus B) Assures the mother this is a normal variant C) Obtains a skull x-ray film D) Refers the neonate to a neurologist for evaluation - CORRECT ANSWER b A 12-year-old child is seen in the office for a yearly physical. The boy states that he has recently noticed an enlargement of his testes and scrotum. When counseling the child about what to expect next in pubertal development, the PNP states that: A) The penis will grow in length B) Changes in voice will occur C) The penis will grow in width D) Facial hair will appear - CORRECT ANSWER a A number of the PNP's adolescent patients are using natural herbal remedies. What general advice should the PNP give these patients? A) "Herbs are drugs even though they are marketed as being grown naturally." B) "Interactions between different types of herbal remedies are rare." C) "Herbs are safe because they are natural." D) "Let's look for evidence-based resources to check out the herbal remedies that you are interested in trying." - CORRECT ANSWER d The mother of a 4-year-old who was adopted at birth asks the PNP when is the most appropriate time to tell the child that he is adopted. The PNP responds: A) Do not offer the information until the child is developmentally ready at about age 10 years B) When the mother feels the child is able to understand the concept of adoption C) Wait until the child is in third grade and can share with other children his adoption story D) Most children by the age of 3 years are old enough to understand the st - CORRECT ANSWER d The PNP is examining a 6-month-old exclusively breastfed infant. The infant is growing appropriately. The mother would like to delay introducing solid foods until the infant is at least 12 months of age. The PNP tells the mother the nutrient most likely lacking if solid foods were to be delayed is: A) Vitamin B6 B) Phosphorus C) Iron D) Calcium - CORRECT ANSWER c In providing anticipatory guidance to the parents of a 5-year-old child with cerebral palsy, the PNP teaches the parents to be aware of abnormal eye movements, drooling, and shaking movements that cannot be stopped. These are all signs of: A) Syncope B) Seizures C) Febrile seizures D) Infantile spasms - CORRECT ANSWER b A 16-year-old adolescent comes to the clinic for a yearly examination. During the discussion of breast self-examination (BSE), the PNP should inform the adolescent that: A) Breast masses that are painless do not require further evaluation B) Asymmetric breast development is common in puberty C) The best time to perform a BSE is during menses D) Palpation of the nipple area is not necessary - CORRECT ANSWER b The grandmother of a 2-year-old child requests information regarding toilet training readiness. The PNP tells the grandmother that the most important factor in determining readiness to toilet train is that the child: A) Is strong willed and shows an interest in the potty B) Is able to communicate needs and follow directions C) Is not bothered by wet or soiled pants D) Wakes up from naps with soaked diapers - CORRECT ANSWER b A 3-year-old child has recently started having breath-holding spells. The parents state that an older sibling "did the same thing" and ask for help. The PNP advises the mother that after a breath-holding event she should: A) Do everything possible to keep the child from feeling any extreme emotions B) Ignore the child's behavior similar to ignoring temper tantrums C) Throw water on the child's face to stimulate breathing D) Stand and watch but do nothing while the breath-holding occurs - CORRECT ANSWER b The PNP is completing a preadmission college physical for a student entering college in the fall. The PNP discusses with the student the need for a: A) Hepatitis A vaccine B) Meningococcal vaccine C) Lyme disease vaccine D) DT vaccine - CORRECT ANSWER b A new mother requests that her infant be placed on low-iron formula. She has heard that regular formula causes constipation. The PNP discusses with the mother: A) Changing to low-iron formula and using iron drops as a supplement B) Changing to a soy-based formula, which is low in iron and more easily digested C) The need for the formula to provide adequate iron for normal growth and development D) The importance of not listening to what people who are not professionals say about how to feed and - CORRECT ANSWER c A 5-year-old who occasionally reverses letters without any other suggestion of learning problems: A) Should be referred to a neurologist immediately B) May need special education classes for dyslexia C) Will probably grow out of this but should be monitored for other signs of difficulty D) Is likely to have school problems with reading and writing - CORRECT ANSWER c A 14-year-old adolescent comes to the school-based clinic with PMS. The PNP advises the adolescent to: A) Limit intake of chocolate, fats, and sodium and increase exercise B) Limit intake of sugar and fat and decrease exercise C) Limit intake of fats and sodium and decrease exercise D) Limit intake of sugars, fats, and sodium and participate in aerobic exercise - CORRECT ANSWER d A 5-year-old child is at the clinic for a routine well-child visit before beginning kindergarten. The mother reports the child's list of chores as follows: clean own room, make own bed, wash and dry own clothing, take out the trash, and help wash the dishes. If the chores are not completed, the child is grounded from other activities, such as playing with friends or watching television. The mother states that the child frequently must be grounded. How should the PNP respond? A) Suggest use of na - CORRECT ANSWER c The mother of a 4-year-old child tells the PNP that her son "plays with himself in front of the TV." The PNP advises the mother to: A) Explain to the child that this is normal behavior, but if it continues, he will have to see a mental health professional B) Send the child to his room with no comment; the PNP should explain to the mother that this is normal behavior C) Explain to the child that masturbation is normal but should be done in private D) Ignore the behavior and make no comment to the - CORRECT ANSWER c The PNP is providing anticipatory guidance to the foster mother of a 10-month-old infant weighing 20 lb, 1 oz. The most appropriate advice the PNP can give the foster mother concerning the use of car seats is: A) "A child weighing more than 20 pounds may face forward in the car seat." B) "Keep using a rear-facing car seat until age 1 year." C) "It is safe to keep your child in a car seat in the front seat of the car as long as there is not a passenger-side airbag." D) "A booster seat should be u - CORRECT ANSWER b In counseling an adolescent vegetarian, the PNP discusses the need to ensure adequate intake of: A) Niacin, thiamine, iron, and calcium B) Vitamin C, calcium, iron, and zinc C) Calcium, vitamin D, thiamine, and iron D) Calcium, zinc, iron, vitamin B12, and vitamin D - CORRECT ANSWER d The mother of a 5-month-old breastfed infant requests information about weaning. The PNP advises the mother: A) Immediately substitute all breastfeedings with formula B) Wean the infant slowly from the breast to the bottle C) Substitute all breastfeedings with formula, and pump breasts frequently to avoid engorgement D) Continue breastfeeding for at least 1 more month - CORRECT ANSWER b A 12-month-old infant is at the clinic for a scheduled well-child visit. The mother expresses concern that the infant awakens in the middle of the night screaming. The mother refuses to let the infant cry because the crying wakes up siblings, so she rocks the infant back to sleep. In addition to telling the parents that the infant's nighttime awakening has become a habit, the PNP should offer what other advice? A) The infant should be allowed to stay up late so he or she will be tired and sleep - CORRECT ANSWER b A 4-year-old child is examined in the clinic. The parents report that the child has limited language skills and does not speak in three-word sentences. The PNP is interested in further assessing the child and chooses the following: A) Clinical Adaptive Test/Clinical Linguistics Auditory Milestone Scale (CAT/CLAMS) B) Vineland Social Maturity C) Denver Articulating Screening Examination (DASE) D) Denver II - CORRECT ANSWER c A 12-month-old is seen in the clinic for a well-child check-up. The mother expresses concern about the infant's sleep pattern. What is the average daily sleep time in infants at age 12 months? A) 6 to 12 hours B) 12 to 18 hours C) 10 to 15 hours D) 3 to 7 hours - CORRECT ANSWER c What advice would the PNP give a parent about fruit juice during the 4-month well child visit? A) "Offer any juice except orange juice now." B) "Begin to offer orange juice now." C) "Do not offer the infant fruit juice now." D) "Begin to offer juice to the infant now." - CORRECT ANSWER c The mother of a 4-month-old infant is planning to return to work. The mother is investigating licensed day care centers and asks about the appropriate ratio of infants to adults. The PNP responds: A) Three infants to one adult B) Eight infants to one adult C) Five infants to one adult D) Four infants to one adult - CORRECT ANSWER a A mother is asking the PNP about puberty in her daughter. The PNP advises the mother that: A) Papanicolaou testing should begin at age 16 years B) Pubic hair development precedes breast budding C) Menses occur about 2 years after thelarche D) The adolescent growth spurt begins after pubic hair development - CORRECT ANSWER c A 5-year-old child with a gross motor deficit is brought to the office for a preschool physical examination. The child's language skills are normal. The parents ask about potential learning difficulties. The PNP stresses the importance of being vigilant and involved with school to optimize the child's success. The PNP also informs the parents that: A) The child's language skills are a strength because language performance is the skill most highly correlated with school success B) It is impossibl - CORRECT ANSWER a A mother asks about the term Moro reflex. The PNP explains that the reflex is present in a neonate and disappears or phases out at about age: A) 6 to 8 months B) 3 to 5 months C) 7 to 9 months D) 10 to 12 months - CORRECT ANSWER b The father of a 2-year-old child calls the office, requesting help in managing bedtime resistance. The PNP suggests that children should be expected to: A) Set their own bedtime based on their internal time clock B) Put themselves to sleep somewhere other than their own bed and then be carried to bed once asleep C) Make ongoing requests, requiring the parent to come to the room and meet those demands D) Protest going to bed but parents should be consistent in expecting the child to stay in bed - CORRECT ANSWER d The mother of a 2-year-old child is concerned about the child's thumb sucking during the day when not playing and at night while sleeping. The PNP suggests that: A) The parents should ignore the behavior because it is not harmful at this age B) The mother should obtain an orthodontic device for the child's mouth to discourage the sucking behavior C) This is not a problem unless the thumb sucking continues past age 3 years D) The parents should send the child to time-out when caught thumb sucking - CORRECT ANSWER a A 12-year-old girl has questions about the potential for her to get much taller. She aspires to be a professional basketball player. On examination she is in Tanner stage IV. Menarche was 1.5 years ago. In considering answers to her questions, it is important to realize which of the following? A) Girls reach peak height velocity immediately before menarche B) She will not reach her peak height velocity until she is in Tanner stage V C) Menarche occurs an average of 2 years after thelarche D) Boy - CORRECT ANSWER a At the 6-month well-child visit the PNP is counseling a mother about what foods to add to the infant's diet. The mother asks if she can offer cow's milk in a cup. The PNP informs the mother that cow's milk: A) Is safe and nutritious B) Has too much fat for the infant C) Should not be introduced to an infant D) Is more expensive than formula - CORRECT ANSWER c A 7-month-old infant is brought to the clinic for a well-child visit. The infant has attained the expected developmental milestones for age. The physical examination is normal, and the infant appears to hear well. The mother is concerned that the infant no longer gibbers and imitates sounds as in past months. The PNP responds: A) "We need to administer the Denver II before providing a referral to an audiologist." B) "The child's hearing needs to be checked, and I'll give you a referral to an aud - CORRECT ANSWER c A 3-year-old child is brought to the clinic for a well-child visit. The mother reports that the child has recently begun waking up in the middle of the night screaming and appearing frightened and requests information on how to deal with nightmares. The PNP responds: A) Anxious parents tend to have fearful, anxious children B) The mother should ignore the behavior and not go to the child's room C) The mother should not allow the child to relive the nightmare D) The mother should listen to the ch - CORRECT ANSWER d A 13-month-old child is brought to the clinic for a well-child visit. The child is in the 15th percentile for weight and the 50th percentile for height. The head circumference is in the 50th percentile. The child drinks about 23 oz of skim milk per day and drinks about 20 oz of juice per day. How would the PNP guide this parent? A) "The child should not be allowed to drink all that skim milk and juice." B) "The height is at the 50th percentile, but the weight is at the 15th percentile; give the - CORRECT ANSWER d A 13-year-old is in the clinic for her yearly checkup and asks the PNP when she will start her menstrual periods. Her breast and pubic hair development are Tanner stage II and her height is at the 25th percentile. The best response to her is: A) "You need to be seen by an endocrinologist immediately." B) "Looks like you are going to be a late bloomer." C) "You will probably begin in the next 6 months." D) "Tell me more about your concerns." - CORRECT ANSWER d A 3-year-old child is brought to the clinic for a well-child visit. The mother requests help with toilet training. The child refuses to use the toilet to have a bowel movement and squats, wearing a diaper, in a secluded corner to pass a stool. The parents offer encouragement through rewards and the "big kid" approach to using the toilet, but nothing seems to help. The PNP should respond that: A) Refusal to use the toilet after age 2 1/2 years is the result of caregivers (parents, day care worker - CORRECT ANSWER c The PNP is returning a phone call to a parent who has questions regarding the use of over-the-counter medications for a 2-year-old child. The mother reports that the child has been running a temperature of 101.2°F with no other symptoms of illness. The mother has been treating the fever by alternating doses of acetaminophen and ibuprofen every 3 to 4 hours. The child is playful and has a normal appetite. Appropriate counseling for this mother should include: A) Continue giving the medicines eve - CORRECT ANSWER c When asked at a prenatal visit by the mother, "How should I feed my infant—breast or bottle?" How should the PNP reply? A) "Breastfeeding is the best way to feed the infant." B) "A combination of breast and bottle is best." C) "Formula is an easy way to offer adequate nutrition." D) "Feed the infant whichever way you want." - CORRECT ANSWER a When speaking to a fifth grade class about injury prevention, based on the national top five causes for fatal injuries in this age group, which of the following topics would the PNP emphasize? A) Bike safety (wearing a helmet) and drowning prevention B) Drinking alcohol and driving, and safe sex C) Sniffing glue and smoking prevention D) Drowning prevention and substance abuse - CORRECT ANSWER a A child who is able to run, kick a ball, wash and dry the hands, tower two cubes, and use pronouns indiscriminately is probably aged: A) 24 months B) 18 months C) 40 months D) 36 months - CORRECT ANSWER a A 16-year-old female adolescent with Down's syndrome, who is healthy and has had no major medical problems, comes to the clinic for a well-child visit. The adolescent has been "mainstreamed" into a regular classroom and is doing well. The physical examination reveals Tanner stage III breast development and pubic hair. Today the PNP should: A) Order cervical spine x-ray films, a vision screen, and echocardiography B) Perform hearing and vision screens, and refer the adolescent to a long-term plan - CORRECT ANSWER c Children born with cleft lip/palate are prone to: A) GER B) Pyloric stenosis C) Milk bottle caries D) Nasal allergies - CORRECT ANSWER c One of a toddler's greatest fears is: A) Not being accepted by peers B) Not getting enough food C) Separation from parents or caregivers D) Having limits set by caregivers - CORRECT ANSWER c The mother of an 18-year-old with epilepsy brings her to the clinic for family planning. The PNP discusses the options and recommends: A) An intrauterine device (IUD) B) Levonorgestrel (Norplant) C) Oral contraceptive pills (OCPs) D) Medroxyprogesterone (Depo-Provera) - CORRECT ANSWER d A 2-year-old child is brought to the clinic for the first time. The child has microcephaly, age-appropriate weight and height, and developmental functioning below expectations in all categories on the Denver II. The PNP should: A) Refer the child to an early intervention program B) Refer the child to a speech pathologist C) Order chromosomal studies and a CT scan D) Repeat Denver II screening in 1 month - CORRECT ANSWER a Which of the following is the major cause of death among adolescents and young adults aged 15 to 24 years? A) Homicide B) Motor vehicle-related accidents C) Acquired immunodeficiency syndrome (AIDS) D) Suicide - CORRECT ANSWER b The PNP examines a 12-month-old child for routine care. Of the following fine motor milestones, which would be most characteristic of a 12-month-old child? A) Uses a neat pincer grasp B) Transfers objects between hands C) Scribbles spontaneously D) Builds a tower of 4 cubes - CORRECT ANSWER a The grandmother of an 8-month-old infant brings the infant to the clinic for a well-child visit. The infant missed both the 4- and 6-month examinations and is behind on immunizations. The grandmother states that the parents were substance abusers during the prenatal period and are still using drugs and alcohol. At this visit, appropriate care and guidance should include: A) Growth assessment, Denver II screening, and immunizations B) Growth and developmental assessment, referral for genetics eva - CORRECT ANSWER d The PNP is seeing a 4-year-old Romanian child for an initial checkup. A local couple, whose other two children also come to the clinic for care, recently adopted the child. The child had been living in a state-operated orphanage, and no medical records are available. Today's visit should include the following: A) Physical examination and referral for counseling with a counselor specializing in adoptive families B) Complete physical examination and an attempt to obtain previous medical records C) - CORRECT ANSWER d A 2-year-old child screened for lead poisoning has a lead level of 14 mcg/dL. The intervention provided by the PNP should include: A) A long bone x-ray film for lead B) A low-calcium diet C) Referral to the health department for environmental investigation D) A high-calcium diet - CORRECT ANSWER d A 5-year-old child is brought to the clinic for a prekindergarten physical. The father has refused the MMR vaccine, stating that the child is allergic to eggs. Upon further questioning, it is determined that the child experienced stomach cramps and diarrhea after eating eggs. The child has had 4 DTaP, 3 IVP, 3 Hep b, 3 Hib, a PPD, and varicella. What immunizations would be appropriate to administer to this child today? A) DTaP, IPV, MMR, and varicella, as well as a PPD test (if warranted) B) DTa - CORRECT ANSWER a A child diagnosed with HIV can best be monitored by assessing: A) CD4 antigen counts and viral burden B) Anorexia and fatigue C) ELISA results D) Weight loss and fatigue - CORRECT ANSWER a A 14-year-old adolescent has been diagnosed with early hypothyroidism based on increased TSH and decreased T4 levels. The PNP consults with the physician and in discussing the management decides to: A) Increase the dietary intake of iodized salt B) Refer to an endocrinologist C) Treat with levothyroxine D) Recheck levels in 2 months - CORRECT ANSWER c A 7-year-old child is brought to the clinic for a routine examination. The child likes to skateboard and does it almost every day. The PNP reviews specific injury prevention related to skateboarding. The most common injuries that occur during skateboarding are: A) Injuries to the trunk B) Head and neck injuries C) Injuries or fractures of the lower extremities D) Injuries or fractures of the upper extremities - CORRECT ANSWER d The PNP is performing a neonatal examination in the nursery. A pustular type of eruption is noted on the neonate's face and trunk; some of the lesions are transient. The PNP diagnoses: A) Streptococcal infection B) Staphylococcal infection C) Erythema toxicum D) Herpes infection - CORRECT ANSWER c The PNP is examining a 14-year-old adolescent with a strained ankle received during football practice. Which of the following is true regarding sports-related injuries? A) Wearing appropriate protective equipment reduces the injury risk B) Rapid growth phases in which muscle growth is greater than bone growth cause increased clumsiness and increase the injury potential C) Children younger than 14 years have twice the injury risk of those aged 14 years and older D) Overuse injuries cause less per - CORRECT ANSWER a The first step in the process of developing an IEP for a child with disabilities is to: A) Perform a comprehensive assessment of the child by a multidisciplinary team B) Perform a battery of psychological tests that are valid and reliable C) Perform a complete history and physical examination to identify organic causes D) Provide government funds to the child and family to supplement income - CORRECT ANSWER a While examining a 3-year-old child for a well-child visit, labial adhesions are noted. The PNP discusses with the mother: A) Surgical treatment of the adhesions B) Proper hygiene C) Concern of sexual abuse D) Causes of UTI - CORRECT ANSWER b The PNP notes anal fissures on the physical examination of a 2-year-old child. There is no history of constipation. The PNP is concerned about: A) Ulcerative colitis B) Crohn's disease C) A foreign body D) Sexual abuse - CORRECT ANSWER d The PNP is speaking with a sixth grade class about injury prevention. Which of the following topics would the PNP emphasize? A) Bike safety and wearing a helmet, drowning prevention B) Drowning prevention, substance abuse C) Drinking alcohol and driving, safe sex D) Sniffing glue, smoking prevention - CORRECT ANSWER a A 6-month-old infant is brought to the clinic for a well-child visit. On physical examination, increased tone and lower leg scissoring are noted. The PNP recognizes this as a probable sign of: A) Down's syndrome B) Cerebral palsy C) Fragile X syndrome D) Muscular dystrophy - CORRECT ANSWER b The PNP is performing a preschool physical examination on a 4-year-old child who is new to the practice. A bifid uvula is noted, and on further examination of the back of the hard palate, the PNP finds an abnormal contour. The PNP decides to: A) Check the child's gag reflex and determine whether the child has tonsils B) Determine whether the child's thyroglossal cranial nerve is intact C) Test the child's speech and hearing D) Test the child's hearing and ability to lateralize sound - CORRECT ANSWER c A child who has a 250-word vocabulary, uses three-word sentences, walks upstairs alternating steps, rides a tricycle, and uses a spoon well is approximately aged: A) 3 years B) 6 years C) 5 years D) 4 years - CORRECT ANSWER a The first sign of puberty in girls is usually: A) Acne B) Pubic hair growth C) Thelarche D) Menarche - CORRECT ANSWER c Which of the following best describes the Denver II? A) It screens fine and gross motor, expressive and receptive language, and cognitive academic skills by direct observation of children aged 2 to 6 years B) It assesses receptive and expressive language based on observation and parental interview of infants and children aged 1 to 36 months C) It assesses receptive and expressive language by interview, observation, and test administration for infants and children from birth to age 36 months D) I - CORRECT ANSWER d At an 18-month-old child's well-child visit the mother states that the child is a picky eater and has 4 oz of milk by bottle at nap time and 8 oz of milk by bottle at bedtime. The rest of the child's fluids are taken by cup. Of the following, what is the most pertinent information for the PNP to obtain? A) The total intake of milk for 24 hours B) The family's financial status C) A list of foods the child refuses D) A 24-hour recall diet - CORRECT ANSWER d The mother of a preschool-aged child is discussing intellectual giftedness and asks the PNP what is the best predictor and most frequently noted indicator of an intellectually gifted child. The PNP responds: A) Ability to entertain self B) Early language development C) Extreme curiosity D) Participation in an early enrichment program - CORRECT ANSWER b The PNP is examining a 12-year-old female child with enlarged breasts and areolas and curly, dark, coarse pubic hair. The PNP uses Tanner staging and classifies this child as: A) Tanner stage V B) Tanner stage II C) Tanner stage IV D) Tanner stage III - CORRECT ANSWER d The PNP should consider referring for a more comprehensive developmental evaluation a child who: A) Has explosive temper tantrums at age 19 months B) Does not speak by age 15 months C) Has lost the ability to perform previously attained developmental milestones D) Has consistently tracked in the 5th percentile for growth since birth - CORRECT ANSWER c A 2-week-old infant is brought to the clinic for a well-child visit. During the interview, which of the following questions would be most appropriate to ask the parent? A) Do you have any questions about your car seat and how to use it? B) Is your hot water heater set at 120°F or lower? C) Have you installed safety locks or window guards on all windows? D) Do you use sunscreen to prevent sunburn? - CORRECT ANSWER a A 4-year-old child is brought to the office for a preschool physical examination. The PNP asks the child to sit on the floor in a cross-legged position and then get up without using the hands. The child is unable to do so and rolls onto all four extremities before standing. The PNP is concerned that the child may have a: A) Developmental problem B) Cognitive deficit C) Neurologic deficit D) Neuromuscular disorder - CORRECT ANSWER d Which of the following parental responses would discourage sibling rivalry? A) Serving as role models by solving conflicts in a nonconfrontational (nonaggressive) manner B) Comparing qualities of the siblings C) Labeling children in negative and positive roles D) Favoring one child over the other - CORRECT ANSWER a Unintentional injuries are the major cause of death in preschool-aged children. Of the following, which causes the most deaths in preschool-aged children? A) Motor vehicle injury B) Burns C) Drowning D) Poisoning - CORRECT ANSWER a The normal motor development of a 2-month-old infant includes: A) Grasping a rattle and lifting the head and shoulder off the bed when in the prone position B) Briefly fixating on a face or object and grasping an object when placed in the palm of the hand C) Reaching for objects, transferring objects from one hand to the other, and lack of head lag when pulled to a sitting position D) Bearing weight on the forearms when in the prone position, rolling from the prone to the supine position, and be - CORRECT ANSWER a A PNP is following a child placed in foster care. The most common health problem of children in foster care is: A) Parasitic infection B) Neurological disorders C) Respiratory disease D) Growth failure - CORRECT ANSWER c An infant's neurologic status is being evaluated. The PNP preceptor asks when the parachute developmental reflex occurs. The PNP student responds: A) Age 8 to 10 months B) Age 2 to 4 months C) Age 5 to 7 months D) Age 4 to 6 months - CORRECT ANSWER a The PNP is examining a sexually active 17-year-old boy who admits to having unprotected sex over the last several weeks. Appropriate care for this patient includes: A) Evaluation of immunization status, hepatitis B screening panel, HIV screen, and penile smear for gonorrhea B) Complete blood count and penile smear for gonorrhea C) Physical examination, STD history and screening tests, immunization update, anticipatory guidance regarding responsible and safe sexual behavior, and recheck in 6 mont - CORRECT ANSWER c The PNP is evaluating a postterm (42 weeks' gestation) neonate in the nursery. On the physical examination the PNP would expect to find: A) An elevated weight/length ratio B) Smooth soles of the feet C) Excessive vernix caseosa D) Cracked, peeling skin - CORRECT ANSWER d At a well-child visit the parents report that the child, who is now approximately age 2.5 years, frequently rocks back and forth and has done so since about age 6 months. The physical examination is normal, as is the developmental history. The most likely cause of the rocking behavior is: A) Emotional deprivation B) Cerebral dysfunction C) Autism D) Normal variation - CORRECT ANSWER d In the event of a malpractice lawsuit against the PNP, several elements must be proved. Included in these elements are the following: the PNP owed the plaintiff a duty or responsibility, the PNP's conduct or care fell below a recognized standard of care, and: A) The PNP operated outside a recognized standard of care B) The PNP was not properly credentialed C) The PNP did not consult with a supervising physician D) The plaintiff was injured by the PNP's conduct - CORRECT ANSWER d The PNP has recently read a report, published by the Committee on the Quality of Health Care in America, about medical errors in office practice. Which of the following suggestions should the PNP make to the managed care administration to decrease medical errors in the clinical area? A) Require that health care providers tell the patient one time the medications being prescribed B) Encourage health care providers to dictate all chart entries C) Stress the need for health care providers to write - CORRECT ANSWER c Public Laws 94-142 and 101-476 mandate that a child with disabilities is entitled to: A) An individualized education program (IEP) B) Genetic evaluation and counseling C) Benefits provided by the Supplemental Security Income Program for the Aged, Blind, and Disabled D) Evaluation with the Wechsler Intelligence Scale for Children III (WISC III) - CORRECT ANSWER a In the health care system the major perceived barrier to use of a PNP's services in a primary health care setting is: A) Lack of quality of service provided B) Organized nursing C) Organized medicine D) Lack of consumer confidence - CORRECT ANSWER c Title V of the Social Security Act of 1935 was amended in 1981 and 1986 to expand coverage for children with special health care needs under: A) The Supplemental Security Income (SSI) Program for the Aged, Blind, and Disabled B) Title XIX of the Social Security Act C) The Maternal and Child Health Services Block D) The U.S. Public Health Service - CORRECT ANSWER c The first nurse practitioner program was designed to: A) Manage severely ill children at Colorado General Hospital B) Provide limited access to health care for low-income children in Colorado C) Provide increased access to health care for children in Colorado D) Provide emergency care to children in Colorado - CORRECT ANSWER c The most common form of reimbursement for services provided by PNPs is: A) Fee charged at 85% of the physician's payment B) Same fee as physician, "equal pay for equal service" C) Fee for service D) Fee charged at 90% of the cost under the physician's billing - CORRECT ANSWER c The PNP is employed by a physician and performs services under direct supervision of the physician; these services provided by the PNP are billed under the physician's name and are called: A) Incidence to service B) Salary C) Direct billing for service D) Capitation reimbursement - CORRECT ANSWER a 4. Although relatively rare in the U.S., diphtheria can occur among under-immunized children. Which of the following clusters of signs, symptoms, and physicals findings would suggest diphtheria in a child presenting with upper respiratory complaints? a. Low grade fever, sore throat, nasal discharge, grayish-white pseudo-membrane in his/her throat b. Abrupt onset of high fever, severe sore throat, nasal discharge, and grayish-white pseudo-membrane in his/her throat c. Low grade fever, abrupt onse - CORRECT ANSWER a 5. Infants younger than 6mo of age with pertussis frequently require hospitalization to manage: a. Fever, cough, dehydration b. Coughing paroxysms, apnea, cyanosis, feeding difficulties c. Coughing paroxysms, dehydration, renal failure d. Seizure, fever, pneumonia - CORRECT ANSWER b 7. Which of the following symptoms are characteristic of rubella? a. Vesicular, crusted lesions & high fever b. Postauricular lymphadenopathy & low grade fever c. Intense pruritus, usually in finger webs, buttocks, thighs, & ankles d. Rough textured maculopapular rash that blanches with pressure - CORRECT ANSWER b 8. Although uncommon, a potential sequela of rubella may include: a. Pneumonia & chronic otitis media b. Arthritis, thrombocytopenia & encephalitis c. Oophoritis & infertility d. Arthritis, carditis, and neurological involvement - CORRECT ANSWER b 9. Rubeola is: a. Preventable by active immunization b. Caused by human herpesvirus 6 c. Treated with intravenous acyclovir d. Not associated with severe complications (e.g., encephalitits, pneumonia - CORRECT ANSWER a 10. You are examining a child who has fever, coryza, cough, conjunctivitis, malaise, & anorexia. During the oral examination, you observe red eruptions with white centers on the buccal mucosa. What are these eruptions called? a. Pastia's spots b. Rubelola spots c. Koplik's spots d. Strawberry spots - CORRECT ANSWER c 11. Which of the following best describes the treatment for roseola? a. Acetaminophen or ibuprofen for fever, parental reassurance b. Warm compresses for salivary gland swelling c. Oral acyclovir, 20mg/kg/dose, four times/day d. Bed rest, saline gargles for sore throat - CORRECT ANSWER a 12. Fifth disease is usually: a. Seen in age 5-14yo children b. Transmitted via the deer tick c. Treated with oral erythromycin d. Characterized by prolonged coughing episodes - CORRECT ANSWER a 13. Which of the following statements is not true regarding the transmission of chickenpox? a. Susceptible individuals can contract chickenpox from patients with varicella zoster (shingles) b. Children with chickenpox are infectious only during the period of time when skin lesions are present c. Children with chickenpox are no longer infectious once crusting of skin lesions has occurred d. Varicella-zoster immune globulin (VZIG) should be administered to susceptible immunocompromised individuals - CORRECT ANSWER b 14. A child with chickenpox and temperature of 102F should receive which medication for fever? a. Aspirin b. Amoxicillin c. Acetaminophen d. Acyclovir - CORRECT ANSWER c 15. The most appropriate agent for use in treating varicella zoster infection in an immunocompromised host is: a. Ganciclovir b. Acyclovir c. Cetriaxone d. Chloramphenicol - CORRECT ANSWER b 16. Varicella zoster infection is most commonly associated with which of the following skin lesions? a. Vesicle b. Comedone c. Nodule d. Macule - CORRECT ANSWER a 17. Which of the following is not a complication of mumps? a. Meningitis b. Pneumonia c. Oophoritis d. Pancreatitis - CORRECT ANSWER b 18. What recommendation would you make to a parent whose son has been diagnosed with mumps and wants to know when he can return to child-care? a. He can return once he becomes afebrile and can tolerate eating b. He can return 9 days after onset of symptoms c. He can return when he is well enough participate in activities d. He can return after a minimum of 5 days of antibiotic therapy - CORRECT ANSWER b 20. Which of the following describes a characteristic rash associated with which disease? Initially erythematous and macular, becoming maculopapular and petechial. The rash first appears on the wrists & ankles, spreading proximally to the trunk. The palms and soles are often involved. a. Lyme disease b. Roseola c. Rubeola (measles) d. Rocky mountain spotted fever - CORRECT ANSWER d 22. Which of the following factors is not associated with increased risk for infantile botulism? a. Rural environments b. Use of honey c. Use of corn syrup d. Farm families - CORRECT ANSWER c 24. Which of the following are associated with paralytic poliomyelitis? a. Lacy, erythematous, pruritic rash b. Respiratory compromise, speech disturbances, urinary incontinence c. Abdominal swelling, lympthadenopathy, and jaundice d. Nonspecific abdominal pain, nausea, and vomiting - CORRECT ANSWER b 25. Muscle spasms associated with tetanus are aggravated by which of the following? a. Fever b. Tetanus immunoglobulin c. External stimuli d. NSAID - CORRECT ANSWER c 26. Classic symptoms associated with malaria include: a. Low grade fever, upper respiratory congestion, cough b. Annular rash, conjunctivitis, headache, arthralgia c. High fever, chills, rigors, sweats, headache d. High fever, jaundice, lethargy, vomiting - CORRECT ANSWER c 27. Lyme disease is most closely associated with which of the following skin lesions? a. Erythema migrans b. Nodule c. Scale d. Pustule - CORRECT ANSWER a 28. Many infectious diseases present with rashes along with general complaints of fever, malaise, and headaches. Which of the following clusters of symptoms would make you consider Lyme disease as a likely diagnosis? a. Fever, malaise, headace, arthalgia, and well circumscribed, erythematous, annular rash with central clearing b. Fever, malaise, headache, transient bone pain, and generalized erythematous, maculopapular rash that began on the face and spread to trunk and extremities c. Fever, mal - CORRECT ANSWER a 29. Which of the following would be included in patient education regarding lyme disease? a. Educate caretakers regarding complications, including hypertension and renal calculi due to immobility b. Avoid use of aspirin-containing products for fever control due to association with increased risk for Reye's syndrome c. Protective clothing and tick repellent should be worn in heavily wooded areas d. Educate caretakers regarding natural history of the illness, and specific follow up needed after in - CORRECT ANSWER c 13. the following blood level is not considered lead poisoning? a. <10ug/dl b. 10-14ug/dl c. >15ug/dl d. >25ug/dl - CORRECT ANSWER a 14. Which of the following is not a precipitating factor for hemolysis in G6PD deficiency? a. Drugs b. Exposure to extreme temps c. Ingestion of fava beans d. Infection - CORRECT ANSWER b 15. What percent of factor VII/IX is associated with severe hemophilia A & B? a. >1 b. 1-5 c. 5-25 d. 30-50 - CORRECT ANSWER a 16. What type of hemorrhage would be expected with severe factor VIII deficiency? a. Severe hemorrhage following moderate to severe trauma b. Gross bleeding following mild to moderate trauma c. Gynecologic hemorrhage d. Spontaneous hemarthosis - CORRECT ANSWER d 17. Which of the following is the most common type of congenital bleeding disorder? a. Hemophilia A b. Hemophilia b c. Von Willebrand Disease d. Idiopathic thrombocytopenia purpura - CORRECT ANSWER c 26. The following is not a characteristic feature of DiGeorge syndrome: a. Hypertelorism b. Cleft palate c. Cardiac defect d. Frontal bossing - CORRECT ANSWER d 27. The following diagnostic finding is consistent with Wiskott-Aldrich syndrome: a. IgG-normal b. IgA-decreased c. IgM-increased d. B cells-decreased - CORRECT ANSWER a 10. Which one of the following is not characteristic of constitutional growth delay? a. There is generally no hx of a similar growth pattern in other family members b. The child usually remains constitutionally small as an adult c. Final adult stature tends to be normal d. Weight & height at birth are generally in the lower percentiles - CORRECT ANSWER c 11. A newborn or infant with birth length <50% and microphallus should be suspected of having: a. Growth hormone deficiency b. Congenital hypothyroidism c. Primordial short stature d. Down syndrome - CORRECT ANSWER a 12. An adolescent male who fails to develop secondary sex characteristics at puberty and who has small, underdeveloped testes should be suspected of having: a. Adrenal hyperplasia b. Klinefelter's syndrome c. Marfan syndrome d. Cerebral gigantism (sotos syndrome) - CORRECT ANSWER b 17. Which one of the following is not found in children with growth hormone excess? a. Tall stature b. Prominent mandible & supraorbital ridge c. High or normal plasma growth hormone d. Short stature - CORRECT ANSWER d 18. A pathognomonic skin finding in children with chronic adrenal insufficiency (Addison's) is: a. Purple striae b. Increased pigmentation in the axilla, groin, areola, hand creases, & in surgical scars c. Dry, thickened skin d. Increased perspiration - CORRECT ANSWER b 1. which of the following is a characteristic physical sign of fragile X syndrome in adolescent males? a. Small posteriorly rotated ears b. Macroorchidism c. Hypertonia d. Double hair whirl - CORRECT ANSWER b 2. Which of the following physical stigmata are common in newborns with Down syndrome? a. Microcephaly, large ears & mouth, flattened philtrum b. Hypotonia, large appearing tongue & small mouth, upward slant to eyes c. Fair mottled skin, large hands & feet, broad stocky neck d. Funnel or pigeon-breasted chest, Brushfield spots, extra digits - CORRECT ANSWER b 3. a 2mo infant, with hx of sacral myelomeningocele repair, has an increase in head circumference from the 75th to the 95th percentile. What is the most appropriate first action? a. Order a stat head CT scan b. Refer to neurosurgery for management c. Recheck it at the next well-child visit d. Recheck & replot the child's head circumference - CORRECT ANSWER d 4. a 2 week old presents with mucopurulent eye discharge with injection and edema of the conjunctiva. The 15yr old mother had no prenatal care. What is the most likely organism? a. Staphylococcus aureus b. Gonococcus c. C. pneumoniae d. C. trachomatis - CORRECT ANSWER d 5. a newborn presents with a large VSD, rocker bottom feet, overlapping second & third fingers, & fourth & fifth fingers with hypotonia. What is the most likely diagnosis? a. Fragile X syndrome b. Down syndrome c. Edwards Syndrome d. Klinefelter's syndrome - CORRECT ANSWER c 6. Which of the following problems is common in a child with Hurler's syndrome? a. Developmental delay from birth b. Sleep obstructive apnea c. Ectopic lentis d. Congenital heart disease - CORRECT ANSWER b 7. Young infants with cerebral palsy often show: a. Voracious appetite & weight gain b. Increased muscle tone in the first weeks of life c. Hypotonia in the first weeks of life d. Unusually severe reactions to their first immunizations - CORRECT ANSWER c 8. A 5yo child is born at 28 weeks gestation has mild spastic diplegia. Which evaluation is important before the child is placed in regular kindergarten? a. Stanford-binet IQ test b. Carey scale of temperament c. CBC with diff d. Chromosome studies - CORRECT ANSWER a 9. A mother with Marfan syndrome comes with her 5yo for a checkup. The child has myopia & a + wrist & thumb sign. She reports that she was told by her last health care provider not to worry about this child. What is the next best step? a. Raise the issue of a genetic referral b. Reassure her c. Follow the child for further signs of Marfan syndrome d. Refer to ophthalmology - CORRECT ANSWER a 10. Which of the following is appropriate advice for the mother of a newborn? a. The child should sleep on her back b. The child should sleep on her back or side c. The child should be allowed to sleep in a carseat at night d. The child can sleep on her comforter - CORRECT ANSWER a 11. A 5yo female presents for a school physical with a complaint of hyperactivity, a mild developmental delay, aversion of gaze, hand mannerism, long thing face with slightly dysmorphic ear. What is the likely diagnosis? a. Fragile X b. Turner's Syndrome c. Fetal alcohol syndrome d. Williams syndrome - CORRECT ANSWER a 12. Congenital HIV infection: a. Is diagnosed through finding maternal antibodies in infant serum b. Does not respond to antiretroviral therapy c. May be latent for years before clinical signs develop d. Does not include lymphadenopathy as a physical finding - CORRECT ANSWER c 13. an infant presents with cataracts, congetinal glaucoma, congenital heart disease, hepatosplenomegaly, thrombocytopenia, blueberry muffin rash, & growth retardation. What is the most likely diagnosis? a. Cytomegalovirus b. Taxoplasmosis c. Rubella syndrome d. Syphilis - CORRECT ANSWER c 14. A well-appearing, well dressed 11mo old has a long philtrum, midface hypoplasia, microcephaly, mild developmental delay, & myopia. The mother denies drinking alcohol during the pregnancy. What is the next best step? a. Refer the mother-child to child protective services b. Refer the child to early intervention c. Refer the child to cardiology d. Refer the mother to alcoholics anonymous - CORRECT ANSWER b 15. An 18mo old child exposed to HIV in utero had negative HIV DNA at 2 weeks and 8 weeks. What is indicated at the 18-month visit? a. No further testing is needed b. HIV DNA should be repeated c. Enzyme immunoassay for antibody to HIV-1 d. CBC with diff & immunoglobulins - CORRECT ANSWER c 17. A baby is born to a mother with chronic hepatitis B. what is the best treatment approach? a. Administer interferon within 24hrs b. Administer hepatitis B vaccine within 24hrs c. Administer interferon & nucleotide in combination daily d. Administer HBIG & Hepatitis B vaccine within 12 hours - CORRECT ANSWER d 18. a 2yo female has lymphedema of the hands & foot, with low posterior hairline, cubitus valgus, & a hx of intrauterine growth retardation. Which of the following defects is the most common among the children with this defect? a. Aortic valve stenosis b. Mitral valve prolapse c. Dissecting aortic aneurysm d. Coarctation of the aorta - CORRECT ANSWER d 19. Joshua is the 9mo old infant son of parents of Lousinana-French descent. The child stopped rolling over. Mother reports he is increasingly irritable. Which of the following physical exam findings is most consistent with Tay-Sachs disease? a. Cardiomyopathy b. Retinal detachment c. "cherry red" spot on retina d. hepatomegaly - CORRECT ANSWER c 20. Prader-Willi syndrome is a congenital genetic disorder characterized by: a. Failure to thrive b. 100% detection rate with chromosome analysis for a 15q deletion c. emergence of spasticity during toddler years d. voracious appetite & development of obesity - CORRECT ANSWER d [Show More]

Last updated: 6 months ago

Preview 1 out of 167 pages

Reviews( 0 )

$11.00

Add to cart

Instant download

Can't find what you want? Try our AI powered Search

OR

GET ASSIGNMENT HELP
84
0

Document information


Connected school, study & course


About the document


Uploaded On

Nov 08, 2023

Number of pages

167

Written in

Seller


seller-icon
Nolan19

Member since 2 years

10 Documents Sold


Additional information

This document has been written for:

Uploaded

Nov 08, 2023

Downloads

 0

Views

 84

Recommended For You


$11.00
What is Browsegrades

In Browsegrades, a student can earn by offering help to other student. Students can help other students with materials by upploading their notes and earn money.

We are here to help

We're available through e-mail, Twitter, Facebook, and live chat.
 FAQ
 Questions? Leave a message!

Follow us on
 Twitter

Copyright © Browsegrades · High quality services·